om exam 1 chap 1-5

187
8/13/2019 OM Exam 1 Chap 1-5 http://slidepdf.com/reader/full/om-exam-1-chap-1-5 1/187 Operations Management, 10e (Heizer/Render) Chapter 1 Operations and Productivity True/False 1) Some of the operations-related activities of Hard Rock Café include designing meals and anal!ing them for ingredient cost and la"or re#uirements$ %ns&er' TR( *iff' 1 Topic' +lo"al compan profile ,"ective' .,1-1 ) 0ecause Hard Rock Cafes are themed restaurants operations managers focus their laout design efforts on attractiveness &hile paing little attention to efficienc$ %ns&er' F%.S *iff' 1 Topic' +lo"al compan profile ,"ective' no ., 2) %ll organi!ations including service firms such as "anks and hospitals have a production function$ %ns&er' TR( *iff' Topic' 3hat is operations management4 ,"ective' .,1-1 5) ,perations management is the set of activities that creates value in the form of goods and services " transforming inputs into outputs$ %ns&er' TR( *iff' 1 Topic' 3hat is operations management4 ,"ective' .,1-1 6) %n e7ample of a 8hidden8 production function is mone transfers at "anks$ %ns&er' TR( *iff' Topic' 3hat is operations management4 ,"ective' .,1-1 9) ,ne reason to stud operations management is to learn ho& people organi!e themselves for productive enterprise$ %ns&er' TR( *iff' 1 Topic' 3h stud ,:4 ,"ective' .,1-1 1-1 Copright ; <11 =earson ducation >nc$

Upload: brainy12345

Post on 04-Jun-2018

232 views

Category:

Documents


0 download

TRANSCRIPT

Page 1: OM Exam 1 Chap 1-5

8/13/2019 OM Exam 1 Chap 1-5

http://slidepdf.com/reader/full/om-exam-1-chap-1-5 1/187

Operations Management, 10e (Heizer/Render)Chapter 1 Operations and Productivity

True/False

1) Some of the operations-related activities of Hard Rock Café include designing meals andanal !ing them for ingredient cost and la"or re#uirements$%ns&er' TR(*iff' 1Topic' +lo"al compan profile," ective' .,1-1

) 0ecause Hard Rock Cafes are themed restaurants operations managers focus their la outdesign efforts on attractiveness &hile pa ing little attention to efficienc $%ns&er' F%.S*iff' 1

Topic' +lo"al compan profile," ective' no .,

2) %ll organi!ations including service firms such as "anks and hospitals have a productionfunction$%ns&er' TR(*iff' Topic' 3hat is operations management4," ective' .,1-1

5) ,perations management is the set of activities that creates value in the form of goods and

services " transforming inputs into outputs$%ns&er' TR(*iff' 1Topic' 3hat is operations management4," ective' .,1-1

6) %n e7ample of a 8hidden8 production function is mone transfers at "anks$%ns&er' TR(*iff' Topic' 3hat is operations management4," ective' .,1-1

9) ,ne reason to stud operations management is to learn ho& people organi!e themselves for productive enterprise$%ns&er' TR(*iff' 1Topic' 3h stud ,:4," ective' .,1-1

1-1Cop right ; <11 =earson ducation >nc$

Page 2: OM Exam 1 Chap 1-5

8/13/2019 OM Exam 1 Chap 1-5

http://slidepdf.com/reader/full/om-exam-1-chap-1-5 2/187

?) The operations manager performs the management activities of planning organi!ing staffingleading and controlling of the ,: function$%ns&er' TR(*iff' 1Topic' 3hat operations managers do

," ective' .,1-1@) 8Ho& much inventor of this item should &e have48 is &ithin the critical decision area ofmanaging #ualit $%ns&er' F%.S*iff' 1Topic' 3hat operations managers do," ective' .,1-1

A) >n order to have a career in operations management one must have a degree in statistics or#uantitative methods$%ns&er' F%.S*iff' 1Topic' 3hat operations managers do," ective' .,1-1

1<) Henr Ford is kno&n as the Father of Scientific :anagement$%ns&er' F%.S*iff' 1Topic' The heritage of operations management," ective' .,1-1

11) She&hartBs contri"utions to operations management came during the Scientific :anagementra$%ns&er' F%.S*iff' 1Topic' The heritage of operations management," ective' .,1-1

1 ) Customer interaction is often high for manufacturing processes "ut lo& for services$%ns&er' F%.S*iff' Topic' ,perations in the service sector ," ective' .,1-

12) =roductivit is more difficult to improve in the service sector$%ns&er' TR(*iff' Topic' The productivit challenge," ective' .,1-9

1-Cop right ; <11 =earson ducation >nc$

Page 3: OM Exam 1 Chap 1-5

8/13/2019 OM Exam 1 Chap 1-5

http://slidepdf.com/reader/full/om-exam-1-chap-1-5 3/187

15) :anufacturing no& constitutes the largest economic sector in postindustrial societies$%ns&er' F%.S*iff' Topic' ,perations in the service sector ," ective' .,1-

16) >n the past half-centur &hile the num"er of people emplo ed in manufacturing in the(nited States has decreased slightl the output per &orker has increased significantl $

%ns&er' TR(*iff' 1Topic' ,perations in the service sector ," ective' .,1-

19) % kno&ledge societ is one that has migrated from &ork "ased on kno&ledge to one "asedon manual &ork$%ns&er' F%.S*iff' 1Topic' The productivit challenge," ective' .,1-2

1?) =roductivit is the total value of all inputs to the transformation process divided " the totalvalue of the outputs produced$%ns&er' F%.S*iff' 1Topic' The productivit challenge," ective' .,1-2

1@) :easuring the impact of a capital ac#uisition on productivit is an e7ample of multifactor productivit $%ns&er' F%.S*iff' Topic' The productivit challenge," ective' .,1-6

1A) thical and social dilemmas arise "ecause stakeholders of a "usiness have conflicting perspectives$%ns&er' TR(*iff' 1Topic' thics and social responsi"ilit%%CS0' thical Reasoning," ective' no .,

1-2Cop right ; <11 =earson ducation >nc$

Page 4: OM Exam 1 Chap 1-5

8/13/2019 OM Exam 1 Chap 1-5

http://slidepdf.com/reader/full/om-exam-1-chap-1-5 4/187

:ultiple Choice

1) 3hich of the follo&ing is not one of the Ten Critical *ecisions of ,perations :anagement4%) location strateg0) human resources and o" design

C) managing #ualit*) design of goods and services) determining the financial leverage position%ns&er' *iff' Topic' 3hat operations managers do," ective' no .,

) %n operations task performed at Hard Rock Café is%) "orro&ing funds to "uild a ne& restaurant0) advertising changes in the restaurant menu

C) calculating restaurant profit and loss*) preparing emplo ee schedules) all of the a"ove%ns&er' **iff' Topic' +lo"al compan profile," ective' .,1-1

2) ,perations management is applica"le%) mostl to the service sector 0) to services e7clusivelC) mostl to the manufacturing sector *) to all firms &hether manufacturing or service) to the manufacturing sector e7clusivel%ns&er' **iff' Topic' 3hat is operations management4," ective' .,1-1

5) 3hich of the follo&ing are the primar functions of all organi!ations4%) production/operations marketing and human resources0) marketing human resources and finance/accountingC) sales #ualit control and production/operations*) marketing production/operations and finance/accounting) research and development finance/accounting and purchasing%ns&er' **iff' Topic' ,rgani!ing to produce goods and services," ective' .,1-1

1-5Cop right ; <11 =earson ducation >nc$

Page 5: OM Exam 1 Chap 1-5

8/13/2019 OM Exam 1 Chap 1-5

http://slidepdf.com/reader/full/om-exam-1-chap-1-5 5/187

6) 3hich of the follo&ing pioneers &as not making a professional impact during the Scientific:anagement ra4%) Frank +il"reth0) 3$ d&ards *emingC) Henr .$ +antt

*) .illian +il"reth) Frederick 3$ Ta lor %ns&er' 0*iff' Topic' The heritage of operations management," ective' no .,

9) 3hich of the follo&ing &ould not "e an operations function in a commercial "ank4%) auditing0) teller schedulingC) maintenance

*) collection) check clearing%ns&er' %*iff' Topic' ,rgani!ing to produce goods and services," ective' .,1-1

?) The marketing functionBs main concern is &ith%) producing goods or providing services0) procuring materials supplies and e#uipmentC) "uilding and maintaining a positive image*) generating the demand for the organi!ationBs products or services) securing monetar resources%ns&er' **iff' Topic' ,rgani!ing to produce goods and services," ective' .,1-1

@) 3hich of the follo&ing tasks &ithin an %irline Compan are related to ,perations4%) Cre& Scheduling0) >nternational :onetar 7changeC) Sales*) %dvertising) %ccounts =a a"le%ns&er' %*iff' Topic' ,rgani!ing to produce goods and services," ective' .,1-1

1-6Cop right ; <11 =earson ducation >nc$

Page 6: OM Exam 1 Chap 1-5

8/13/2019 OM Exam 1 Chap 1-5

http://slidepdf.com/reader/full/om-exam-1-chap-1-5 6/187

A) Reasons to stud ,perations :anagement include%) stud ing ho& people organi!e themselves for productive enterprise0) kno&ing ho& goods and services are consumedC) understanding &hat human resource managers do*) learning a"out a costl part of the enterprise

) % and *%ns&er' *iff' Topic' 3h stud ,:4," ective' .,1-1

1<) Reasons to stud ,perations :anagement include learning a"out%) ho& people organi!e themselves for productive enterprise0) ho& goods and services are producedC) &hat operations managers do*) a costl part of the enterprise

) all of the a"ove%ns&er' *iff' 1Topic' 3h stud ,:4," ective' .,1-1

11) The five elements in the management process are%) plan direct update lead and supervise0) accounting finance marketing operations and managementC) organi!e plan control staff and manage*) plan organi!e staff lead and control) plan lead organi!e manage and control%ns&er' **iff' 1Topic' 3hat operations managers do," ective' .,1-1

1 ) >lliterac and poor diets have "een kno&n to cost countries up to &hat percent of their productivit 4%) 0) 6C) 1<*) <) 6<%ns&er' **iff' Topic' =roductivit varia"les%%CS0' :ulticultural and *iversit," ective' .,1-9

1-9Cop right ; <11 =earson ducation >nc$

Page 7: OM Exam 1 Chap 1-5

8/13/2019 OM Exam 1 Chap 1-5

http://slidepdf.com/reader/full/om-exam-1-chap-1-5 7/187

12) 3hich of the follo&ing is not an element of the management process4%) controlling0) leadingC) planning*) pricing

) staffing%ns&er' **iff' 1Topic' 3hat operations managers do," ective' .,1-1

15) %n operations manager is not likel to "e involved in%) the design of goods and services to satisf customersB &ants and needs0) the #ualit of goods and services to satisf customersB &ants and needsC) the identification of customersB &ants and needs*) &ork scheduling to meet the due dates promised to customers

) maintenance schedules%ns&er' C*iff' 1Topic' 3hat operations managers do," ective' .,1-1

16) %ll of the follo&ing decisions fall &ithin the scope of operations management except for %) creating the compan income statement0) design of goods and processesC) location of facilities*) managing #ualit) %ll of the a"ove fall &ithin the scope of operations management$%ns&er' %*iff' 1Topic' 3hat operations managers do," ective' .,1-1

19) The Ten Critical *ecisions of ,perations :anagement include%) la out strateg0) maintenanceC) process and capacit design*) managing #ualit) all of the a"ove%ns&er' *iff' 1Topic' 3h stud ,:4," ective' .,1-1

1-?Cop right ; <11 =earson ducation >nc$

Page 8: OM Exam 1 Chap 1-5

8/13/2019 OM Exam 1 Chap 1-5

http://slidepdf.com/reader/full/om-exam-1-chap-1-5 8/187

1?) 3hich of the follo&ing is not one of The Ten Critical *ecisions of ,perations :anagement4%) la out strateg0) maintenanceC) process and capacit design*) mass customi!ation

) suppl -chain management%ns&er' **iff' Topic' 3h stud ,:4," ective' .,1-1

1@) The Ten Critical *ecisions of ,perations :anagement include%) finance/accounting0) advertisingC) process and capacit design*) pricing

) all of the a"ove%ns&er' C*iff' Topic' 3h stud ,:4," ective' .,1-1

1A) 3hich of the follo&ing are part of the Ten Critical *ecisions of ,perations :anagement4>$ *esign of goods and services>>$ :anaging Dualit>>>$ .a out Strateg>E$ :arketingE$ =ricing of +oods and Services%) > >> E0) > >> >EC) >> >>> E*) > >> >>>) %ll of the a"ove%ns&er' **iff' Topic' 3h stud ,:4," ective' .,1-1

1-@Cop right ; <11 =earson ducation >nc$

Page 9: OM Exam 1 Chap 1-5

8/13/2019 OM Exam 1 Chap 1-5

http://slidepdf.com/reader/full/om-exam-1-chap-1-5 9/187

<) %SD >S: %=>CS and =:> are important professional organi!ations to operationsmanagement "ecause%) the provide certification for professionals0) the allo& professionals to keep up &ith industr developmentsC) the facilitate professional net&orking

*) none of the a"ove) all of the a"ove%ns&er' *iff' Topic' 3hat operations managers do," ective' .,1-1

1) 3alter She&hart is listed among the important people of operations management "ecause ofhis contri"utions to%) assem"l line production0) measuring the productivit in the service sector

C) ust-in-time inventor methods*) statistical #ualit control) all of the a"ove%ns&er' **iff' Topic' The heritage of operations management," ective' .,1-1

) 3alter She&hart in the provided the foundations for in operationsmanagement$%) 1A <sG statistical sampling0) (nited ingdomG mass productionC) ($S$ %rm G logistics*) nineteenth centur G interchangea"le parts) none of the a"ove%ns&er' %*iff' Topic' The heritage of operations management," ective' .,1-1

2) li 3hitne in the provided the foundations for in operationsmanagement$%) 1A <sG statistical sampling0) (nited ingdomG mass productionC) ($S$ %rm G logistics*) nineteenth centur G interchangea"le parts) none of the a"ove%ns&er' **iff' Topic' The heritage of operations management," ective' .,1-1

1-ACop right ; <11 =earson ducation >nc$

Page 10: OM Exam 1 Chap 1-5

8/13/2019 OM Exam 1 Chap 1-5

http://slidepdf.com/reader/full/om-exam-1-chap-1-5 10/187

Page 11: OM Exam 1 Chap 1-5

8/13/2019 OM Exam 1 Chap 1-5

http://slidepdf.com/reader/full/om-exam-1-chap-1-5 11/187

@) The field of operations management is shaped " advances in &hich of the follo&ing fields4%) chemistr and ph sics0) industrial engineering and management scienceC) "iolog and anatom*) information technolog

) all of the a"ove%ns&er' *iff' Topic' The heritage of operations management," ective' .,1-1

A) 3hich of the follo&ing is the "est e7ample of a pure service4%) counseling0) oil ChangeC) heart transplant*) electric Co-,p

) all of the a"ove%ns&er' %*iff' Topic' ,perations in the service sector ," ective' .,1-

2<) 3hich of the follo&ing statements is true 4%) The person most responsi"le for initiating the use of interchangea"le parts in manufacturing&as li 3hitne $0) The origins of management " e7ception are generall credited to Frederick 3$ Ta lor$C) The person most responsi"le for initiating the use of interchangea"le parts in manufacturing&as 3alter She&hart$*) The origins of the scientific management movement are generall credited to Henr Ford$) The person most responsi"le for initiating the use of interchangea"le parts in manufacturing&as Henr Ford$%ns&er' %*iff' Topic' The heritage of operations management," ective' .,1-1

21) The service sector makes up appro7imatel &hat percentage of all o"s in the (nited States4%) 10) 5<C) 99*) ?A) A<%ns&er' **iff' Topic' ,perations in the service sector ," ective' .,1-

1-11Cop right ; <11 =earson ducation >nc$

Page 12: OM Exam 1 Chap 1-5

8/13/2019 OM Exam 1 Chap 1-5

http://slidepdf.com/reader/full/om-exam-1-chap-1-5 12/187

2 ) 3hich is not true regarding differences "et&een goods and services4%) Tangi"le goods are generall produced and consumed simultaneousl G services are not$0) :ost goods are common to man customersG services are often uni#ue to the final customer$C) Services tend to have a more inconsistent product definition than goods$*) Services tend to have higher customer interaction than goods$

) %ll of the a"ove are true$%ns&er' %*iff' Topic' ,perations in the service sector ," ective' .,1-

22) 3hich is not true regarding differences "et&een goods and services4%) Services are generall produced and consumed simultaneousl G tangi"le goods are not$0) Services tend to "e more kno&ledge-"ased than goods$C) Services tend to have a more inconsistent product definition than goods$*) +oods tend to have higher customer interaction than services$

) Ione of the a"ove is true$%ns&er' **iff' Topic' ,perations in the service sector ," ective' .,1-

25) 3hich of the follo&ing services is least likely to be unique i$e$ customi!ed to a particularindividualBs needs4%) dental care0) hairdressingC) legal services*) elementar education) computer consulting%ns&er' **iff' Topic' ,perations in the service sector ," ective' .,1-

26) 3hich of the follo&ing is not a t pical service attri"ute4%) intangi"le product0) eas to storeC) customer interaction is high*) simultaneous production and consumption) difficult to resell%ns&er' 0*iff' Topic' ,perations in the service sector ," ective' .,1-

1-1Cop right ; <11 =earson ducation >nc$

Page 13: OM Exam 1 Chap 1-5

8/13/2019 OM Exam 1 Chap 1-5

http://slidepdf.com/reader/full/om-exam-1-chap-1-5 13/187

29) 3hich of the follo&ing statements is true 4%) :anufacturing no& constitutes the largest economic sector in postindustrial societies0) The num"er of people emplo ed in manufacturing has increased since 1A6<C) ach manufacturing emplo ee no& produces a"out < times more than in 1A6<*) %ll of the a"ove are true$

) Ione of the a"ove is true$%ns&er' C*iff' Topic' ,perations in the service sector ," ective' .,1-

2?) 3hich of the follo&ing attri"utes is most t pical of a service4%) production and consumption occur simultaneousl0) tangi"leC) mass production*) consistenc

) eas to automate%ns&er' %*iff' Topic' ,perations in the service sector ," ective' .,1-

2@) 3hich of the follo&ing is a similarit "et&een goods and services4%) mass production0) consistencC) automation*) application of operations management) all of the a"ove%ns&er' **iff' Topic' ,perations in the service sector ," ective' .,1-

2A) Current trends in operations management include all of the follo&ing except%) ust-in-time performance0) rapid product developmentC) mass customi!ation*) empo&ered emplo ees) %ll of the a"ove are current trends$%ns&er' *iff' Topic' 7citing ne& trends in operations management," ective' .,1-1

1-12Cop right ; <11 =earson ducation >nc$

Page 14: OM Exam 1 Chap 1-5

8/13/2019 OM Exam 1 Chap 1-5

http://slidepdf.com/reader/full/om-exam-1-chap-1-5 14/187

5<) 3hich of the follo&ing is not a current trend in operations management4%) ust-in-time performance0) glo"al focusC) suppl -chain partnering*) mass customi!ation

) %ll of the a"ove are current trends$%ns&er' *iff' Topic' 7citing ne& trends in operations management," ective' .,1-1

51) Ie& trends in operations management include%) glo"al focus0) mass customi!ationC) empo&ered emplo ees*) rapid product development

) %ll of the a"ove are ne& trends in operations management$%ns&er' *iff' Topic' 7citing ne& trends in operations management," ective' .,1-1

5 ) 3hich of the follo&ing statements a"out trends in operations management is false4%) Jo" speciali!ation is giving &a to empo&ered emplo ees$0) .ocal or national focus is giving &a to glo"al focus$C) nvironmentall -sensitive production is giving &a to lo&-cost focus$*) Rapid product development is partl the result of shorter product c cles$) %ll of the a"ove statements are false$%ns&er' C*iff' Topic' 7citing ne& trends in operations management," ective' .,1-1

52) % foundr produces circular utilit access hatches Kmanhole covers)$ >f 1 < covers are produced in a 1<-hour shift the productivit of the line is%) 1$ covers/hr 0) covers/hr C) 1 covers/hr *) 1 << covers/hr ) none of the a"ove%ns&er' C*iff' 1Topic' The productivit challenge%%CS0' %nal tic Skills," ective' .,1-5

1-15Cop right ; <11 =earson ducation >nc$

Page 15: OM Exam 1 Chap 1-5

8/13/2019 OM Exam 1 Chap 1-5

http://slidepdf.com/reader/full/om-exam-1-chap-1-5 15/187

55) % foundr produces circular utilit access hatches Kmanhole covers)$ Currentl 1 < coversare produced in a 1<-hour shift$ >f la"or productivit can "e increased " < it &ould then "e%) 15$5 covers/hr 0) 5 covers/hr C) 5< covers/hr

*) 1 << covers/hr ) none of the a"ove%ns&er' %*iff' Topic' The productivit challenge%%CS0' %nal tic Skills," ective' .,1-5

56) +i"son Ealves produces cast "ron!e valves on an assem"l line$ >f 19<< valves are producedin an @-hour shift the productivit of the line is%) valves/hr

0) 5< valves/hr C) @< valves/hr *) << valves/hr ) 19<< valves/hr %ns&er' **iff' 1Topic' The productivit challenge%%CS0' %nal tic Skills," ective' .,1-5

59) +i"son Ealves produces cast "ron!e valves on an assem"l line currentl producing 19<<valves each @-hour shift$ >f the productivit is increased " 1< it &ould then "e%) 1@< valves/hr 0) << valves/hr C) < valves/hr *) @@< valves/hr ) 1?9< valves/hr %ns&er' C*iff' Topic' The productivit challenge%%CS0' %nal tic Skills," ective' .,1-5

1-16Cop right ; <11 =earson ducation >nc$

Page 16: OM Exam 1 Chap 1-5

8/13/2019 OM Exam 1 Chap 1-5

http://slidepdf.com/reader/full/om-exam-1-chap-1-5 16/187

5?) +i"son Ealves produces cast "ron!e valves on an assem"l line currentl producing 19<<valves per shift$ >f the production is increased to <<< valves per shift la"or productivit &illincrease "%) 1<0) <

C) 6*) 5<) 6<%ns&er' C*iff' Topic' The productivit challenge%%CS0' %nal tic Skills," ective' .,1-5

5@) The *ulac 0o7 plant produces 6<< c press packing "o7es in t&o 1<-hour shifts$ 3hat is the productivit of the plant4

%) 6 "o7es/hr 0) 6< "o7es/hr C) 6<<< "o7es/hr *) none of the a"ove) not enough data to determine productivit%ns&er' %*iff' Topic' The productivit challenge%%CS0' %nal tic Skills," ective' .,1-5

5A) The *ulac 0o7 plant &orks t&o @-hour shifts each da $ >n the past 6<< c press packing "o7es &ere produced " the end of each da $ The use of ne& technolog has ena"led them toincrease productivit " 2< $ =roductivit is no& appro7imatel%) 2 $6 "o7es/hr 0) 5<$9 "o7es/hr C) 9 $6 "o7es/hr *) @1$ 6 "o7es/hr ) 2<< "o7es/hr %ns&er' 0*iff' Topic' The productivit challenge%%CS0' %nal tic Skills," ective' .,1-5

1-19Cop right ; <11 =earson ducation >nc$

Page 17: OM Exam 1 Chap 1-5

8/13/2019 OM Exam 1 Chap 1-5

http://slidepdf.com/reader/full/om-exam-1-chap-1-5 17/187

6<) The *ulac 0o7 plant produces 6<< c press packing "o7es in t&o 1<-hour shifts$ *ue tohigher demand the have decided to operate three @-hour shifts instead$ The are no& a"le to

produce 9<< "o7es per da $ 3hat has happened to productivit 4%) >t has not changed$0) >t has increased " 2?$6 "o7es/hr$

C) >t has increased " < $*) >t has decreased " @$2 $) >t has decreased " A$1 $%ns&er' %*iff' Topic' The productivit challenge%%CS0' %nal tic Skills," ective' .,1-5

61) =roductivit measurement is complicated "%) the competitionBs output

0) the fact that precise units of measure are often unavaila"leC) sta"le #ualit*) the &orkforce si!e) the t pe of e#uipment used%ns&er' 0*iff' Topic' The productivit challenge," ective' .,1-2

6 ) The total of all outputs produced " the transformation process divided " the total of theinputs is%) utili!ation0) greater in manufacturing than in servicesC) defined onl for manufacturing firms*) multifactor productivit) none of the a"ove%ns&er' **iff' Topic' The productivit challenge," ective' .,1-6

62) 3hich productivit varia"le has the greatest potential to increase productivit 4%) la"or 0) glo"ali!ationC) management*) capital) none of the a"ove%ns&er' C*iff' Topic' The productivit challenge," ective' .,1-9

1-1?Cop right ; <11 =earson ducation >nc$

Page 18: OM Exam 1 Chap 1-5

8/13/2019 OM Exam 1 Chap 1-5

http://slidepdf.com/reader/full/om-exam-1-chap-1-5 18/187

65) 3hich of the follo&ing nets the largest productivit improvement4%) increase output 160) decrease input 16C) increase "oth output and input " 6*) increase output 1< decrease input 2

) decrease input 1< increase output 2%ns&er' 0*iff' Topic' The productivit challenge%%CS0' %nal tic Skills," ective' .,1-6

66) =roductivit can "e improved "%) increasing inputs &hile holding outputs stead0) decreasing outputs &hile holding inputs steadC) increasing inputs and outputs in the same proportion

*) decreasing inputs &hile holding outputs stead) none of the a"ove%ns&er' **iff' Topic' The productivit challenge," ective' .,1-9

69) The largest contri"utor to productivit increases is estimated to "e responsi"lefor of the annual increase$%) managementG over one-half 0) :r$ *emingG one-half C) la"orG t&o-thirds*) capitalG A<) technolog G over one-half %ns&er' %*iff' Topic' The productivit challenge," ective' .,1-9

6?) The factor responsi"le for the largest portion of productivit increase in the ($S$ is%) la"or 0) managementC) capital*) %ll three com"inedG it is impossi"le to determine the contri"ution of individual factors$) Ione of these%ns&er' 0*iff' Topic' The productivit challenge," ective' .,1-9

1-1@Cop right ; <11 =earson ducation >nc$

Page 19: OM Exam 1 Chap 1-5

8/13/2019 OM Exam 1 Chap 1-5

http://slidepdf.com/reader/full/om-exam-1-chap-1-5 19/187

6@) 3hich of the follo&ing is not true &hen e7plaining &h productivit tends to "e lo&er in theservice sector than in the manufacturing sector4%) Services are t picall la"or-intensive$0) Services are often difficult to evaluate for #ualit $C) Services are often an intellectual task performed " professionals$

*) Services are difficult to automate$) Service operations are t picall capital intensive$%ns&er' *iff' Topic' The productivit challenge," ective' .,1-9

6A) Three commonl used productivit varia"les are%) #ualit e7ternal elements and precise units of measure0) la"or capital and managementC) technolog ra& materials and la"or

*) education diet and social overhead) #ualit efficienc and lo& cost%ns&er' 0*iff' Topic' The productivit challenge," ective' .,1-2

9<) The service sector has lo&er productivit improvements than the manufacturing sector "ecause%) the service sector uses less skilled la"or than manufacturing0) the #ualit of output is lo&er in services than manufacturingC) services usuall are la"or-intensive*) service sector productivit is hard to measure) none of the a"ove%ns&er' C*iff' Topic' The productivit challenge," ective' .,1-9

91) =roductivit tends to "e more difficult to improve in the service sector "ecause the &ork is%) often difficult to automate0) t picall la"or-intensiveC) fre#uentl processed individuall*) often an intellectual task performed " professionals) %ll of the a"ove make service productivit more difficult$%ns&er' *iff' Topic' The productivit challenge," ective' .,1-9

1-1ACop right ; <11 =earson ducation >nc$

Page 20: OM Exam 1 Chap 1-5

8/13/2019 OM Exam 1 Chap 1-5

http://slidepdf.com/reader/full/om-exam-1-chap-1-5 20/187

9 ) Firm % operates 1< hours each da producing 1<< parts/hour$ >f productivit &ere increased< ho& man hours &ould the plant have to &ork to produce 1<<< parts4%) less than hours0) "et&een A and 1< hoursC) "et&een and 9 hours

*) "et&een 9 and @ hours) "et&een @ and A hours%ns&er' *iff' Topic' The productivit challenge%%CS0' %nal tic Skills," ective' .,1-5

92) % cleaning compan uses 1< l"s each of chemicals % 0 and C for each house it cleans$%fter some #ualit complaints the compan has decided to increase its use of chemical % " anadditional 1< l"s for each house$ 0 &hat has productivit Khouses per pound of chemical)

fallen4%) <0) 1<C) 16*) 6) 22%ns&er' **iff' Topic' The productivit challenge%%CS0' %nal tic Skills," ective' .,1-5

95) % cleaning compan uses L1< of chemicals L5< of la"or and L6 of misc$ e7penses for eachhouse it cleans$ %fter some #ualit complaints the compan has decided to increase its use ofchemicals " 6< $ 0 &hat has multifactor productivit fallen4%) <0) @$2C) 6*) 6<) none of the a"ove or una"le to determine%ns&er' 0*iff' Topic' The productivit challenge%%CS0' %nal tic Skills," ective' .,1-6

1- <Cop right ; <11 =earson ducation >nc$

Page 21: OM Exam 1 Chap 1-5

8/13/2019 OM Exam 1 Chap 1-5

http://slidepdf.com/reader/full/om-exam-1-chap-1-5 21/187

96) %mong the ethical and social challenges facing operations managers are%) honoring stakeholder commitments0) maintaining a sustaina"le environmentC) efficientl developing and producing safe #ualit products*) providing a safe &orkplace

) all of the a"ove%ns&er' *iff' 1Topic' thics and social responsi"ilit%%CS0' thical Reasoning," ective' no .,

99) %mong the ethical and social challenges facing operations managers are%) honoring financial commitments0) maintaining a sustaina"le environmentC) developing lo&-cost products*) providing an efficient &orkplace) all of the a"ove%ns&er' 0*iff' Topic' thics and social responsi"ilit%%CS0' thical Reasoning," ective' no .,

9?) 3hich of the follo&ing is not among the ethical and social challenges facing operationsmanagers4%) honoring stakeholder commitments0) maintaining a sustaina"le environmentC) efficientl developing and producing safe #ualit products*) increasing e7ecutive pa) providing a safe &orkplace%ns&er' **iff' 1Topic' thics and social responsi"ilit%%CS0' thical Reasoning," ective' no .,

1- 1Cop right ; <11 =earson ducation >nc$

Page 22: OM Exam 1 Chap 1-5

8/13/2019 OM Exam 1 Chap 1-5

http://slidepdf.com/reader/full/om-exam-1-chap-1-5 22/187

9@) % "usinessBs stakeholders &hose conflicting perspectives cause ethical and social dilemmasinclude%) lenders0) suppliersC) o&ners

*) emplo ees) all of the a"ove%ns&er' *iff' 1Topic' thics and social responsi"ilit%%CS0' thical Reasoning," ective' no .,

Short %ns&er

1) Star"ucks stopped re#uiring signatures on credit-card purchases under L 6 in an attempt to

reduce $%ns&er' transaction time Kor service time)*iff' 1Topic' The productivit challenge," ective' .,1-9

) is the set of activities that transforms inputs into goods and services$%ns&er' ,perations management*iff' 1Topic' 3hat is operations management4," ective' .,1-1

2) :arketing =roduction and are the three functions that all organi!ations must perform to create goods and services$%ns&er' finance/accounting*iff' 1Topic' ,rgani!ing to produce goods and services," ective' .,1-1

5) 8Should &e make or "u this component48 is an issue in the critical decisionarea$%ns&er' suppl chain management*iff' 1Topic' 3hat operations managers do," ective' .,1-1

6) Henr Ford and are credited &ith the development of the moving assem"l line$%ns&er' Charles Sorensen*iff' 1Topic' The heritage of operations management," ective' .,1-1

1-Cop right ; <11 =earson ducation >nc$

Page 23: OM Exam 1 Chap 1-5

8/13/2019 OM Exam 1 Chap 1-5

http://slidepdf.com/reader/full/om-exam-1-chap-1-5 23/187

9) 3hen a tangi"le product is not included in a service such as &ith counseling it is called a $%ns&er' pure service*iff' 1Topic' ,perations in the service sector

," ective' .,1-

?) is the a"ilit of the organi!ation to "e fle7i"le enough to cater to the individual&hims of consumers$%ns&er' :ass customi!ation*iff' Topic' 7citing ne& trends in operations management," ective' .,1-1

@) is the operations management trend that moves more decision making to theindividual &orker$

%ns&er' mpo&ered emplo ees*iff' Topic' 7citing ne& trends in operations management," ective' .,1-1

A) is the total of all outputs produced " the transformation process divided " thetotal of the inputs$%ns&er' :ultifactor productivit*iff' 1Topic' The productivit challenge," ective' .,1-6

1<) =roductivit is the ratio of to $ (sing this relationship productivitcan "e improved " or $%ns&er' outputs intputsG reducing inputs &hile holding outputs constantG increasing outputs&hile holding inputs constant$*iff' Topic' The productivit challenge," ective' .,1-9

11) >dentif three or more operations-related tasks carried out " Hard Rock Café$%ns&er' =roviding custom mealsG designing testing and costing mealsG ac#uiring receiving and storing suppliesG recruiting and training emplo eesG preparing emplo ee schedulesGdesigning efficient restaurant la outs$*iff' 1Topic' +lo"al compan profile," ective' no .,

1- 2Cop right ; <11 =earson ducation >nc$

Page 24: OM Exam 1 Chap 1-5

8/13/2019 OM Exam 1 Chap 1-5

http://slidepdf.com/reader/full/om-exam-1-chap-1-5 24/187

1 ) >dentif t&o operations-related tasks carried out " Hard Rock Café$ :atch each to its areaof the Ten Critical *ecisions$%ns&er' =roviding custom mealsMdesign of goods and servicesG designing testing and costingmealsMdesign of goods and servicesG ac#uiring receiving and storing suppliesMsuppl - chainmanagementG recruiting and training emplo eesMhuman resources o" design and &ork

measurementG preparing emplo ee schedulesMintermediate and short-term schedulingG designingefficient restaurant la outsMla out strateg $*iff' Topic' +lo"al compan profile," ective' no .,

12) *efine operations management$ 3ill our definition accommodate "oth manufacturing andservice operations4%ns&er' ,perations management can "e defined as the management of all activities directlrelated to the creation of goods and/or services through the transformation of inputs into outputs$Nes$

*iff' 1Topic' 3hat is operations management4," ective' .,1-1

15) >dentif the items that Fredrick 3$ Ta lor "elieved management should "e more responsi"lefor$%ns&er' He "elieved that management should "e more responsi"le for matching emplo ees tothe right o" providing the proper training providing proper &ork methods and tools andesta"lishing legitimate incentives for &ork to "e accomplished$*iff' Topic' The heritage of operations management," ective' .,1-1

16) ,perations managers should "e &ell versed in &hat disciplines in order to make gooddecisions4%ns&er' :anagement science information technolog and often one of the "iological or

ph sical sciences$*iff' Topic' The heritage of operations management," ective' .,1-1

19) 3h are services t picall more difficult to standardi!e automate and make efficient4%ns&er' Services t picall re#uire customer interaction &hich makes it difficult to standardi!eautomate and make efficient$*iff' Topic' ,perations in the service sector ," ective' .,1-

1- 5Cop right ; <11 =earson ducation >nc$

Page 25: OM Exam 1 Chap 1-5

8/13/2019 OM Exam 1 Chap 1-5

http://slidepdf.com/reader/full/om-exam-1-chap-1-5 25/187

1?) Ho& do services differ from goods4 >dentif five &a s$%ns&er' =ick from the follo&ing' a service is usuall intangi"leG it is often produced andconsumed simultaneousl G often uni#ueG it involves high customer interactionG product definitionis inconsistentG often kno&ledge-"asedG and fre#uentl dispersed$*iff'

Topic' ,perations in the service sector ," ective' .,1-

1@) Services are often kno&ledge-"ased$ =rovide t&o e7amples and e7plain &h the arekno&ledge-"ased$%ns&er' %ns&er &ill var "ut the te7t"ook used educational medical and legal services$*iff' Topic' ,perations in the service sector ," ective' .,1-

1A) 3h are organi!ations changing from "atch Klarge) shipments to ust-in-time KJ>T)

shipments4%ns&er' ,rgani!ations are s&itching to J>T shipments "ecause inventor re#uires a largefinancial investment and impedes the responsiveness to changes in the marketplace$*iff' Topic' 7citing ne& trends in operations management," ective' .,1-1

<) 3h are organi!ations "ecoming more glo"al4%ns&er' ,rgani!ations are "ecoming more glo"al &ith the decline in the cost of communicationand transportation$ %dditionall resourcesMcapital material talent and la"orMare also

"ecoming more glo"al$*iff' Topic' 7citing ne& trends in operations management," ective' .,1-1

1) >dentif the three productivit varia"les used in the te7t$%ns&er' The three common varia"les are la"or capital and management$*iff' Topic' =roductivit varia"les," ective' .,1-9

) 3hat is a kno&ledge societ 4%ns&er' % kno&ledge societ is one in &hich much of the la"or force has migrated frommanual &ork to &ork "ased on kno&ledge$*iff' Topic' The productivit challenge," ective' .,1-9

1- 6Cop right ; <11 =earson ducation >nc$

Page 26: OM Exam 1 Chap 1-5

8/13/2019 OM Exam 1 Chap 1-5

http://slidepdf.com/reader/full/om-exam-1-chap-1-5 26/187

2) 3h are operations managers faced &ith ethical and social challenges4%ns&er' 0usinesses have diverse stakeholders &hich include o&ners emplo ees lenders anddistri"utors$ These stakeholders hold conflicting perspectives$*iff' 1Topic' thics and social responsi"ilit

%%CS0' thical Reasoning," ective' .,1-1

5) 3hat are some of the ethical and social challenges faced " operations managers4%ns&er' fficientl developing and producing safe #ualit productsG maintaining a cleanenvironmentG providing a safe &orkplaceG honoring communit commitments$*iff' 1Topic' thics and social responsi"ilit%%CS0' thical Reasoning," ective' .,1-1

6) %s the administrative manager in a la& office ou have "een asked to develop a s stem forevaluating the productivit of the 16 la& ers in the office$ 3hat difficulties are ou going tohave in doing this and ho& are ou going to overcome them4%ns&er' =roductivit measures for a la& office are difficult$ Simple criteria like num"er ofcases processed fail to consider comple7it of the case$ ven counting &ins is difficult as mancases are settled &ith some sort of compromise$ 7ternal elements such as the #ualit of theopposing counsel and the tenacit of the opposition also make counting look rather sill $

Categories of cases can helpMKi$e$ uncontested divorce no personal in ur auto case etc$)Ho&ever man firms end up counting hours "illed$ This in turns leads to other pro"lems asnoted " the num"er of false "illing cases$*iff' Topic' The productivit challenge%%CS0' Reflective Thinking," ective' .,1-9

1- 9Cop right ; <11 =earson ducation >nc$

Page 27: OM Exam 1 Chap 1-5

8/13/2019 OM Exam 1 Chap 1-5

http://slidepdf.com/reader/full/om-exam-1-chap-1-5 27/187

Page 28: OM Exam 1 Chap 1-5

8/13/2019 OM Exam 1 Chap 1-5

http://slidepdf.com/reader/full/om-exam-1-chap-1-5 28/187

2) The *ulac 0o7 plant produces &ooden packing "o7es to "e used in the local seafood industr $Current operations allo& the compan to make 6<< "o7es per da in t&o @-hour shifts K 6<

"o7es per shift)$ The compan has introduced some small changes in e#uipment and conductedappropriate o" training so that production levels have risen to 2<< "o7es per shift$ Thesechanges did not re#uire an change in the amount of capital spending or energ use$ 3hat is the

firmBs ne& la"or productivit 4%ns&er' 9<< "o7es per da / 19 hours O 2?$6 "o7es per hour *iff' Topic' The productivit challenge%%CS0' %nal tic Skills," ective' .,1-5

5) :arkBs Ceramics spent L5<<< on a ne& kiln last ear in the "elief that it &ould cut energusage 6 over the old kiln$ This kiln is an oven that turns 8green&are8 into finished potter $:ark is concerned that the ne& kiln re#uires e7tra la"or hours for its operation$ :ark &ants tocheck the energ savings of the ne& oven and also to look over other measures of their

productivit to see if the change reall &as "eneficial$ :ark has the follo&ing data to &ork &ith'

.ast Near This Near =roduction Kfinished units) 5<<< 5<<<+reen&are Kpounds) 6<<< 6<<<.a"or Khrs) 26< 2?6Capital KL) 16<<< 1A<<<nerg Kk3h) 2<<< 9<<

3ere the modifications "eneficial4%ns&er' The energ modifications did not generate the e7pected energ savingsG also la"or and

capital productivit decreased$

Resource .ast Near This Near Change=ct$Change

.a"or 5<<< / 26< O11$52

5<<< / 2?6 O1<$9? -<$?9 -9$?

Capital5<<< / 16<<< O<$ ?

5<<< / 1A<<< O $1 -<$<9< - $

nerg5<<< / 2<<< O1$22

5<<< / 9<< O1$65 <$ 1 16$5

*iff' Topic' The productivit challenge%%CS0' %nal tic Skills," ective' .,1-6

1- @Cop right ; <11 =earson ducation >nc$

Page 29: OM Exam 1 Chap 1-5

8/13/2019 OM Exam 1 Chap 1-5

http://slidepdf.com/reader/full/om-exam-1-chap-1-5 29/187

6) :artin :anufacturing has implemented several programs to improve its productivit $ Thehave asked ou to evaluate the firmBs productivit " comparing this earBs performance &ith lastearBs$ The follo&ing data are availa"le'

.ast Near This Near

,utput 1< 6<< units 1 1<< units.a"or Hours 1 <<< 12 <<(tilities L? 9<< L@ 6<Capital L@2 <<< L@@ <<<

Has :artin :anufacturing improved its productivit during the past ear4%ns&er' =roductivit improved in all three categories this earG utilities sho&ed the greatestincrease and la"or the least$

Resource .ast Near This Near Change=ct$Change

.a"or 1<6<< / 1 <<< O<$@@ 1 1<< / 12 << O <$A <$<5 5$@

Capital1<6<< / ?9<< O1$2@ 1 1<< / @ 6< O 1$5? <$<A 9$

nerg1<6<< / @2<<< O<$12 1 1<< / @@<<< O <$15 <$<1 ?$9A

*iff' Topic' The productivit challenge%%CS0' %nal tic Skills," ective' .,1-6

9) Felicien gro&s mirlitons KthatBs Ca un for Cha ote s#uash) in his 1<< " 1<< foot garden$ Hethen sells the crop at the local farmersB market$ T&o summers ago he &as a"le to produce andsell 1 << pounds of mirlitons$ .ast summer he tried a ne& fertili!er that promised a 6<increase in ield$ He harvested 1A<< pounds$ *id the fertili!er live up to its promise4%ns&er' Since the productivit gain &as 6@$2 not 6< the fertili!er &as at least as good asadvertised$

T&o Summersago .ast Summer Change1 << P 1< <<< O $

1 1"s/s#$ ft

1A<< P 1< <<< O $

1A l"s/s#$ ft

K$1A - $1 ) P $1 O

6@$2

*iff' Topic' The productivit challenge%%CS0' %nal tic Skills," ective' .,1-5

1- ACop right ; <11 =earson ducation >nc$

Page 30: OM Exam 1 Chap 1-5

8/13/2019 OM Exam 1 Chap 1-5

http://slidepdf.com/reader/full/om-exam-1-chap-1-5 30/187

?) The *ulac 0o7 plant produces &ooden packing "o7es to "e used in the local seafood industr $Current operations allo& the compan to make 6<< "o7es per da in t&o @-hour shifts K 6<

"o7es per shift)$ The compan has introduced some moderate changes in e#uipment andconducted appropriate o" training so that production levels have risen to 2<< "o7es per shift$.a"or costs average L1< per hour for each of the 6 full-time &orkers on each shift$ Capital costs

&ere previousl L2 <<< per da and rose to L2 << per da &ith the e#uipment modifications$nerg costs &ere unchanged " the modifications at L5<< per da $ 3hat is the firmBsmultifactor productivit "efore and after the changes4%ns&er' :F= "efore' 6<< "o7es / KL1<76719 Q L2<<< Q L5<<) O 6<< / 5 << O <$11A

"o7es/dollar :F= after' 9<< "o7es / KL1<76719 Q L2 << Q L5<<) O 9<< / 55<< O <$129 "o7es/dollar *iff' Topic' The productivit challenge%%CS0' %nal tic Skills," ective' .,1-6

@) +i"son =roducts produces cast "ron!e valves for use in offshore oil platforms$ Currentl+i"son produces 19<< valves per da $ The < &orkers at +i"son &ork from ? a$m$ until 5 p$m$&ith 2< minutes off for lunch and a 16 minute "reak during the morning &ork session andanother at the afternoon &ork session$ +i"son is in a competitive industr and needs to increase

productivit to sta competitive$ The feel that a < percent increase is needed$

+i"sonBs management "elieves that the < percent increase &ill not "e possi"le &ithout a changein &orking conditions so the change &ork hours$ The ne& schedule calls on &orkers to &orkfrom ?'2< a$m$ until 5'2< p$m$ during &hich &orkers can take one hour off at an time of theirchoosing$ ,"viousl the num"er of paid hours is the same as "efore "ut production increases

perhaps "ecause &orkers are given a "it more control over their &orkda $ %fter this changevalve production increased to 1@<< units per da $

a$ Calculate la"or productivit for the initial situation "$ Calculate la"or productivit for the h pothetical < percent increasec$ 3hat is the productivit after the change in &ork rules4d$ 3rite a short paragraph anal !ing these results$%ns&er'Ka) 3orkers are active for eight hours per da G la"or productivit is 1< valves/hour K") >f =roductivit rises " < percent to 1 valves/hourG output &ould "e 1 7@7 < O 1A <Kc) Ie& productivit is 1@<< / K < 7 @) O 11$ 6 valves/hour Kd) +i"son did not gain the desired < percent increase in productivit "ut the did gain over 11

percent &ithout e7tra e#uipment or energ and &ithout increasing the la"or cost$*iff' Topic' The productivit challenge%%CS0' %nal tic Skills," ective' .,1-5

1-2<Cop right ; <11 =earson ducation >nc$

Page 31: OM Exam 1 Chap 1-5

8/13/2019 OM Exam 1 Chap 1-5

http://slidepdf.com/reader/full/om-exam-1-chap-1-5 31/187

A) % local universit is considering changes to its class structure in an effort to increase professor productivit $ The old schedule had each professor teaching 6 classes per &eek &ith each classmeeting an hour per da on :onda 3ednesda and Frida $ ach class contained < students$The ne& schedule has each professor teaching onl 2 classes "ut each class meets dail for anhour$ Ie& classes contain 6< students$

a$ Calculate the la"or productivit for the initial situation Kstudents/hour)$ "$ Calculate the la"or productivit for the schedule change Kstudents/hour)$c$ %re there an ethical considerations that should "e accounted for4d$ Suppose that each teacher also is re#uired to have hours of ,ffice Hours each dahe/she taught class$ >s the schedule change a productivit increase4%ns&er'Ka) =rofessors teach 1<< students in 16 hours or 9$9? students/hour$K") =rofessors teach 16< students in 16 hours or 1< students/hour$Kc) Responses should focus on honoring stakeholder commitment and can include students

per professor ratio class si!es #ualit of education etc$Kd) >nitial productivit is 1<< students in 1 hours or 5$?9 students/hour$ Ie& productivit is

16< students in 6 hours or 9 students/hour an increase or 1$ 5 students/hour$*iff' Topic' The productivit challenge%%CS0' %nal tic Skills," ective' .,1-5

1<) % grocer chain is considering the installation of a set of 5 self-checkout lanes$ The ne&self-checkout lane setup &ill replace old cashier lanes that &ere staffed " a cashier and "aggeron each lane$ ,ne cashier mans all 5 self-checkouts Kans&ering #uestions checking for un-scanned items taking coupons etc)$ Checkout on the ne& lanes takes minutes Kcustomers "agtheir o&n orders) &hile checkout &ith the old lanes took onl 56 seconds$ >n addition theelectricit costs for "oth setups are L$<6 per checkout &hile "agging Kmaterial) costs are L$1 percheckout &ith the old s stem and L$16 for the ne& s stem$ The ne& lanes also re#uire L1<< incapital costs$ %ssume that the lanes are al&a s in use for @ hours per da K1 shift) and that a&orker makes L1</hour$a$ Ho& man checkouts did the old s stem provide in a shift4

"$ Ho& man checkouts does the ne& s stem provide4c$ 3hat is the multifactor productivit for each s stem4%ns&er'Ka) lanes @ hours 29<< seconds/hour 1checkout/56 seconds O 1 @< checkoutsK") 5 lanes @ hours 9< minutes/hour 1checkout/ min O A9< checkoutsKc) Cost for the old s stem 5 &orkers @ hours L1</hour Q L$1 1 @< Q L$<6 1 @<OL61 $ Cost for the ne& s stem 1 &orker @ hours L1</hour Q L$16 A9< Q L$<6 A9< Q L1<<OL2? $ :ultifactor productivit for old s stem O 1 @< checkouts / L61 O $6 checkouts/L$:ultifactor productivit for ne& s stem O A9< checkouts / L2? O $9 checkouts/L$*iff' 2Topic' The productivit challenge%%CS0' %nal tic Skills," ective' .,1-6

1-21Cop right ; <11 =earson ducation >nc$

Page 32: OM Exam 1 Chap 1-5

8/13/2019 OM Exam 1 Chap 1-5

http://slidepdf.com/reader/full/om-exam-1-chap-1-5 32/187

11) % s&imming pool compan has 1<< <<< la"or hours availa"le per summer and &ith a la"or productivit of 6 pools per 9 <<< hours$a$ Ho& man pools can the compan install this summer4

"$ Suppose the multifactor productivit &as one pool per L 6 <<<$ Ho& much should thecompan e7pect to spend this summer constructing the pools4

%ns&er'Ka) 1<< <<< hours 6 pools/9<<< hours O @2$22 or @2 poolsK") @2 pools L 6 <<</pool O L <?6 <<<*iff' Topic' The productivit challenge%%CS0' %nal tic Skills," ective' .,1-5

1 ) %n industrial plant needs to make 1<< <<< parts per month to meet demand$ ach monthcontains < &orking da s each of &hich allo&s for 2 separate @ hour shifts$a$ >f a &orker can produce 1< parts/hour ho& man &orkers are needed on each shift4

"$ >f each shift has 1<< &orkers &hat is the productivit of an individual &orker4c$ >f material costs are L1</part capital costs are L1<< <<< and la"or costs are L1</hour&hat is the multifactor productivit of the plant from part %4%ns&er'a$ 1<< <<< parts 1 hour / 1< parts 1 shift / @ hours 1 &orker/9< shifts O <$@2 O 1&orkers

"$ 1<< <<< parts / K9< shifts/&orker 1<< &orkers @ hours/shift) O $<@ parts/hour c$ 1<< <<< parts / KL1</part 1<< <<< parts Q L1<< <<< Q 1 &orkers 9< shifts/&orker @hours/shift L1</hour) O $<@2 parts/L1*iff' Topic' The productivit challenge%%CS0' %nal tic Skills," ective' .,1-5

12) The local fast food store e7perienced the follo&ing num"er of customers on the night shiftHour----------------Customers1 %: 21 %: <

%: 162 %: 65 %:6 %: 1>f the store &as staffed " t&o &orkers &hat &as the average productivit per &orker incustomers/hour4%ns&er' K 2Q <Q16Q6Q Q1) customers/ K &orkers 9 hours/&orker) O 6$6 customers/hour *iff' Topic' The productivit challenge%%CS0' %nal tic Skills," ective' .,1-5

1-2Cop right ; <11 =earson ducation >nc$

Page 33: OM Exam 1 Chap 1-5

8/13/2019 OM Exam 1 Chap 1-5

http://slidepdf.com/reader/full/om-exam-1-chap-1-5 33/187

Page 34: OM Exam 1 Chap 1-5

8/13/2019 OM Exam 1 Chap 1-5

http://slidepdf.com/reader/full/om-exam-1-chap-1-5 34/187

Operations Management, 10e (Heizer/Render)Chapter & Operations trate$y in a 'lobal !nviron"ent

True/False

1) I%FT% seeks to phase out all trade and tariff "arriers among Canada :e7ico and the (nitedStates$%ns&er' TR(*iff' Topic' % glo"al vie& of operations%%CS0' :ulticultural and *iversit," ective' no .,

) The 3orld Trade ,rgani!ation has helped to significantl reduce tariffs around the &orld$%ns&er' TR(*iff'

Topic' % glo"al vie& of operations%%CS0' :ulticultural and *iversit," ective' no .,

2) =roduction processes are "eing dispersed to take advantage of national differences in la"orcosts$%ns&er' TR(*iff' Topic' % glo"al vie& of operations%%CS0' :ulticultural and *iversit," ective' no .,

5) I%FT% seeks to phase out all trade and tariff "arriers "et&een the (nited States and %sia$%ns&er' F%.S*iff' Topic' % glo"al vie& of operations%%CS0' :ulticultural and *iversit," ective' no .,

6) ,ne reason for glo"al operations is to gain improvements in the suppl chain$%ns&er' TR(*iff' Topic' % glo"al vie& of operations%%CS0' :ulticultural and *iversit," ective' no .,

1-25Cop right ; <11 =earson ducation >nc$

Page 35: OM Exam 1 Chap 1-5

8/13/2019 OM Exam 1 Chap 1-5

http://slidepdf.com/reader/full/om-exam-1-chap-1-5 35/187

9) ,ne reason to glo"ali!e is to learn to improve operations$%ns&er' TR(*iff' 1Topic' % glo"al vie& of operations%%CS0' :ulticultural and *iversit

," ective' no .,?) To attract and retain glo"al talent and to e7pand a productBs life c cle are "oth reasons toglo"ali!e$%ns&er' TR(*iff' Topic' % glo"al vie& of operations%%CS0' :ulticultural and *iversit," ective' no .,

@) % product &ill al&a s "e in the same stage of its product life c cle regardless of the countr $%ns&er' F%.S*iff' Topic' % glo"al vie& of operations%%CS0' :ulticultural and *iversit," ective' no .,

A) The 3orld Trade ,rgani!ation helps provide governments and industries around the &orld&ith protection from firms that engage in unethical conduct$%ns&er' TR(*iff' Topic' % glo"al vie& of operations%%CS0' thical Reasoning," ective' no .,

1<) 0oeingBs development of the ?@? *reamliner is an e7ample of a compan o"taining acompetitive advantage through product differentiation/innovation$%ns&er' TR(*iff' 1Topic' +lo"al compan profile," ective' no .,

11) %n organi!ationBs strateg is its purpose or rationale for an organi!ationBs e7istence$%ns&er' F%.S*iff' 1Topic' *eveloping missions and strategies," ective' ., -1

1-26Cop right ; <11 =earson ducation >nc$

Page 36: OM Exam 1 Chap 1-5

8/13/2019 OM Exam 1 Chap 1-5

http://slidepdf.com/reader/full/om-exam-1-chap-1-5 36/187

1 ) ,perations strategies are implemented in the same &a in all t pes of organi!ations$%ns&er' F%.S*iff' Topic' *eveloping missions and strategies," ective' ., -1

12) 7perience differentiation is an e7tension of product differentiation accomplished " using peopleBs five senses to create an e7perience rather than simpl providing a service$%ns&er' TR(*iff' Topic' %chieving competitive advantage through operations," ective' ., -

15) %n organi!ationBs a"ilit to generate uni#ue advantages over competitors is central to asuccessful strateg implementation$%ns&er' TR(*iff' Topic' %chieving competitive advantage through operations," ective' ., -

16) .o&-cost leadership is the a"ilit to distinguish the offerings of the organi!ation in an &athat the customer perceives as adding value$%ns&er' F%.S*iff' Topic' %chieving competitive advantage through operations," ective' ., -2

19) :ost services are tangi"leG this factor determines ho& the ten decisions of operationsmanagement are handled differentl for goods than for services$%ns&er' F%.S*iff' Topic' Ten strategic ,: decisions," ective' ., -2

1?) The relative importance of each of the ten operations decisions depends on the ratio of goodsand services in an organi!ation$%ns&er' TR(*iff' Topic' Ten strategic ,: decisions," ective' ., -2

1@) *ecisions that involve &hat is to "e made and &hat is to "e purchased fall under the headingof suppl -chain management$%ns&er' TR(*iff' Topic' Ten strategic ,: decisions," ective' ., -21A) :anufacturing organi!ations have ten strategic ,: decisions &hile service organi!ations

1-29Cop right ; <11 =earson ducation >nc$

Page 37: OM Exam 1 Chap 1-5

8/13/2019 OM Exam 1 Chap 1-5

http://slidepdf.com/reader/full/om-exam-1-chap-1-5 37/187

have onl eight$%ns&er' F%.S*iff' 1Topic' Ten strategic ,: decisions," ective' ., -2

<) rrors made &ithin the location decision area ma over&helm efficiencies in other areas$%ns&er' TR(*iff' Topic' Ten strategic ,: decisions," ective' ., -2

1) South&est %irlinesB core competence is operations$%ns&er' TR(*iff' Topic' Strateg development and implementation

," ective' ., -5) Key success factors and core competencies are s non ms$

%ns&er' F%.S*iff' Topic' Strateg development and implementation," ective' ., -5

2) S3,T anal sis is a method of determining e7ternal strengths and &eaknesses and internalopportunities and threats$%ns&er' F%.S*iff' Topic' Strateg development and implementation," ective' no .,

5) For the greatest chance of success an organi!ationBs operations management strateg mustsupport the compan Bs strateg $%ns&er' TR(*iff' Topic' Strateg development and implementation," ective' ., -5

6) e Success Factors are those activities that are ke to achieving competitive advantage$%ns&er' TR(*iff' Topic' Strateg development and implementation," ective' ., -5

1-2?Cop right ; <11 =earson ducation >nc$

Page 38: OM Exam 1 Chap 1-5

8/13/2019 OM Exam 1 Chap 1-5

http://slidepdf.com/reader/full/om-exam-1-chap-1-5 38/187

9) % multinational corporation has e7tensive international "usiness involvements$%ns&er' TR(*iff' 1Topic' +lo"al operations strateg options%%CS0' :ulticultural and *iversit

," ective' ., -6

?) The multidomestic ,: strateg ma7imi!es local responsiveness &hile achieving asignificant cost advantage$%ns&er' F%.S*iff' Topic' +lo"al operations strateg options%%CS0' :ulticultural and *iversit," ective' ., -6

@) Firms using the glo"al strateg can "e thought of as 8&orld companies$8

%ns&er' F%.S*iff' Topic' +lo"al operations strateg options%%CS0' :ulticultural and *iversit," ective' ., -6

:ultiple Choice

1) 3hich of the follo&ing statements regarding the *reamliner ?@? is true4%) 0oeing has found partners in over a do!en countries$0) The ne& aircraft incorporates a &ide range of aerospace technologies$C) The ne& aircraft uses engines from not one "ut t&o manufacturers$*) 0oeing &ill add onl < to 2< percent of the aircraftBs value$) %ll of the a"ove are true$%ns&er' *iff' Topic' +lo"al compan profile," ective' no .,

) 0oeingBs ne& ?@? *reamliner %) is assem"led in 3ashington *$C$0) uses engines from JapanC) has its fuselage sections "uilt in %ustralia*) has increased efficienc from ne& engine technolog) results from a partnership of a"out a do!en companies%ns&er' **iff' Topic' +lo"al compan profile," ective' no .,

1-2@Cop right ; <11 =earson ducation >nc$

Page 39: OM Exam 1 Chap 1-5

8/13/2019 OM Exam 1 Chap 1-5

http://slidepdf.com/reader/full/om-exam-1-chap-1-5 39/187

2) 7amples of response to the glo"al environment include%) 0oeingBs &orld&ide sales and production0) 0ennetonBs fle7i"ilit in design production and distri"utionC) a Chinese manufacturer Haier opening plants in the (nited States*) FordBs partnerships &ith Eolvo and :a!da

) all of the a"ove%ns&er' *iff' 1Topic' % glo"al vie& of operations%%CS0' :ulticultural and *iversit," ective' ., -6

5) 3hich of the follo&ing is an e7ample of glo"ali!ation of operations strateg 4%) 0oeingBs *reamliner has engines &ith higher fuel/pa load efficienc $0) FordBs ne& auto models have dent-resistant panels$C) % Chinese manufacturer Haier no& operates plants in the (nited States$

*) Hard Rock Café provides an 8e7perience differentiation8 at its restaurants$) %ll of the a"ove are e7amples$%ns&er' C*iff' Topic' % glo"al vie& of operations%%CS0' :ulticultural and *iversit," ective' ., -6

6) Cost cutting in international operations can take place "ecause of %) lo&er ta7es and tariffs0) lo&er &age scalesC) lo&er indirect costs*) less stringent regulations) all of the a"ove%ns&er' *iff' Topic' % glo"al vie& of operations%%CS0' :ulticultural and *iversit," ective' ., -6

9) 3hich of the follo&ing did the authors not suggest as a reason for glo"ali!ing operations4%) reduce costs0) improve the suppl chainC) stockholder approval ratings*) understand markets) Ione of the a"ove &ere suggested$%ns&er' C*iff' Topic' % glo"al vie& of operations%%CS0' :ulticultural and *iversit," ective' ., -6?) :ultinational organi!ations can shop from countr to countr and cut costs through

1-2ACop right ; <11 =earson ducation >nc$

Page 40: OM Exam 1 Chap 1-5

8/13/2019 OM Exam 1 Chap 1-5

http://slidepdf.com/reader/full/om-exam-1-chap-1-5 40/187

%) lo&er &age scales0) lo&er indirect costsC) less stringent regulations*) lo&er ta7es and tariffs) all of the a"ove

%ns&er' *iff' Topic' % glo"al vie& of operations%%CS0' :ulticultural and *iversit," ective' ., -6

@) The term ma#uiladora is most s non mous &ith%) free trade !ones in :e7ico0) Chinese forced la"or campsC) home-"ased or cottage industr*) areas that do not meet ($S$ standards for &orkplace safet and pollution

) none of the a"ove%ns&er' %*iff' Topic' % glo"al vie& of operations%%CS0' :ulticultural and *iversit," ective' no .,

A) 3hich of the follo&ing is true of ma#uiladoras4%) The discourage foreign "usinesses from outsourcing$0) The assess tariffs onl on the value-added &ork done$C) The originated in China$*) none of the a"ove) all of the a"ove%ns&er' 0*iff' Topic' % glo"al vie& of operations," ective' no .,

1<) 3hich of the follo&ing represent reasons for glo"ali!ing operations4%) to gain improvements in the suppl chain0) to improve operationsC) to e7pand a productBs life c cle*) to attract and retain glo"al talent) all of the a"ove%ns&er' *iff' Topic' % glo"al vie& of operations%%CS0' :ulticultural and *iversit," ective' ., -6

11) 3hich of the follo&ing does not represent a reason for glo"ali!ing operations4%) reduce costs

1-5<Cop right ; <11 =earson ducation >nc$

Page 41: OM Exam 1 Chap 1-5

8/13/2019 OM Exam 1 Chap 1-5

http://slidepdf.com/reader/full/om-exam-1-chap-1-5 41/187

0) improve the suppl chainC) reduce responsiveness*) attract and retain glo"al talent) Ione of the a"ove are valid reasons for glo"ali!ing operations$%ns&er' C

*iff' Topic' % glo"al vie& of operations%%CS0' :ulticultural and *iversit," ective' ., -6

1 ) I%FT% seeks to%) su"stitute cheap la"or in :e7ico for e7pensive la"or in the (nited States0) cur" illegal immigration from :e7ico to the (nited StatesC) phase out all trade and tariff "arriers "et&een Iorth %merica and South %merica*) phase out all trade and tariff "arriers "et&een the (nited States Canada and :e7ico) %ll of the a"ove are I%FT% goals$

%ns&er' **iff' Topic' % glo"al vie& of operations%%CS0' :ulticultural and *iversit," ective' no .,

12) 3ith reference to cultural and ethical issues the 3orld Trade ,rgani!ation has%) succeeded in providing e#ual protection of intellectual propert among nations0) made progress in providing e#ual protection of intellectual propert among nationsC) phased out all trade and tariff "arriers "et&een the (nited States and :e7ico*) eliminated slave la"or and child la"or ) pla ed little role in addressing cultural and ethical issues among nations%ns&er' 0*iff' 2Topic' % glo"al vie& of operations%%CS0' thical Reasoning," ective' no .,

15) 3hich of the follo&ing is true a"out "usiness strategies4%) %n organi!ation should stick &ith its strateg for the life of the "usiness$0) %ll firms &ithin an industr &ill adopt the same strateg $C) 3ell defined missions make strateg development much easier$*) Strategies are formulated independentl of S3,T anal sis$) ,rgani!ational strategies depend on operations strategies$%ns&er' C*iff' Topic' *eveloping missions and strategies," ective' ., -1

16) 3hich of the follo&ing activities takes place most immediatel once the mission has "eendeveloped4%) The firm develops alternative or "ack-up missions in case the original mission fails$

1-51Cop right ; <11 =earson ducation >nc$

Page 42: OM Exam 1 Chap 1-5

8/13/2019 OM Exam 1 Chap 1-5

http://slidepdf.com/reader/full/om-exam-1-chap-1-5 42/187

0) The functional areas develop their functional area strategies$C) The functional areas develop their supporting missions$*) The ten ,: decision areas are prioriti!ed$) ,perational tactics are developed$%ns&er' C

*iff' Topic' *eveloping missions and strategies," ective' ., -1

19) 3hich of the follo&ing statements a"out organi!ational missions is alse 4%) The reflect a compan Bs purpose$0) The indicate &hat a compan intends to contri"ute to societ $C) The are formulated after strategies are kno&n$*) The define a compan Bs reason for e7istence$) The provide guidance for functional area missions$%ns&er' C

*iff' Topic' *eveloping missions and strategies," ective' ., -1

1?) 3hat term descri"es ho& an organi!ation e7pects to achieve its missions and goals4%) conditional e7pectation0) tacticC) S3,T*) strateg) competitive advantage%ns&er' **iff' 1Topic' *eveloping missions and strategies," ective' ., -1

1@) The fundamental purpose of an organi!ationBs mission statement is to%) create a good human relations climate in the organi!ation0) define the organi!ationBs purpose in societC) define the operational structure of the organi!ation*) generate good pu"lic relations for the organi!ation) define the functional areas re#uired " the organi!ation%ns&er' 0*iff' Topic' *eveloping missions and strategies," ective' ., -1

1-5Cop right ; <11 =earson ducation >nc$

Page 43: OM Exam 1 Chap 1-5

8/13/2019 OM Exam 1 Chap 1-5

http://slidepdf.com/reader/full/om-exam-1-chap-1-5 43/187

1A) 3hich of the follo&ing is true 4%) Corporate mission is shaped " functional strategies$0) Corporate strateg is shaped " functional strategies$C) Functional strategies are shaped " corporate strateg $*) 7ternal conditions are shaped " corporate mission$

) Functional area missions are merged to "ecome the organi!ational mission$%ns&er' C*iff' 2Topic' *eveloping missions and strategies," ective' ., -1

<) 3hich of the international operations strategies involves a focus on high cost reductions andlo& local responsiveness4

%) international strateg0) glo"al strategC) transnational strateg

*) multidomestic strateg) none of the a"ove%ns&er' 0*iff' Topic' +lo"al operations strateg options%%CS0' :ulticultural and *iversit," ective' ., -6

1) 3hich of the follo&ing is least likel to "e a Cost .eadership competitive advantage4%) lo& overhead0) effective capacit useC) inventor management*) "road product line) mass production%ns&er' **iff' Topic' %chieving competitive advantage through operations," ective' ., -

) %ccording to the authors &hich of the follo&ing strategic concepts allo& firms to achievetheir missions4%) productivit efficienc and #ualit leadership0) differentiation cost leadership and #uick responseC) differentiation #ualit leadership and #uick response*) distinctive competenc cost leadership and e7perience) differentiation distinctive competenc #ualit leadership and capacit%ns&er' 0*iff' Topic' %chieving competitive advantage through operations," ective' ., -

2) % firm can effectivel use its operations function to ield competitive advantage through all1-52

Cop right ; <11 =earson ducation >nc$

Page 44: OM Exam 1 Chap 1-5

8/13/2019 OM Exam 1 Chap 1-5

http://slidepdf.com/reader/full/om-exam-1-chap-1-5 44/187

of the follo&ing e7cept%) customi!ation of the product0) setting e#uipment utili!ation goals "elo& the industr averageC) speed of deliver*) constant innovation of ne& products

) maintaining a variet of product options%ns&er' 0*iff' Topic' %chieving competitive advantage through operations," ective' ., -

5) 3hich of the follo&ing has progressed the furthest along its product life c cle4%) drive-thru restaurants0) 0oeing ?@?C) i=ods*) T&itter

) "o7 29<%ns&er' %*iff' Topic' >ssues in operations strateg," ective' no .,

6) The a"ilit of an organi!ation to produce goods or services that have some uni#ueness intheir characteristics is%) mass production0) time-"ased competitionC) competing on productivit*) competing on fle7i"ilit) competing on differentiation%ns&er' *iff' Topic' %chieving competitive advantage through operations," ective' ., -

9) 3hich of the international operations strategies involves a focus on lo& cost reductions andhigh local responsiveness4%) international strateg0) glo"al strategC) transnational strateg*) multidomestic strateg) none of the a"ove%ns&er' **iff' Topic' +lo"al operations strateg options%%CS0' :ulticultural and *iversit," ective' ., -6?) % strateg is aKn)%) set of opportunities in the marketplace

1-55Cop right ; <11 =earson ducation >nc$

Page 45: OM Exam 1 Chap 1-5

8/13/2019 OM Exam 1 Chap 1-5

http://slidepdf.com/reader/full/om-exam-1-chap-1-5 45/187

0) "road statement of purposeC) simulation used to test various product line options*) plan for cost reduction) action plan to achieve the mission%ns&er'

*iff' Topic' %chieving competitive advantage through operations," ective' ., -

@) 3hich of the follo&ing statements "est characteri!es deliver relia"ilit 4%) a compan that al&a s delivers on the same da of the &eek 0) a compan that al&a s delivers at the promised timeC) a compan that delivers more fre#uentl than its competitors*) a compan that delivers faster than its competitors) a compan that has a computeri!ed deliver scheduling s stem%ns&er' 0

*iff' 2Topic' %chieving competitive advantage through operations," ective' ., -

A) 3hich of the follo&ing is an e7ample of competing on the "asis of differentiation4%) % firm manufactures its product &ith less ra& material &aste than its competitors do$0) % firmBs products are introduced into the market faster than its competitorsB products are$C) % firmBs distri"ution net&ork routinel delivers its product on time$*) % firm offers more relia"le products than its competitors do$) % firm advertises more than its competitors do$%ns&er' **iff' Topic' %chieving competitive advantage through operations," ective' ., -

2<) The a"ilit of an organi!ation to produce services that " utili!ing the consumerBs fivesenses have some uni#ueness in their characteristics is%) mass production0) time-"ased competitionC) differentiation*) fle7i"le response) e7perience differentiation%ns&er' *iff' Topic' %chieving competitive advantage through operations," ective' ., -

1-56Cop right ; <11 =earson ducation >nc$

Page 46: OM Exam 1 Chap 1-5

8/13/2019 OM Exam 1 Chap 1-5

http://slidepdf.com/reader/full/om-exam-1-chap-1-5 46/187

21) 3hich of the follo&ing "est descri"es 8e7perience differentiation84%) immerses consumers in the deliver of a service0) uses peopleBs five senses to enhance the serviceC) complements ph sical elements &ith visual and sound elements*) consumers ma "ecome active participants in the product or service

) %ll of the a"ove are elements of e7perience differentiation$%ns&er' *iff' 1Topic' %chieving competitive advantage through operations," ective' ., -

2 ) 7perience *ifferentiation%) isolates the consumer from the deliver of a service0) is an e7tension of product differentiation in the service sector C) uses onl the consumerBs senses of vision and sound*) keeps consumers from "ecoming active participants in the service

) is the same as product differentiation "ut applied in the service sector %ns&er' 0*iff' 1Topic' %chieving competitive advantage through operations," ective' ., -

22) 3hich of the follo&ing is the "est e7ample of competing on lo&-cost leadership4%) % firm produces its product &ith less ra& material &aste than its competitors$0) % firm offers more relia"le products than its competitors$C) % firmBs products are introduced into the market faster than its competitorsB products$*) % firmBs research and development department generates man ideas for ne& products$) % firm advertises more than its competitors$%ns&er' %*iff' 2Topic' %chieving competitive advantage through operations," ective' ., -

25) Fran! Colru t has achieved lo&-cost leadership through%) Spartan head#uarters0) a"sence of voice mailC) lo&-cost facilities*) no "ackground music) all of the a"ove%ns&er' *iff' 1Topic' %chieving competitive advantage through operations," ective' ., -

1-59Cop right ; <11 =earson ducation >nc$

Page 47: OM Exam 1 Chap 1-5

8/13/2019 OM Exam 1 Chap 1-5

http://slidepdf.com/reader/full/om-exam-1-chap-1-5 47/187

26) Fran! Colru t has achieved lo&-cost leadership through%) effective use of voice mail0) plastic not paper shopping "agsC) "ackground music that su"tl encourages shoppers to "u more*) converting factories garages and theaters into retail outlets

) e7clusive use of the uro%ns&er' **iff' 1Topic' %chieving competitive advantage through operations," ective' ., -

29) 3hich of the follo&ing is an e7ample of competing on #uick response4%) % firm produces its product &ith less ra& material &aste than its competitors$0) % firm offers more relia"le products than its competitors$C) % firmBs products are introduced into the market faster than its competitorsB products$*) % firm utili!es is capacit more effectivel than its competitors$

) % firm advertises more than its competitors$%ns&er' C*iff' 2Topic' %chieving competitive advantage through operations," ective' ., -

2?) % firm producing a good is more likel to have &hich set of the follo&ing characteristicscompared to a firm providing a service4%) man su" ective #ualit standards tangi"le product and locate a&a from customers0) man su" ective #ualit standards intangi"le product and locate a&a from customersC) man o" ective #ualit standards tangi"le product and locate near customers*) man o" ective #ualit standards tangi"le product and locate a&a from customers) man o" ective #ualit standards intangi"le product and locate a&a from customers%ns&er' **iff' Topic' Ten strategic ,: decisions," ective' ., -2

2@) 3hich of the follo&ing influences la out design4%) inventor re#uirements0) capacit needsC) personnel levels*) technolog decisions) %ll of the a"ove influence la out decisions$%ns&er' *iff' Topic' Ten strategic ,: decisions," ective' ., -2

1-5?Cop right ; <11 =earson ducation >nc$

Page 48: OM Exam 1 Chap 1-5

8/13/2019 OM Exam 1 Chap 1-5

http://slidepdf.com/reader/full/om-exam-1-chap-1-5 48/187

Page 49: OM Exam 1 Chap 1-5

8/13/2019 OM Exam 1 Chap 1-5

http://slidepdf.com/reader/full/om-exam-1-chap-1-5 49/187

52) 3hich of the follo&ing &ill more likel locate near their customers4%) an automo"ile manufacturer 0) an aluminum manufacturer C) an insurance compan head#uarters*) a medical clinic

) %ll of the a"ove &ill tend to locate near their customers$%ns&er' **iff' Topic' Ten strategic ,: decisions," ective' ., -2

55) 3hich of the follo&ing statements concerning the operations management decision isrelevant to services4%) There are man o" ective #ualit standards$0) The customer is not involved in most of the process$C) The &ork forceBs technical skills are ver important$

*) .a"or standards var depending on customer re#uirements$) %"ilit to inventor ma allo& the leveling of the output rates$%ns&er' **iff' Topic' Ten strategic ,: decisions," ective' ., -2

56) 3hich of these organi!ations is likel to have the most important inventor decisions4%) a marketing research firm0) a lo"" ing agencC) a management consulting firm*) an aluminum manufacturer ) a la& firm%ns&er' **iff' Topic' Ten strategic ,: decisions," ective' ., -2

59) 0efore esta"lishing and implementing strateg a resources vie& &ould ensure that &hich ofthe follo&ing resources are availa"le'%) financial0) ph sicalC) human*) technological) all of the a"ove%ns&er' *iff' Topic' >ssues in operations strateg," ective' ., -5

1-5ACop right ; <11 =earson ducation >nc$

Page 50: OM Exam 1 Chap 1-5

8/13/2019 OM Exam 1 Chap 1-5

http://slidepdf.com/reader/full/om-exam-1-chap-1-5 50/187

5?) =orterBs Five Forces :odel is used to evaluate competition "ased on &hich 6 aspects4%) research and development cost legal regulations suppliers customers0) immediate rivals potential entrants customers suppliers and su"stitute productsC) potential entrants customers suppliers legal regulations and cost*) immediate rivals potential entrants cost su"stitute products and legal regulations

) none of the a"ove%ns&er' 0*iff' Topic' >ssues in operations strateg," ective' ., -5

5@) =orterBs Five Forces :odel contains &hich of the follo&ing4%) immediate rivals0) potential entrantsC) customers*) suppliers

) all of the %"ove%ns&er' *iff' Topic' >ssues in operations strateg," ective' ., -5

5A) Ealue-chain anal sis is used to identif activities that represent or can develop into the corecompetencies of a "usiness$ 3ho introduced this concept4%) Ford0) To otaC) =orter *) Smith) Ione of the a"ove%ns&er' C*iff' Topic' >ssues in operations strateg," ective' ., -5

6<) Ealue-chain anal sis can "e used to determine if a "usiness is adding uni#ue value in &hichof the follo&ing areas4%) product research0) human resourcesC) process innovation*) #ualit management) all of the a"ove%ns&er' *iff' Topic' >ssues in operations strateg," ective' ., -5

1-6<Cop right ; <11 =earson ducation >nc$

Page 51: OM Exam 1 Chap 1-5

8/13/2019 OM Exam 1 Chap 1-5

http://slidepdf.com/reader/full/om-exam-1-chap-1-5 51/187

61) 3hich of the follo&ing is not part of value-chain anal sis4%) product research0) #ualit managementC) suppl -chain management*) marketing

) human resources%ns&er' **iff' Topic' >ssues in operations strateg," ective' ., -5

6 ) 3hich of the follo&ing environmental factors has not "een a factor in :icrosoftBs changingstrateg 4%) +oogle0) securit issuesC) faster processors

*) the >nternet) none of the a"ove%ns&er' *iff' Topic' >ssues in operations strateg," ective' ., -5

62) Standardi!ation is an appropriate strateg in &hich stage of the product life c cle4%) introduction0) gro&thC) maturit*) decline) retirement%ns&er' C*iff' Topic' >ssues in operations strateg," ective' ., -5

65) Cost minimi!ation is an appropriate strateg in &hich stage of the product life c cle4%) introduction0) gro&thC) adolescence*) decline) retirement%ns&er' **iff' Topic' >ssues in operations strateg," ective' ., -5

1-61Cop right ; <11 =earson ducation >nc$

Page 52: OM Exam 1 Chap 1-5

8/13/2019 OM Exam 1 Chap 1-5

http://slidepdf.com/reader/full/om-exam-1-chap-1-5 52/187

66) 3hich of the follo&ing ,: Strateg />ssues should a firm &ith a product in the maturitstage of its life c cle "e least concerned &ith at the present time4%) increase capacit0) long production runsC) standardi!ation

*) cost cutting) fe&er rapid product changes%ns&er' %*iff' Topic' >ssues in operations strateg," ective' no .,

69) 3hich of the follo&ing is least likel to result in a strateg change4%) change in the compan Bs financial situation0) a compan Bs adoption of ne& technologC) change in the product life c cle

*) change in the competitive environment) change in o" scheduling techni#ues%ns&er' *iff' Topic' >ssues in operations strateg," ective' ., -5

6?) 3hich of the follo&ing statements is most correct4%) SFs are often necessar "ut not sufficient for competitive advantage$0) SFs are often sufficient "ut not necessar for competitive advantage$C) SFs are neither necessar nor sufficient for competitive advantage$*) SFs are "oth necessar and sufficient for competitive advantage$) Ione of the a"ove statements is correct$%ns&er' %*iff' 2Topic' Strateg development and implementation," ective' ., -5

6@) +iven the position of the i=od in the gro&th stage of its life c cle &hich of the follo&ing,: Strateg />ssues should the makers of i=ods "e least concerned &ith at the current time4%) forecasting0) cost cuttingC) increasing capacit*) product and process relia"ilit) enhancing distri"ution%ns&er' 0*iff' Topic' >ssues in operations strateg," ective' ., -5

1-6Cop right ; <11 =earson ducation >nc$

Page 53: OM Exam 1 Chap 1-5

8/13/2019 OM Exam 1 Chap 1-5

http://slidepdf.com/reader/full/om-exam-1-chap-1-5 53/187

6A) 3hich of these organi!ations is likel to have the most comple7 inventor decisions4%) a marketing research firm0) a stock "rokerage firmC) a management consulting firm*) a computer manufacturing compan

) a high school%ns&er' **iff' Topic' Strateg development and implementation," ective' ., -5

9<) The three steps of the operations managerBs o" in order are'%) develop the strateg esta"lish the organi!ational structure find the right staff 0) develop the strateg find the right staff esta"lish the organi!ational structureC) find the right staff esta"lish the organi!ational structure develop the strateg*) find the right staff develop the strateg esta"lish the organi!ational structure

) esta"lish the organi!ational structure find the right staff develop the strateg%ns&er' %*iff' Topic' Strateg development and implementation," ective' ., -5

91) 3hen developing the operations strateg for a ne& manufacturing organi!ation one of themost important considerations is that it%) re#uires minimal capital investment0) utili!es as much automation as possi"leC) utili!es an e#ual "alance of la"or and automation*) supports the overall competitive strateg of the compan) none of the a"ove%ns&er' **iff' Topic' Strateg development and implementation," ective' ., -5

9 ) 3hich of the international operations strategies involves high cost reductions and high localresponsiveness4%) international strateg0) glo"al strategC) transnational strateg*) multidomestic strateg) none of the a"ove%ns&er' C*iff' Topic' +lo"al operations strateg options%%CS0' :ulticultural and *iversit," ective' ., -6

92) 3hich of the international operations strategies involves lo& cost reductions and lo& local1-62

Cop right ; <11 =earson ducation >nc$

Page 54: OM Exam 1 Chap 1-5

8/13/2019 OM Exam 1 Chap 1-5

http://slidepdf.com/reader/full/om-exam-1-chap-1-5 54/187

responsiveness4%) international strateg0) glo"al strategC) transnational strateg*) multidomestic strateg

) none of the a"ove%ns&er' %*iff' Topic' +lo"al operations strateg options%%CS0' :ulticultural and *iversit," ective' ., -6

95) 3hich of the international operations strategies uses import/e7port or licensing of e7isting products4%) international strateg0) glo"al strateg

C) transnational strateg*) multidomestic strateg) none of the a"ove%ns&er' %*iff' Topic' +lo"al operations strateg options%%CS0' :ulticultural and *iversit," ective' ., -6

96) 3hich of the international operations strategies uses the e7isting domestic model glo"all 4%) international strateg0) glo"al strategC) transnational strateg*) multidomestic strateg) none of the a"ove%ns&er' **iff' Topic' +lo"al operations strateg options%%CS0' :ulticultural and *iversit," ective' ., -6

1-65Cop right ; <11 =earson ducation >nc$

Page 55: OM Exam 1 Chap 1-5

8/13/2019 OM Exam 1 Chap 1-5

http://slidepdf.com/reader/full/om-exam-1-chap-1-5 55/187

Page 56: OM Exam 1 Chap 1-5

8/13/2019 OM Exam 1 Chap 1-5

http://slidepdf.com/reader/full/om-exam-1-chap-1-5 56/187

) are areas located along the ($S$-:e7ico "order in &hich factories receive preferentialtariff treatment$%ns&er' :a#uiladoras*iff'

Topic' % glo"al vie& of operations%%CS0' :ulticultural and *iversit," ective' no .,

2) is a free trade agreement among Canada :e7ico and the (nited States$%ns&er' I%FT%--Iorth %merican Free Trade %greement*iff' Topic' % glo"al vie& of operations%%CS0' :ulticultural and *iversit," ective' no .,

5) and are t&o issues &here significant cultural differences are largeand progress to&ard glo"al uniformit has "een slo&$%ns&er' 0ri"er protection of intellectual propert*iff' 1Topic' +lo"al compan profile%%CS0' thical Reasoning," ective' no .,

6) %n organi!ation that has &orked to achieve glo"al uniformit in cultural and ethical issuessuch as "ri"er child la"or and environmental regulations is $%ns&er' the 3orld Trade ,rgani!ation*iff' 1Topic' +lo"al compan profile%%CS0' thical Reasoning," ective' no .,

9) >n goods-producing organi!ations and ma "einventoried$%ns&er' ra& materials &ork-in-process finished goods*iff' Topic' Ten strategic ,: decisions," ective' ., -2

?) Strateg is not static "ut d namic "ecause of changes in the and $%ns&er' organi!ation environment*iff' Topic' >ssues in operations strateg," ective' ., -1

1-69Cop right ; <11 =earson ducation >nc$

Page 57: OM Exam 1 Chap 1-5

8/13/2019 OM Exam 1 Chap 1-5

http://slidepdf.com/reader/full/om-exam-1-chap-1-5 57/187

@) is the stage in product life c cle at &hich it is a poor time to change #ualit $%ns&er' :aturit*iff' Topic' >ssues in operations strateg," ective' ., -5

A) The is ho& an organi!ation e7pects to achieve its missions and goals$%ns&er' strateg*iff' Topic' *eveloping missions and strategies," ective' ., -1

1<) The creation of a uni#ue advantage over competitors is called a $%ns&er' competitive advantage*iff' Topic' %chieving competitive advantage through operations

," ective' ., -11) Service organi!ations can immerse the consumer in the service or have the consumer

"ecome a participant in the service as the practice $%ns&er' e7perience differentiation*iff' Topic' %chieving competitive advantage through operations," ective' ., -

1 ) Competitive advantage in operations can "e achieved " and/or $%ns&er' differentiation lo& cost response*iff' Topic' %chieving competitive advantage through operations," ective' ., -

12) %Kn) is a firm that has e7tensive involvement in international "usiness o&ningor controlling facilities in more than one countr $%ns&er' :IC--multinational corporation*iff' Topic' +lo"al operations strateg options%%CS0' :ulticultural and *iversit," ective' ., -6

15) The strateg utili!es a standardi!ed product across countries$%ns&er' glo"al*iff' Topic' +lo"al operations strateg options%%CS0' :ulticultural and *iversit," ective' ., -6

1-6?Cop right ; <11 =earson ducation >nc$

Page 58: OM Exam 1 Chap 1-5

8/13/2019 OM Exam 1 Chap 1-5

http://slidepdf.com/reader/full/om-exam-1-chap-1-5 58/187

16) The strateg uses e7ports and licenses to penetrate glo"all $%ns&er' international*iff' Topic' +lo"al operations strateg options%%CS0' :ulticultural and *iversit

," ective' ., -619) The strateg uses su"sidiaries franchises or oint ventures &ith su"stantialindependence$%ns&er' multidomestic*iff' Topic' +lo"al operations strateg options%%CS0' :ulticultural and *iversit," ective' ., -6

1?) The strateg descri"es a condition in &hich material people and ideas cross ortransgress national "oundaries$%ns&er' transnational*iff' Topic' +lo"al operations strateg options%%CS0' :ulticultural and *iversit," ective' ., -6

1@) >dentif five countries from &hich 0oeingBs ?@? *reamliner has suppliers$%ns&er' France +erman ( >tal Japan China South orea S&eden ($S$*iff' 1Topic' +lo"al compan profile%%CS0' :ulticultural and *iversit," ective' no .,

1A) >dentif five parts &hich 0oeingBs ?@? *reamliner gets from glo"al suppliersG match each part &ith the countr that supplies it$%ns&er' FranceMlanding gear +erman Minterior lighting ( Mfuel pumps or electronics >tal M

part of fuselage or hori!ontal sta"ili!er JapanM&ing "o7 or h draulic actuators ChinaMrudder orgeneral parts South oreaM&ingtips S&edenMcargo and access doors ($S$M+ engines*iff' Topic' +lo"al compan profile%%CS0' :ulticultural and *iversit," ective' no .,

<) Ho& can glo"al operations improve the suppl chain4%ns&er' The suppl chain can often "e improved " locating facilities in countries &hereuni#ue resources e7ist$*iff' Topic' % glo"al vie& of operations%%CS0' :ulticultural and *iversit," ective' no .,1) Ho& do glo"al operations attract ne& markets4

1-6@Cop right ; <11 =earson ducation >nc$

Page 59: OM Exam 1 Chap 1-5

8/13/2019 OM Exam 1 Chap 1-5

http://slidepdf.com/reader/full/om-exam-1-chap-1-5 59/187

%ns&er' Since international operations re#uire local interaction &ith customers suppliers andother competitive "usinesses international firms inevita"l learn a"out uni#ue opportunities forne& products and services$*iff' Topic' % glo"al vie& of operations

%%CS0' :ulticultural and *iversit," ective' no .,

) State t&o e7amples of cultural and ethical issues that face operations managers in a glo"alenvironment$%ns&er' Student responses &ill var "ut there are several issues on &hich there are &idedifferences from countr to countr culture to culture$ %mong those listed in the te7t are "ri"erchild la"or slave la"or and intellectual propert rights$ Students ma "ring for&ard from anearlier chapter issues such as environmental regulation or safe &ork environment and ma raiseissues such as product safet $*iff' Topic' % glo"al vie& of operations%%CS0' thical Reasoning," ective' no .,

2) 3hat is the difference "et&een a firmBs mission and its strateg 4%ns&er' % firmBs mission is its purpose or rationale for an organi!ationBs e7istence &hereas afirmBs strateg is ho& it e7pects to achieve its mission and goals$*iff' Topic' *eveloping missions and strategies," ective' ., -1

5) Since the earl 1AA<s residents in a num"er of developing countries have overcome culturereligious ethnic and political productivit "arriers$ These disappearing "arriers coupled &ithsimultaneous advances in technolog relia"le shipping and cheap communication have all led tothe gro&th of &hat three things4%ns&er'1$ 3orld trade$$ +lo"al capital markets$2$ >nternational movement of people$*iff' Topic' % glo"al vie& of operations%%CS0' :ulticultural and *iversit," ective' no .,

1-6ACop right ; <11 =earson ducation >nc$

Page 60: OM Exam 1 Chap 1-5

8/13/2019 OM Exam 1 Chap 1-5

http://slidepdf.com/reader/full/om-exam-1-chap-1-5 60/187

6) =rovide an e7ample of an organi!ation that achieves competitive advantage throughe7perience differentiation$ 7plain$%ns&er' %ns&ers &ill var "ut *isne and Hard Rock Café are illustrated in the te7t$Competing on e7perience differentiation implies providing uni#ueness to our service offeringthrough immersion of the consumer into the service &ith visual or sound elements to turn the

service into an e7perience$*iff' Topic' %chieving competitive advantage through operations," ective' ., -

9) 3ith regard to the scheduling decision ho& are goods-producing organi!ations differentfrom service companies4 *iscuss$%ns&er' +oods-producing companies' the a"ilit to inventor ma allo& leveling the outputratesG service companies' primaril concerned &ith meeting the customerBs immediate schedule$*iff' 2Topic' Ten strategic ,: decisions

," ective' ., -2?) Ho& do goods and services differ &ith regard to handling the #ualit decision4%ns&er' There are man o" ective #ualit standards for goods &hereas there are mansu" ective #ualit standards for services$*iff' Topic' Ten strategic ,: decisions," ective' ., -2

@) 3hat is the difference "et&een goods and services in terms of their location selection4%ns&er' :anufacturers of goods ma need to "e located close to ra& materials or la"or force$Services on the other hand t picall are located close to the customer$*iff' Topic' Ten strategic ,: decisions," ective' ., -2

A) Ho& has Fran! Colru t achieved lo&-cost leadership4 .ist three specific e7amples anddescri"e each "riefl $%ns&er' Several methods are listed in the te7t$ %mong these are no shopping "ags dim lightingno voice mail conversion of older "uildings Spartan offices$*iff' Topic' %chieving competitive advantage through operations," ective' ., -

2<) *efine core competencies.%ns&er' % set of skills talents and activities that a firm does particularl &ell$*iff' Topic' Strateg development and implementation," ective' ., -5

1-9<Cop right ; <11 =earson ducation >nc$

Page 61: OM Exam 1 Chap 1-5

8/13/2019 OM Exam 1 Chap 1-5

http://slidepdf.com/reader/full/om-exam-1-chap-1-5 61/187

21) For &hat t pe of organi!ation might the location decision area "e the least important of itsten decision areas4 For &hat t pe of organi!ation might the location decision "e the mostimportant of the ten decision areas4 *iscussG augment our response &ith e7amples$%ns&er' The relationship "et&een the organi!ation and its suppliers or its customers is ke $ >fthat relationship is ver &eak Kas in no transportation costs or customers can reach the firm from

an location) location diminishes in importance$ >f that relationship is strong Kuni#ueness ofsite high transportation costs customers &ill not travel far) location increases in importance$8.east8 e7amples' Telemarketing firm ta7 help-line >nternet sales$ 8:ost8 e7amples' goldmine oil &ell ski resort$*iff' 2Topic' Ten strategic ,: decisions%%CS0' Reflective Thinking," ective' ., -2

2 ) 3hat is S3,T anal sis4 .ist its four elements and descri"e its purpose$%ns&er' The four elements of S3,T are strengths &eaknesses opportunities and threats$ >ts

purpose is to ma7imi!e opportunities and minimi!e threats in the environment &hilema7imi!ing the advantages of the organi!ationBs strengths and minimi!ing the &eaknesses$*iff' Topic' Strateg development and implementation," ective' ., -1

22) %re strategies static or d namic4 3hat are the forces that lead to this result4%ns&er' Strategies should "e d namic "ecause of changes &ithin the organi!ation and changesin the environment$*iff' Topic' Strateg development and implementation," ective' ., -1

25) >dentif and e7plain the four "asic glo"al operations strategies$ +ive an e7ample of eachstrateg $%ns&er' The multidomestic strateg decentrali!es operating decisions to each countr toenhance local responsiveness$ The primar e7ample from the te7t"ook is :c*onaldBs$ Theglo"al strateg centrali!es operating decisions &ith head#uarters coordinating thestandardi!ation and learning "et&een facilities$ The te7t"ook names Te7as >nstruments andCaterpillar$ The international strateg uses e7ports and licenses to penetrate the glo"al markets$Students ma cite =ier ,ne 3orld :arket or an &ine store$ The transnational strateg e7ploitsthe economies of scale and learning as &ell as pressure for responsiveness " recogni!ing thatcore competence does not reside in ust the 8home8 countr "ut can e7ist an &here in theorgani!ation$ 7amples from the te7t"ook include 0ertelsmann Reuters and Iestlé$*iff' 2Topic' +lo"al operations strateg options%%CS0' :ulticultural and *iversit," ective' ., -6

1-91Cop right ; <11 =earson ducation >nc$

Page 62: OM Exam 1 Chap 1-5

8/13/2019 OM Exam 1 Chap 1-5

http://slidepdf.com/reader/full/om-exam-1-chap-1-5 62/187

ssa

1) Iike is the &orldBs largest athletic "rand$ >ts innovative and "road product line helps drivesales ho&ever a large ma orit of those sales are in the foot&ear "usiness$ :ost of IikeBs goodsare produced overseas in lo&-cost factories and then imported to the final market$ Iike currentl

has man of the top ($S$ athletes under contract K:ichael Jordon Tiger 3oods *&a ne 3ade) "ut international sales are still small in emerging markets$ Ho&ever man competitors haveattempted to cop IikeBs "usiness model Khigh-value "randed products manufactured at lo&-cost) including %didas and Ree"ok &hile man retailers have attempted to pass on the lo&-cost

pressure of retail consumers$ =erform a S3,T anal sis for Iike$%ns&er'Strengths- >nnovative products athletes under contract3eakness- :uch of revenue is from foot&ear eroding market share could cost Iike its

profita"ilit,pportunities- Sales can "e increased in emerging markets using &ell-kno&n athletes "road

product line can "e e7panded into high profit sectors K e&elr sunglasses golf etc)

Threats- >nternational "usiness makes Iike vulnera"le to currenc changes lo&-cost pressurefrom retailers can decrease profit per item competition could erode e7isting market share$%thletes personal lives could &eaken IikeBs reputation$*iff' Topic' Strateg development and implementation," ective' ., -5

) Star"ucks is one of the "est kno&n coffeehouse chains in the &orld$ ach store sells a varietof innovative products to complement the arra of coffee choices availa"le$ Ho&ever ?6 ofcurrent stores are located in the (nited States and the e7pensive nature of the coffee leavesStar"ucks vulnera"le to changes in consumer spending "ehavior Ksuch as recessions)$ RecentlStar"ucks has "egun initiatives to sell its specialt coffee "eans for home use presenting achance for a large increase in revenue and diversification$ Ho&ever Star"ucks faces fiercecompetition seeking a piece of its lucrative market share and the threat of consumer "ehaviorchanges given its reputation rides on a singular product$ =erform a S3,T anal sis forStar"ucks$%ns&er'Strengths- High profit specialt coffee &ell kno&n "rand image3eakness- :ost stores located in ($S$ most profits come from coffee K"oth are lack of diversification),pportunities- merging market in coffee "eans for home use chance for glo"al e7pansionThreats- >ntense competition consumer "ehavior changes Kless spending during a recession onits lu7ur coffee change in "everage preferences)*iff' Topic' Strateg development and implementation," ective' ., -5

1-9Cop right ; <11 =earson ducation >nc$

Page 63: OM Exam 1 Chap 1-5

8/13/2019 OM Exam 1 Chap 1-5

http://slidepdf.com/reader/full/om-exam-1-chap-1-5 63/187

2) =erform a S3,T anal sis of 0oeingBs ?@? *reamliner using the information presented &ithinthe te7t$%ns&er'Strengths- Technological advances K@ increase in fuel efficienc electronic maintenancemonitoring)

3eaknesses- *iverse suppliers and assem"l locations leave 0oeing vulnera"le to currence7change rates and make #ualit control difficult,pportunities- one of fastest-selling commercial ets ever gives 0oeing a chance to increasemarket share 0oeing can parla its use of diverse supplier locations into diversif ing customer

"aseThreats- competition from %ir"us divestment of the manufacture process risks losing compantrade secrets Kcompetitive advantage)*iff' Topic' Strateg development and implementation," ective' ., -5

5) =erform a S3,T anal sis of Hard Rock Café using the information presented &ithinChapters 1 and of the te7t$%ns&er'Strengths- Hard RockBs uni#ue coupling of an e7perience &ith its menu gives it a competitiveadvantage diverse portfolio of stores in 5< countries lo&ers risk 3eaknesses-Hard Rock is kno&n e7clusivel for its 8e7perience8 this one-dimensionalcompetitive advantage presents high risk ,pportunities- Ie& 8e7perience8 t pes at restaurants K0lues Hip-Hop etc themes) addition ofne& stores to the e7isting 1 <Q storesThreats- Hard RockBs e7clusive Rock BnB Roll theme leaves it vulnera"le to changes in consumermusic preferences competition in the restaurant "usiness is ver tough*iff' Topic' Strateg development and implementation," ective' ., -5

6) =erform a S3,T anal sis for Fran! Colru t using the information presented &ithin the te7t$%ns&er'Strengths- High profit margin of 9$6 from its e7treme lo&-cost competitive advantage3eaknesses- .ack of "agging in-store music dim lighting and lack of aesthetic in-store appealall limit potential customer "ase$,pportunities- Fran! Colru t onl operates 19< stores so there is opportunit for glo"ale7pansionThreats- High profit margin might encourage competition to cop Fran! Colru tBs lo&-costcompetitive advantage eroding it$ Changes in consumer "ehavior Kincreased prosperit ) mightdecrease customers interested in onl a lo&-cost alternative$*iff' Topic' Strateg development and implementation," ective' ., -5

1-92Cop right ; <11 =earson ducation >nc$

Page 64: OM Exam 1 Chap 1-5

8/13/2019 OM Exam 1 Chap 1-5

http://slidepdf.com/reader/full/om-exam-1-chap-1-5 64/187

Operations Management, 10e (Heizer/Render)Chapter * Pro+ect %ana$e"ent

True/False

1) The S of an activit that has onl one predecessor is simpl the F of that predecessor$%ns&er' TR(*iff' Topic' *etermining the pro ect schedule," ective' .,2-2

) ,ne phase of a large pro ect is scheduling$%ns&er' TR(*iff' 1Topic' The importance of pro ect management," ective' no .,

2) % pro ect organi!ation &orks "est for an organi!ation &hen the pro ect resides in onl one ofits functional areas$%ns&er' F%.S*iff' Topic' =ro ect planning," ective' no .,

5) 0 their ver nature pro ects have a limited lifetime and that sets pro ect management apartfrom the management of more traditional activities$%ns&er' TR(*iff' Topic' =ro ect planning," ective' no .,

6) ,ne responsi"ilit of a pro ect manager is to make sure that the pro ect meets its #ualit goals$%ns&er' TR(*iff' Topic' =ro ect planning," ective' no .,

9) 3ork 0reakdo&n Structure is a useful tool in pro ect management "ecause it addresses thetiming of individual &ork elements$%ns&er' F%.S*iff' Topic' =ro ect planning," ective' no .,

1-95Cop right ; <11 =earson ducation >nc$

Page 65: OM Exam 1 Chap 1-5

8/13/2019 OM Exam 1 Chap 1-5

http://slidepdf.com/reader/full/om-exam-1-chap-1-5 65/187

?) =ro ect managers have their o&n code of ethics esta"lished " the =ro ect :anagement>nstitute$%ns&er' TR(*iff' Topic' thical issues faced in pro ect management

%%CS0' thical Reasoning," ective' no .,

@) thical issues &hich can arise in pro ects include gifts from contractors pressure to maskdela s &ith false status reports and pressure to compromise pro ect #ualit for cost or timesavings$%ns&er' TR(*iff' 1Topic' thical issues faced in pro ect management%%CS0' thical Reasoning," ective' no .,

A) +antt charts give a timeline for each of a pro ect s activities "ut the do not ade#uatelillustrate the interrelationships "et&een the activities and the resources$%ns&er' TR(*iff' Topic' =ro ect scheduling," ective' .,2-1

1<) = RT "ut not C=: has the a"ilit to consider the precedence relationships in a pro ect$%ns&er' F%.S*iff' Topic' =ro ect scheduling," ective' .,2-6

11) The shortest of all paths through the net&ork is the critical path$%ns&er' F%.S*iff' Topic' =ro ect management techni#ues' = RT and C=:," ective' .,2-5

1 ) The fundamental difference "et&een = RT and C=: is that = RT uses the "eta distri"utionfor crashing pro ects &hile C=: uses cost estimates$%ns&er' F%.S*iff' Topic' =ro ect management techni#ues' = RT and C=:," ective' .,2-6

1-96Cop right ; <11 =earson ducation >nc$

Page 66: OM Exam 1 Chap 1-5

8/13/2019 OM Exam 1 Chap 1-5

http://slidepdf.com/reader/full/om-exam-1-chap-1-5 66/187

12) Slack is the amount of time an activit can "e dela ed &ithout dela ing the entire pro ect$%ns&er' TR(*iff' Topic' =ro ect management techni#ues' = RT and C=:," ective' .,2-2

15) ver net&ork has at least one critical path$%ns&er' TR(*iff' Topic' =ro ect management techni#ues' = RT and C=:," ective' .,2-5

16) The critical path can "e determined " use of either the 8for&ard pass8 or the 8"ack&ard pass$8%ns&er' F%.S*iff'

Topic' =ro ect management techni#ues' = RT and C=:," ective' .,2-5

19) The = RT pessimistic time estimate is an estimate of the minimum time an activit &illre#uire$%ns&er' F%.S*iff' 1Topic' =ro ect management techni#ues' = RT and C=:," ective' .,2-6

1?) The standard deviation of pro ect duration is the average of the standard deviation of allactivities on the critical path$%ns&er' F%.S*iff' Topic' =ro ect management techni#ues' = RT and C=:," ective' .,2-6

1@) >n = RT anal sis the identification of the critical path can "e incorrect if a noncriticalactivit takes su"stantiall more than its e7pected time$%ns&er' TR(*iff' 2Topic' =ro ect management techni#ues' = RT and C=:," ective' .,2-6

1A) Shortening the pro ectBs duration " deleting unnecessar activities is called 8pro ectcrashing$8%ns&er' F%.S*iff' Topic' Cost-time trade-offs and pro ect crashing," ective' .,2-9

<) >n pro ect management crashing an activit must consider the impact on all paths in the1-99

Cop right ; <11 =earson ducation >nc$

Page 67: OM Exam 1 Chap 1-5

8/13/2019 OM Exam 1 Chap 1-5

http://slidepdf.com/reader/full/om-exam-1-chap-1-5 67/187

net&ork$%ns&er' TR(*iff' Topic' Cost-time trade-offs and pro ect crashing," ective' .,2-9

:ultiple Choice

1) 3hich of the follo&ing statements regarding 0echtel is true4%) >ts competitive advantage is pro ect management$0) 0echtel &as the construction contractor for the Hoover *am$C) 3hile helping to re"uild >ra# 0echtelBs cre&s had to travel under armed escort$*) 0echtelBs procurement program is glo"al in nature$) %ll of the a"ove are true$%ns&er' *iff' 1

Topic' +lo"al compan profile," ective' no .,

) 3hich of the follo&ing statements a"out 0echtel is true4%) ven though 0echtel is over 1<< ears old the u&aiti oil fields &as its first 8pro ect$80) 0echtel is the &orldBs premier manager of massive construction and engineering pro ects$C) 0echtelBs competitive advantage is suppl -chain management$*) 3hile its pro ects are &orld&ide its net&ork of suppliers is largel in the ($S$) %ll of the a"ove are true$%ns&er' 0*iff' Topic' +lo"al compan profile," ective' no .,

2) 3hich of the follo&ing pro ects &ere completed " 0echtel4%) Hoover *am0) 0oston Central %rter /TunnelC) Re"uilding u&ait after >ra#Bs invasion in 1AA<*) Re"uilding >ra# after the ($S$ invasion starting in <<2) %ll of the a"ove &ere 0echtel pro ects%ns&er' *iff' Topic' +lo"al compan profile," ective' no .,

1-9?Cop right ; <11 =earson ducation >nc$

Page 68: OM Exam 1 Chap 1-5

8/13/2019 OM Exam 1 Chap 1-5

http://slidepdf.com/reader/full/om-exam-1-chap-1-5 68/187

Page 69: OM Exam 1 Chap 1-5

8/13/2019 OM Exam 1 Chap 1-5

http://slidepdf.com/reader/full/om-exam-1-chap-1-5 69/187

@) % code of ethics especiall for pro ect managers%) has "een esta"lished " the =ro ect :anagement >nstitute0) has "een formulated " the Federal governmentC) has "een formulated " the 3orld Trade ,rgani!ation*) is inappropriate since ever one should use the same guidance on ethical issues

) does not e7ist at this time%ns&er' %*iff' Topic' =ro ect planning%%CS0' thical Reasoning," ective' no .,

A) thical issues that ma arise in pro ects large and small include%) gifts from contractors0) e7aggerated e7pense reportsC) compromised safet or health standards to save time or reduce costs

*) pressure to mask dela s &ith false status reports) all of the a"ove%ns&er' *iff' 1Topic' =ro ect planning%%CS0' thical Reasoning," ective' no .,

1<) The pro ect organi!ation &orks "est &hen &hich of the follo&ing conditions are satisfied4>$ 3ork can "e defined &ith a specific goal and deadline$>>$ The o" is t pical and familiar to the e7isting organi!ation$>>>$ The &ork contains interrelated tasks re#uiring speciali!ed skills>E$ The pro ect is temporar "ut unimportant to long-term organi!ational successE$ The pro ect cuts across organi!ational lines$%) > >> >>> >E E0) Ione of the a"ove conditions need to "e satisfiedC) > >>> E*) > >> >>> E) > >>> >E E%ns&er' C*iff' Topic' =ro ect planning," ective' no .,

1-9ACop right ; <11 =earson ducation >nc$

Page 70: OM Exam 1 Chap 1-5

8/13/2019 OM Exam 1 Chap 1-5

http://slidepdf.com/reader/full/om-exam-1-chap-1-5 70/187

Page 71: OM Exam 1 Chap 1-5

8/13/2019 OM Exam 1 Chap 1-5

http://slidepdf.com/reader/full/om-exam-1-chap-1-5 71/187

Page 72: OM Exam 1 Chap 1-5

8/13/2019 OM Exam 1 Chap 1-5

http://slidepdf.com/reader/full/om-exam-1-chap-1-5 72/187

1@) The difference "et&een %,I and %,% net&orks is that%) %,% net&orks sometimes re#uire dumm activities0) nodes consume no resources or time in %,% net&orksC) "oth are accepta"le in practice ho&ever :icrosoft =ro ect uses %,I*) in %,I nodes designate activities in %,% arro&s designate activities

) %ll of the a"ove are true$%ns&er' *iff' Topic' =ro ect management techni#ues' = RT and C=:," ective' .,2-

1A) 3hich of the follo&ing statements regarding critical paths is true4%) The shortest of all paths through the net&ork is the critical path$0) Some activities on the critical path ma have slack$C) ver net&ork has onl one critical path$*) ,n a specific pro ect there can "e multiple critical paths all &ith e7actl the same duration$

) The duration of the critical path is the average duration of all paths in the pro ect net&ork$%ns&er' **iff' Topic' =ro ect management techni#ues' = RT and C=:," ective' .,2-5

<) 3hich of the follo&ing statements regarding C=: is true4%) The critical path is the shortest of all paths through the net&ork$0) The critical path is that set of activities that has positive slack$C) Some net&orks have no critical path$*) %ll activities on the critical path have their .S e#ual to the ma7imum F of all immediate

predecessors$) %ll of the a"ove are false$%ns&er' **iff' Topic' =ro ect management techni#ues' = RT and C=:," ective' .,2-5

1) % simple C=: net&ork has three activities % 0 and C$ % is an immediate predecessor of 0and of C$ 0 is an immediate predecessor of C$ The activit durations are %O5 0O2 CO@$%) The critical path is %-0-C duration 16$0) The critical path is %-C duration 1 $C) The critical path is %-0-C duration 12$6*) The critical path cannot "e determined &ithout kno&ing = RT e7pected activit times$) The net&ork has no critical path$%ns&er' %*iff' 2Topic' =ro ect management techni#ues' = RT and C=:%%CS0' %nal tic Skills," ective' .,2-5

) % simple C=: net&ork has three activities * and F$ * is an immediate predecessor of 1-?

Cop right ; <11 =earson ducation >nc$

Page 73: OM Exam 1 Chap 1-5

8/13/2019 OM Exam 1 Chap 1-5

http://slidepdf.com/reader/full/om-exam-1-chap-1-5 73/187

and of F$ is an immediate predecessor of F$ The activit durations are *O5 O2 FO@$%) The critical path is *- -F duration 16$0) The critical path is *-F duration 1 $C) Slack at * is 2 units$*) Slack at is 2 units$

) 0oth % and C are true$%ns&er' %*iff' 2Topic' =ro ect management techni#ues' = RT and C=:%%CS0' %nal tic Skills," ective' .,2-5

2) % simple C=: net&ork has five activities % 0 C * and $ % is an immediate predecessorof C and of *$ 0 is also an immediate predecessor of C and of *$ C and * are "oth immediate

predecessors of $%) There are t&o paths in this net&ork$

0) There are four paths in this net&ork$C) There are five paths in this net&ork$*) There are 6 paths through this net&ork$) Ione of these statements is true$%ns&er' 0*iff' Topic' =ro ect management techni#ues' = RT and C=:%%CS0' %nal tic Skills," ective' .,2-2

5) %ctivit * on a C=: net&ork has predecessors 0 and C and has successor F$ * has duration9$ 0Bs earliest finish is 1@ &hile CBs is <$ FBs late start is 9$ 3hich of the follo&ing is definiteltrue4%) 0 is a critical activit $0) C is completed "efore 0$C) * has no slack "ut is not critical$*) * is critical and has !ero slack$) F is critical and has !ero slack$%ns&er' **iff' 2Topic' =ro ect management techni#ues' = RT and C=:%%CS0' %nal tic Skills," ective' .,2-5

1-?2Cop right ; <11 =earson ducation >nc$

Page 74: OM Exam 1 Chap 1-5

8/13/2019 OM Exam 1 Chap 1-5

http://slidepdf.com/reader/full/om-exam-1-chap-1-5 74/187

Page 75: OM Exam 1 Chap 1-5

8/13/2019 OM Exam 1 Chap 1-5

http://slidepdf.com/reader/full/om-exam-1-chap-1-5 75/187

$

%ctivit *uration>mmediate

=redecessors% 5 ---

0 %C ? --* 5 %

6 0 C *

%) %-0-*G 1<0) %-0- G 11C) C- G 1*) %-*- G 12) %-0-C-*- G %ns&er' *

*iff' 2Topic' =ro ect management techni#ues' = RT and C=:%%CS0' %nal tic Skills," ective' .,2-5

2<) The critical path for the net&ork activities sho&n "elo& is &ith duration $

%ctivit *uration>mmediate

=redecessors% --

0 5 --C 9 % 0* 1 % 0

0 C *

%) %-*- G 60) 0- G 9C) 0-*- G ?*) %-C- G 1<) 0-C- G 1%ns&er'

*iff' 2Topic' =ro ect management techni#ues' = RT and C=:%%CS0' %nal tic Skills," ective' .,2-5

1-?6Cop right ; <11 =earson ducation >nc$

Page 76: OM Exam 1 Chap 1-5

8/13/2019 OM Exam 1 Chap 1-5

http://slidepdf.com/reader/full/om-exam-1-chap-1-5 76/187

21) The distri"ution is used " = RT anal sis to calculate e7pected activit timesand variances$%) Iormal0) 0etaC) %lpha

*) +aussian) 0inomial%ns&er' 0*iff' 1Topic' =ro ect management techni#ues' = RT and C=:," ective' .,2-6

2 ) The e7pected activit time in = RT anal sis is calculated as%) the simple average of the optimistic pessimistic and most likel times0) the &eighted average of a m and " &ith m &eighted 5 times as heavil as a and "C) the sum of the optimistic pessimistic and most likel times

*) the sum of the optimistic pessimistic and most likel times divided " si7) the sum of the activit variances divided " si7%ns&er' 0*iff' Topic' =ro ect management techni#ues' = RT and C=:," ective' .,2-6

22) The critical path for the net&ork activities sho&n "elo& is &ith duration $

%ctivit *uration>mmediate

=redecessors% 1< ---0 @ ---C %* 5 %

6 0 C *

%) %-CG 10) %-*- G 1AC) 0- G 12*) %-0-C-*- G A

) none of the a"ove%ns&er' 0*iff' 2Topic' =ro ect management techni#ues' = RT and C=:%%CS0' %nal tic Skills," ective' .,2-5

1-?9Cop right ; <11 =earson ducation >nc$

Page 77: OM Exam 1 Chap 1-5

8/13/2019 OM Exam 1 Chap 1-5

http://slidepdf.com/reader/full/om-exam-1-chap-1-5 77/187

Page 78: OM Exam 1 Chap 1-5

8/13/2019 OM Exam 1 Chap 1-5

http://slidepdf.com/reader/full/om-exam-1-chap-1-5 78/187

Page 79: OM Exam 1 Chap 1-5

8/13/2019 OM Exam 1 Chap 1-5

http://slidepdf.com/reader/full/om-exam-1-chap-1-5 79/187

51) %n activit on a = RT net&ork has these time estimates' optimistic O most likel O 6 and pessimistic O 1<$ >ts e7pected time is%) 6$<<0) 6$22C) 6$9?

*) 1?$<<) none of these%ns&er' 0*iff' Topic' =ro ect management techni#ues' = RT and C=:%%CS0' %nal tic Skills," ective' .,2-6

5 ) %n activit on a = RT net&ork has these time estimates' optimistic O 1 most likel O and pessimistic O 6$ >ts e7pected time is%) $<<

0) $22C) $9?*) @$<<) none of these%ns&er' 0*iff' Topic' =ro ect management techni#ues' = RT and C=:%%CS0' %nal tic Skills," ective' .,2-6

52) %n activit on a = RT net&ork has these time estimates' optimistic O most likel O 2 and pessimistic O @$ >ts e7pected time and variance Kif it is a critical activit ) are%) 2$9?G 10) 2$9?G 9C) 5$22G 1*) 5$22G 9) none of these%ns&er' %*iff' Topic' =ro ect management techni#ues' = RT and C=:%%CS0' %nal tic Skills," ective' .,2-6

1-?ACop right ; <11 =earson ducation >nc$

Page 80: OM Exam 1 Chap 1-5

8/13/2019 OM Exam 1 Chap 1-5

http://slidepdf.com/reader/full/om-exam-1-chap-1-5 80/187

Page 81: OM Exam 1 Chap 1-5

8/13/2019 OM Exam 1 Chap 1-5

http://slidepdf.com/reader/full/om-exam-1-chap-1-5 81/187

5?) % contractorBs pro ect "eing anal !ed " = RT has an estimated time for the critical path of1 < da s$ The sum of all activit variances is @1G the sum of variances along the critical path is95$ The pro"a"ilit that the pro ect &ill take 12< or more da s to complete is%) <$1<6?0) <$1226

C) <$@61*) <$@A52) 1$ A%ns&er' %*iff' Topic' =ro ect management techni#ues' = RT and C=:%%CS0' %nal tic Skills," ective' .,2-6

5@) %nal sis of a = RT pro"lem sho&s the estimated time for the critical path to "e 1<@ da s&ith a variance of 95$ There is a $A< pro"a"ilit that the pro ect &ill "e completed "efore

appro7imatel da $%) A@0) 1<@C) 1<A*) 116) 11@%ns&er' *iff' 2Topic' =ro ect management techni#ues' = RT and C=:%%CS0' %nal tic Skills," ective' .,2-6

5A) % pro ect &hose critical path has an estimated time of 1 < da s &ith a variance of 1<< has a< chance that the pro ect &ill "e completed "efore da Krounded to nearest da )$%) A@0) 11C) 1 <*) 1 5) <%ns&er' 0*iff' 2Topic' =ro ect management techni#ues' = RT and C=:%%CS0' %nal tic Skills," ective' .,2-6

1-@1Cop right ; <11 =earson ducation >nc$

Page 82: OM Exam 1 Chap 1-5

8/13/2019 OM Exam 1 Chap 1-5

http://slidepdf.com/reader/full/om-exam-1-chap-1-5 82/187

6<) % pro ect &hose critical path has an estimated time of @ < da s &ith a variance of 6 has a< chance that the pro ect &ill "e completed "efore da Krounded to nearest da )$%) 9210) 9@AC) @<?

*) @22) 1<<A%ns&er' C*iff' 2Topic' =ro ect management techni#ues' = RT and C=:%%CS0' %nal tic Skills," ective' .,2-6

61) Contract re#uirements state that a pro ect must "e completed &ithin 1@< &orking da s or it&ill incur penalties for late completion$ %nal sis of the activit net&ork reveals an estimated

pro ect time of 156 &orking da s &ith a pro ect variance of 5<<$ 3hat is the pro"a"ilit that the pro ect &ill "e completed "efore the late-pa ment deadline4%) <$<5<10) <$56AAC) <$@<69*) <$A6AA) near 1$<<<< or almost certain%ns&er' **iff' 2Topic' =ro ect management techni#ues' = RT and C=:%%CS0' %nal tic Skills," ective' .,2-6

6 ) 3hich of these statements regarding time-cost tradeoffs in C=: net&orks is true4%) Crashing is not possi"le unless there are multiple critical paths$0) Crashing a pro ect often reduces the length of long-duration "ut noncritical activities$C) %ctivities not on the critical path can never "e on the critical path even after crashing$*) Crashing shortens the pro ect duration " assigning more resources to one or more of thecritical tasks$) Ione of the a"ove is true$%ns&er' **iff' Topic' Cost-time trade-offs and pro ect crashing," ective' .,2-9

1-@Cop right ; <11 =earson ducation >nc$

Page 83: OM Exam 1 Chap 1-5

8/13/2019 OM Exam 1 Chap 1-5

http://slidepdf.com/reader/full/om-exam-1-chap-1-5 83/187

Page 84: OM Exam 1 Chap 1-5

8/13/2019 OM Exam 1 Chap 1-5

http://slidepdf.com/reader/full/om-exam-1-chap-1-5 84/187

Page 85: OM Exam 1 Chap 1-5

8/13/2019 OM Exam 1 Chap 1-5

http://slidepdf.com/reader/full/om-exam-1-chap-1-5 85/187

6A) T&o critical path activities are candidates for crashing on a C=: net&ork$ %ctivit detailsare in the ta"le "elo&$ To cut one da from the pro ect s duration activit should "ecrashed first adding to pro ect cost$

%ctivit Iormal Time Iormal Cost Crash Time Crash Cost

0 5 da s L9 <<< 2 da s L@ <<<C 9 da s L5 <<< 5 da s L9 <<<

%) 0G L <<<0) 0G L@ <<<C) CG L1 <<<*) CG L <<<) CG L9 <<<%ns&er' C*iff' Topic' Cost-time trade-offs and pro ect crashing

%%CS0' %nal tic Skills," ective' .,2-9

Short %ns&er

1) is an organi!ation formed to ensure that programs Kpro ects) receive the propermanagement and attention$%ns&er' =ro ect organi!ation*iff' 1Topic' =ro ect planning," ective' no .,

) divides a pro ect into more and more detailed components$%ns&er' 3ork "reakdo&n structure or 30S*iff' Topic' =ro ect planning," ective' no .,

2) The has esta"lished a code of ethics especiall for pro ect managers$%ns&er' =ro ect :anagement >nstitute*iff' 1Topic' =ro ect planning

%%CS0' thical Reasoning," ective' no .,

5) is a net&ork techni#ue using onl one time factor per activit that ena"lesmanagers to schedule monitor and control large and comple7 pro ects$%ns&er' Critical path method or C=:*iff' 1Topic' =ro ect management techni#ues' = RT and C=:," ective' .,2-5

1-@6Cop right ; <11 =earson ducation >nc$

Page 86: OM Exam 1 Chap 1-5

8/13/2019 OM Exam 1 Chap 1-5

http://slidepdf.com/reader/full/om-exam-1-chap-1-5 86/187

6) % diagram of all activities and the precedence relationships that e7ist "et&een these activitiesin a pro ect is aKn) $%ns&er' net&ork *iff' Topic' =ro ect management techni#ues' = RT and C=:

," ective' .,2-

9) The is the computed longest time pathKs) through a net&ork$%ns&er' critical path*iff' 1Topic' =ro ect management techni#ues' = RT and C=:," ective' .,2-5

?) The net&ork anal sis method that allo&s activit times to var is $%ns&er' = RT or =rogram valuation and Revie& Techni#ue*iff'

Topic' =ro ect management techni#ues' = RT and C=:," ective' .,2-6

@) is the amount of time an individual activit in a net&ork can "e dela ed &ithoutdela ing the entire pro ect$%ns&er' Slack time*iff' 1Topic' =ro ect management techni#ues' = RT and C=:," ective' .,2-5

A) The distri"ution is appropriate for calculating e7pected activit times and activitvariances in = RT net&orks$%ns&er' 0eta*iff' 1Topic' =ro ect management techni#ues' = RT and C=:," ective' .,2-6

1<) is decreasing activit time in a net&ork to reduce time on the critical path sototal completion time is reduced$%ns&er' Crashing*iff' 1Topic' Cost-time trade-offs and pro ect crashing," ective' .,2-9

1-@9Cop right ; <11 =earson ducation >nc$

Page 87: OM Exam 1 Chap 1-5

8/13/2019 OM Exam 1 Chap 1-5

http://slidepdf.com/reader/full/om-exam-1-chap-1-5 87/187

11) 3hat are the three phases of a pro ect4 *escri"e each in a sentence or t&o$%ns&er' The three phases are planning scheduling and controlling$ =lanning includes goalsetting defining the pro ect and team organi!ation$ Scheduling relates people mone andsupplies to specific activities and relates activities to each other$ Controlling is &here the firmmonitors resources costs #ualit and "udgets$ >t also revises or changes plans and shifts

resources to meet time and cost demands$*iff' Topic' The importance of pro ect management," ective' no .,

1 ) >dentif the responsi"ilities of pro ect managers$%ns&er' =ro ect managers are directl responsi"le for making sure that K1) all necessaractivities are finished in proper se#uence and on timeG K ) the pro ect comes in &ithin "udgetG K2)the pro ect meets its #ualit goalsG and K5) the people assigned to the pro ect receive themotivation direction and information needed to do their o"s$*iff'

Topic' =ro ect planning," ective' no .,

12) 3hat is a pro ect organi!ation4%ns&er' % pro ect organi!ation is a form of management so that people and other resources are

pooled for a limited amount of time to complete a specific goal or pro ect$*iff' Topic' =ro ect planning," ective' no .,

15) *escri"e some of the challenges faced in the construction of the ne& 11-stor "uilding at%rnold =almer Hospital in ,rlando Florida$%ns&er' =rior to "eginning actual construction regulator and funding issues added as the do&ith most pro ects su"stantial time to the overall pro ect$ Cities have !oning and parking issuesthe =% has drainage and &aste issues and regulator authorities have their o&n re#uirementsas do issuers of "onds$*iff' Topic' =ro ect controlling," ective' no .,

1-@?Cop right ; <11 =earson ducation >nc$

Page 88: OM Exam 1 Chap 1-5

8/13/2019 OM Exam 1 Chap 1-5

http://slidepdf.com/reader/full/om-exam-1-chap-1-5 88/187

16) 3hat are some of the ethical issues faced " pro ect managers4 3hich of these are likel tooccur "efore a pro ect "egins &hich are likel to occur &hile the pro ect is under&a and &hichma occur after a pro ect is complete4 >llustrate an one of these from recent ne&s$%ns&er' % pro ect manager can "e e7posed to a &ide variet of ethical issues$ These include

"ut are not limited to gifts from contractors pressure to mask dela s &ith false status reports

falsif ing Ke7aggerating) e7pense reports and compromising #ualit &ith su"standard materialsor practices to save either time or mone $ Those &ho report on completed pro ects might not "etruthful a"out the pro ectBs success Kconsider the massive changes that took place at I%S% aftert&o shuttle disasters and note the ver recent failure of the F0>Bs Carnivore soft&are)*iff' Topic' =ro ect planning and pro ect scheduling%%CS0' thical Reasoning," ective' no .,

19) >dentif and descri"e "riefl each of the purposes of pro ect scheduling$%ns&er' =ro ect scheduling sho&s the relationship of each activit to others and to the &hole

pro ect$ >t identifies the precedence relationship among activities$ >t encourages the setting ofrealistic time and cost estimates for each activit $ >t helps make "etter use of people mone andmaterial resources " identif ing critical "ottlenecks in the pro ect$

*iff' Topic' =ro ect scheduling," ective' no .,

1?) 3hat is the o" ective of critical path anal sis4%ns&er' Critical path anal sis determines the longest path through a net&ork of activities$ Thislongest path is the ke to making the schedule that provides for completing all activities in theshortest time$ Critical path anal sis identifies those activities critical to timel completion of allactivities so the can receive management focus$*iff' Topic' =ro ect management techni#ues' = RT and C=:," ective' .,2-5

1@) 7plain &h the critical path is the longest not the shortest path through a net&ork$%ns&er' Critical path is that set of activities in a pro ect net&ork that controls the duration of theentire pro ect$ The controlling element to completion of all activities is the longest pathG anshorter path &ill not allo& for all activities to "e completed$*iff' Topic' =ro ect management techni#ues' = RT and C=:," ective' .,2-5

1A) *efine slack$%ns&er' Slack is the amount of time an activit can "e dela ed &ithout dela ing the entire

pro ect assuming its preceding activities are completed as earl as possi"le$*iff' Topic' =ro ect management techni#ues' = RT and C=:," ective' .,2-5<) >dentif in order the si7 steps "asic to "oth = RT and C=:$

1-@@Cop right ; <11 =earson ducation >nc$

Page 89: OM Exam 1 Chap 1-5

8/13/2019 OM Exam 1 Chap 1-5

http://slidepdf.com/reader/full/om-exam-1-chap-1-5 89/187

Page 90: OM Exam 1 Chap 1-5

8/13/2019 OM Exam 1 Chap 1-5

http://slidepdf.com/reader/full/om-exam-1-chap-1-5 90/187

%ns&er' The earliest start time is the earliest time at &hich an activit ma start and still satisfall precedence re#uirements$ The latest start time is the latest time at &hich an activit ma startand still satisf "oth precedence re#uirements and the overall pro ect completion time$*iff' Topic' *etermining the pro ect schedule

," ective' no .,

9) Ho& is the e7pected completion time of a pro ect activit and of a = RT pro ect computed4%ns&er' The e7pected completion time of a pro ect activit uses the 0eta distri"utionG e7pectedtime is the &eighted average of optimistic most likel and pessimistic time estimates$ 7pectedcompletion of a = RT pro ect is the sum of the e7pected times for individual activities that areon the critical path$*iff' Topic' *etermining the pro ect schedule," ective' .,2-6

?) *escri"e in &ords ho& to calculate a pro ectBs standard deviation$ 3hat assumption allo&sthat calculation to "e accurate4%ns&er' %dd the variances of the activities on the critical path and then take the s#uare root$3e can do this "ecause &e assume that the activities are independent$*iff' 2Topic' Earia"ilit in activit times," ective' .,2-6

@) 0riefl descri"e the concept of cost/time trade-off and ho& it is used$%ns&er' Cost/time trade-off is fundamentall = RT &ith additional information provided thatena"les one to monitor and control pro ect cost and to stud possi"le cost/time trade-offs$ Thiscan "e done " making a "udget for the entire pro ect using the activit cost estimates and "monitoring the "udget as the pro ect takes place$ (sing this approach &e can determine thee7tent to &hich a pro ect is incurring a cost overrun or a cost underrun$ >n addition &e can usethe same techni#ue to determine the e7tent to &hich a pro ect is ahead of schedule or "ehindschedule$*iff' Topic' Cost-time trade-offs and pro ect crashing," ective' .,2-9

A) 3hat are the advantages of using = RT and C=:4%ns&er' The advantages include its usefulness for scheduling and controlling large pro ects itsstraightfor&ard concept its graphical displa s of relationships "et&een activities its critical pathand slack time anal sis its a"ilit to document processes its &ide range of applica"ilit and itsusefulness in monitoring schedules and costs$*iff' Topic' % criti#ue of = RT and C=:," ective' .,2-6

ssa

1) Consider the net&ork pictured "elo&$1-A<

Cop right ; <11 =earson ducation >nc$

Page 91: OM Exam 1 Chap 1-5

8/13/2019 OM Exam 1 Chap 1-5

http://slidepdf.com/reader/full/om-exam-1-chap-1-5 91/187

a$ numerate all paths through this net&ork$ "$ Calculate the critical path for the net&ork$c$ 3hat is the minimum duration of the pro ect4d$ Ho& much slack e7ists at each activit 4

%ns&er' Ka) =ossi"le paths are =-S Klength ) =-R-T Klength @) and D-T Klength 1)$ K") Thelongest of these =-R-T is the critical path at Kc) @ time units$ Kd) There is no slack at = R or Tsince these are critical tasks$ S has 9 units slack since the path it is on totals onl unitscompared to the critical path length of @$ D has ? units of slack since it is on a 1 length path ?less than the ma7imum$*iff' Topic' =ro ect management techni#ues' = RT and C=:%%CS0' %nal tic Skills," ective' .,2-5

1-A1Cop right ; <11 =earson ducation >nc$

Page 92: OM Exam 1 Chap 1-5

8/13/2019 OM Exam 1 Chap 1-5

http://slidepdf.com/reader/full/om-exam-1-chap-1-5 92/187

) % net&ork consists of the activities in the follo&ing list$ Times are given in &eeks$

%ctivit =receding Time% -- @0 -- 2

C % ?* % 0 2

C 5

a$ *ra& the net&ork diagram$ "$ Calculate the S F .S .F and Slack for each activit $c$ 3hat is pro ect completion time4%ns&er'Ka)

1-ACop right ; <11 =earson ducation >nc$

Page 93: OM Exam 1 Chap 1-5

8/13/2019 OM Exam 1 Chap 1-5

http://slidepdf.com/reader/full/om-exam-1-chap-1-5 93/187

K" c)

Results

Task arl Startarl

Finish .ate Start.ate

Finish Slack

% < @ < @ <0 < 2 ? 1< ?C @ 16 @ 16 <* @ 11 1< 12

16 1A 16 1A <F 11 1? 12 1A

=ro ect 1A*iff' Topic' =ro ect management techni#ues' = RT and C=:%%CS0' %nal tic Skills," ective' .,2-2

1-A2Cop right ; <11 =earson ducation >nc$

Page 94: OM Exam 1 Chap 1-5

8/13/2019 OM Exam 1 Chap 1-5

http://slidepdf.com/reader/full/om-exam-1-chap-1-5 94/187

2) The net&ork "elo& represents a pro ect "eing anal !ed " Critical =ath :ethods$ %ctivitdurations are %O6 0O CO1 *O2 O6 FO1 +O? HO >O1< and JO9$

a$ 3hat task must "e on the critical path regardless of activit durations4

"$ 3hat is the duration of path %-0- -H-J4c$ 3hat is the critical path of this net&ork4d$ 3hat is the length of the critical path4e$ 3hat is slack time at activit H4f$ 3hat is the .ate Finish of activit H4g$ >f activit C &ere dela ed " t&o time units &hat &ould happen to the pro ect duration4

1-A5Cop right ; <11 =earson ducation >nc$

Page 95: OM Exam 1 Chap 1-5

8/13/2019 OM Exam 1 Chap 1-5

http://slidepdf.com/reader/full/om-exam-1-chap-1-5 95/187

%ns&er'Ka) JG K") <G Kc) %-0-+->-JG Kd) 2<G Ke) 6G Kf) 5G Kg) no impact$

Results

Task arl Start

arl

Finish .ate Start

.ate

Finish Slack % < 6 < 6 <0 6 ? 6 ? <C 6 1? 1< 6* < 2 1< 12 1<

? 1 1? 1<F 2 5 12 15 1<+ ? 15 ? 15 <H 1? 1A 5 6> 15 5 15 5 <J 5 2< 5 2< <

=ro ect 2<*iff' Topic' =ro ect management techni#ues' = RT and C=:%%CS0' %nal tic Skills," ective' .,2-5

1-A6Cop right ; <11 =earson ducation >nc$

Page 96: OM Exam 1 Chap 1-5

8/13/2019 OM Exam 1 Chap 1-5

http://slidepdf.com/reader/full/om-exam-1-chap-1-5 96/187

5) % net&ork consists of the follo&ing list$ Times are given in &eeks$

%ctivit =receding *uration% -- A0 %

C % 1* % 6

0 9F 0 @+ C F 2H *> H @J + > 9

J

a$ *ra& the net&ork diagram$ "$ 3hich activities form the critical path4c$ Ho& much slack e7ists at activities % and F4d$ 3hat is the duration of the critical path4%ns&er'Ka) Iet&ork diagram

K") paths %-*-H->-J- and %-C-+-J- are criticalKc) % has no slackG F has unitsKd) 2 &eeks*iff' Topic' =ro ect management techni#ues' = RT and C=:

%%CS0' %nal tic Skills," ective' .,2-5

1-A9Cop right ; <11 =earson ducation >nc$

Page 97: OM Exam 1 Chap 1-5

8/13/2019 OM Exam 1 Chap 1-5

http://slidepdf.com/reader/full/om-exam-1-chap-1-5 97/187

6) % partiall solved = RT pro"lem is detailed in the ta"le "elo&$ Times are given in &eeks$

%ctivit =receding,ptimistic

Time=ro"a"le

Time=essimistic

Time7pected

Time Eariance% -- ? A 15 1$291

0 % @ <C % @ 1 19 <* % 2 6 1< 1$291

0 5 9 @ <F 0 9 @ 1< <+ C F 2 5 <H * @ 1$<<<> H 9 @ 19 $??@J + > 5 9 15 $??@

J 6 <$ 6<

a$ Calculate the e7pected time for each activit $ nter these values in the appropriatecolumn in the ta"le a"ove$

"$ 3hich activities form the critical path4c$ 3hat is the estimated time of the critical path4d$ 3hat are the pro ect variance and the pro ect standard deviation4e$ 3hat is the pro"a"ilit of completion of the pro ect after &eek 5<4%ns&er' Ka) %OA$6 0O2 CO1 *O6$6 O9 FO@ +O2 HO2 >OA JO? O $6K") %-*-H->-J- G Kc) 29$6G Kd) A$62 2$<AG Ke) <$12$*iff' Topic' =ro ect management techni#ues' = RT and C=:%%CS0' %nal tic Skills," ective' .,2-5

1-A?Cop right ; <11 =earson ducation >nc$

Page 98: OM Exam 1 Chap 1-5

8/13/2019 OM Exam 1 Chap 1-5

http://slidepdf.com/reader/full/om-exam-1-chap-1-5 98/187

9) Consider the net&ork descri"ed in the ta"le "elo&$

%ctivit>mmediate

=redecessorKs) =essimistic =ro"a"le ,ptimisticJ -- 16 1< @

-- A @ ?. J 1< 9 6: J 2 2 2 I : A 6 1, : 1< ? 5= . I 1< @ 2

a$ Calculate the e7pected duration of each activit $ "$ Calculate the e7pected duration and variance of the critical path$c$ Calculate the pro"a"ilit that the pro ect &ill "e completed in fe&er than 2< time units$

1-A@Cop right ; <11 =earson ducation >nc$

Page 99: OM Exam 1 Chap 1-5

8/13/2019 OM Exam 1 Chap 1-5

http://slidepdf.com/reader/full/om-exam-1-chap-1-5 99/187

%ns&er'Ka) See ta"le "elo&$K") Tasks J-:-I-= are critical$ The sum of their e7pected durations is 9$<<G the sum of theirvariances is 5$6<$Kc) The standard deviation along the path is O $1 G the pro"a"ilit that *uration U 2< is the

pro"a"ilit that ! U K2< - 9$<<)/ $1 O 1$@A$ The associated normal curve area is <$A?<9 $

Task arlStart

arlFinish

.ateStart

.ateFinish Slack :ean Eariance

J < 1<$6 < 1<$6 < 1<$6 1$291111< @ 6$6 12$6 6$6 @

. 1<$6 1? 1A$6 9 A 9$6: 1<$6 12$6 1<$6 12$6 < 2 <

I 12$6 1@$6 12$6 1@$6 < 6 1$?????@, 12$6 <$6 1A 9 6$6 ?= 1@$6 9 1@$6 9 < ?$6 1$291111

=ro ect 9 =ro ect 5$6Std$dev $1 12

*iff' Topic' =ro ect management techni#ues' = RT and C=:%%CS0' %nal tic Skills," ective' .,2-5

1-AACop right ; <11 =earson ducation >nc$

Page 100: OM Exam 1 Chap 1-5

8/13/2019 OM Exam 1 Chap 1-5

http://slidepdf.com/reader/full/om-exam-1-chap-1-5 100/187

?) The net&ork "elo& represents a pro ect "eing anal !ed " Critical =ath :ethods$ %ctivitdurations are indicated on the net&ork$

a$ >dentif the activities on the critical path$ "$ 3hat is the duration of the critical path4c$ Calculate the amount of slack time at activit H$d$ >f activit > &ere dela ed " ten time units &hat &ould "e the impact on the pro ect duration4

1-1<<Cop right ; <11 =earson ducation >nc$

Page 101: OM Exam 1 Chap 1-5

8/13/2019 OM Exam 1 Chap 1-5

http://slidepdf.com/reader/full/om-exam-1-chap-1-5 101/187

Page 102: OM Exam 1 Chap 1-5

8/13/2019 OM Exam 1 Chap 1-5

http://slidepdf.com/reader/full/om-exam-1-chap-1-5 102/187

@) Three critical path activities are candidates for crashing on a C=: net&ork$ %ctivit detailsare in the ta"le "elo&$

%ctivit Iormal Time Iormal CostCrash

*uration Crash Cost

@ da s L9 <<< 9 da s L@ <<<N 2 da s L1 @<< da s L 5<<V 1 da s L6 <<< A da s L? ?<<

a$ 3hat is the crash cost per unit time for each of the three activities4 "$ 3hich activit should "e crashed first to cut one da from the pro ectBs durationG ho& much isadded to pro ect cost4c$ 3hich activit should "e the ne7t activit crashed to cut a second da from the pro ectBsdurationG ho& much is added to pro ect cost4%ns&er'Ka) crash cost O L1 <<< per da G crash cost N O L9<< per da G crash cost V O LA<< per da

K") select N adding L9<<Kc) select V adding LA<<*iff' Topic' Cost-time trade-offs and pro ect crashing%%CS0' %nal tic Skills," ective' .,2-9

A) Three critical path activities are candidates for crashing on a C=: net&ork$ %ctivit detailsare in the ta"le "elo&$

%ctivit Iormal Time Iormal Cost

Crash

*uration Crash Cost% A da s L@ <<< ? da s L1 <<<0 6 da s L <<< 2 da s L1< <<<C 1 da s LA <<< 11 da s L1 <<<

a$ 3hat is the crash cost per unit time for activit %4 "$ 3hat is the crash cost per unit time for activit 04c$ 3hich activit should "e crashed first to cut one da from the pro ectBs durationG ho& much isadded to pro ect cost4d$ 3hich activit should "e the ne7t activit crashed to cut a second da from the pro ectBsdurationG ho& much is added to pro ect cost4

e$ %ssuming no other paths "ecome critical ho& much can this pro ect "e shortened at &hat totaladded cost4%ns&er' Ka) L <<<G K") L5 <<<G Kc) % L <<<G Kd) % again L <<< additionalG Ke) 6 da sL16 <<<$*iff' Topic' Cost-time trade-offs and pro ect crashing%%CS0' %nal tic Skills," ective' .,2-91<) % net&ork consists of the follo&ing list$ Times are given in &eeks$

1-1<Cop right ; <11 =earson ducation >nc$

Page 103: OM Exam 1 Chap 1-5

8/13/2019 OM Exam 1 Chap 1-5

http://slidepdf.com/reader/full/om-exam-1-chap-1-5 103/187

%ctivit =receding ,ptimistic =ro"a"le =essimistic% -- 6 11 150 - 2 2 AC -- 9 1< 15

* % 0 2 6 ?0 5 9 11

F C 9 @ 12+ * 5 9H F 2 2 A

a$ *ra& the net&ork diagram$ "$ Calculate the e7pected duration and variance of each activit $c$ Calculate the e7pected duration and variance of the critical path$d$ Calculate the pro"a"ilit that the pro ect &ill "e completed in less than @ &eeks$

%ns&er'Ka)

1-1<2Cop right ; <11 =earson ducation >nc$

Page 104: OM Exam 1 Chap 1-5

8/13/2019 OM Exam 1 Chap 1-5

http://slidepdf.com/reader/full/om-exam-1-chap-1-5 104/187

K" c)

Task 7pected

time Eariance Std$ dev$ Slack Eariance% 1<$6 $ 6 1$6 2

0 5 1 1 @C 1< 1$??@ 1$222 < 1$??@* 6 <$555 <$99? 2

9$6 1$291 1$19? @F @$6 1$291 1$19? < 1$291+ 5 <$555 <$99? 2H 5 1 1 < 1

=ro ect $6 =ro ect 5$12AStd$ dev$ $<25

Kd) ! O K @- $6)/ $<2 O $?1 K=UO @) O $AA?*iff' Topic' =ro ect management techni#ues' = RT and C=:%%CS0' %nal tic Skills," ective' .,2-6

1-1<5Cop right ; <11 =earson ducation >nc$

Page 105: OM Exam 1 Chap 1-5

8/13/2019 OM Exam 1 Chap 1-5

http://slidepdf.com/reader/full/om-exam-1-chap-1-5 105/187

11) *ra& the %,% and %,I net&orks for the follo&ing pro ect and calculate the minimum pro ect duration$

%ctivit *urationKda s)

>mmediate=redecessors

% 1< ---0 @ ---C %* 5 %

6 0 C *%ns&er'%,I

%,%

:inimum duration is the longest of the three paths %* or a total of 1A da s$*iff' Topic' =ro ect management techni#ues' = RT and C=:%%CS0' %nal tic Skills," ective' .,2-

1 ) +iven the pro ect &ithin the ta"le "elo& calculate1-1<6

Cop right ; <11 =earson ducation >nc$

Page 106: OM Exam 1 Chap 1-5

8/13/2019 OM Exam 1 Chap 1-5

http://slidepdf.com/reader/full/om-exam-1-chap-1-5 106/187

a$ The critical path "$ The minimum pro ect durationc$ The amount of slack for each activit

%ctivit

*uration

Khours)

>mmediate

=redecessors% 5 Ione0 2 IoneC 1< Ione* ? 0 C

1 *F 1+ 6 % F

%ns&er'

K%) The critical path is C* F+K0) :inimum duration is K1<Q?Q1Q1Q6) or 5 hoursKC) C* F+ have no slack since on the critical path$ 0* F+ takes 1? hours so 0 has ? hoursof slack$ %+ takes A hours so % has 16 hours of slack$*iff' Topic' Cost-time trade-offs and pro ect crashing%%CS0' %nal tic Skills," ective' .,2-5

1-1<9Cop right ; <11 =earson ducation >nc$

Page 107: OM Exam 1 Chap 1-5

8/13/2019 OM Exam 1 Chap 1-5

http://slidepdf.com/reader/full/om-exam-1-chap-1-5 107/187

12) =irminBs 0ike Shop is "ehind on a custom "ike and needs to crash @ hours of time from the @-step pro ect$ +iven the pro ect ta"le "elo& calculate the crash cost for @ hours of time-savings$Suppose =irmin calls the customer and asks for a pro ect e7tension reducing the amount of timehe needs to crash$ Calculate "oth the ma7imum time-savings availa"le on a L 6 crash "udgetand the cost to crash four hours of savings$

%ctivit

Iormal*urationKhours)

IormalCost KL)

Crash*urationKhours)

CrashCost KS)

>mmediate=redecessors

% 1< < Ione0 2 16 2 %C 6 6 5 2< 0* 2 < 1 5 C

9 2< 5 56 CF 1 6 1 < C+ ? 26 9 6< F

H 1< 6< ? @< * +

%ns&er'

The critical path is %0C F+H &ith a time of 25 hours$ The cost per crash hour for theseactivities are @K0) 6KC) K*) ?$6K ) 16K+) 1<KH)$ C &ill "e crashed first for a savings of 1hour$ Ie7t &ill "e crashed for a savings of hours$ Then 0 can "e crashed for a savings of 1hour$ Ie7t H &ill "e crashed for a savings of 2 hours netting ? hours total at a cost ofK6Q16Q@Q2<) L6@$ ,nl 1 more hour is needed so + &ill "e crashed for an additional 16 dollarsnetting @ hours saved at a cost of 6@Q16 O L?2$ * &as not crashed "ecause it does not lie on thecritical path after an amount of crashing K al&a s W *)$ 3ith L 6 onl C and can "ecrashed saving 2 hours of time and spending L <$ % fractional component of 0 could "ecrashed K6/@) of an hour to spend the entire L 6 ho&ever most students should interpret this

#uestion as onl &hole-hour increments$ The cost to crash four hours &ould "e CQ Q0 crashingO L @$*iff' 2Topic' Cost-time trade-offs and pro ect crashing%%CS0' %nal tic Skills," ective' .,2-9

1-1<?Cop right ; <11 =earson ducation >nc$

Page 108: OM Exam 1 Chap 1-5

8/13/2019 OM Exam 1 Chap 1-5

http://slidepdf.com/reader/full/om-exam-1-chap-1-5 108/187

15) +iven the critical path "elo& calculate the follo&inga$ The crash cost per unit time savings for each activit $

"$ The ma7imum total crash time savings and cost$c$ The ma7imum total time-savings &ith a L2<<< "udget$

%ctivit Iormal Time Iormal Cost Crash*uration Crash Cost% @ da s L@ <<< ? da s L1 <<<0 6 da s L <<< 2 da s L1< <<<C 1< da s LA <<< A da s L1 <<<

%ns&er'K%) % saves 1 da for L5<<</da 0 saves da s for L5<<</da C saves 1 da at L2<<</da $K0) :a7 time savings is %Q0QC O 5 da s for a total of L16 <<<KC) 1 da could "e crashed from C$ This means ma7imum savings is 1 da $*iff'

Topic' Cost-time trade-offs and pro ect crashing%%CS0' %nal tic Skills," ective' .,2-9

1-1<@Cop right ; <11 =earson ducation >nc$

Page 109: OM Exam 1 Chap 1-5

8/13/2019 OM Exam 1 Chap 1-5

http://slidepdf.com/reader/full/om-exam-1-chap-1-5 109/187

16) Consider the net&ork presented in the follo&ing ta"le$ Calculatea$ %ll possi"le paths

"$ The critical pathc$ The slack availa"le at an non-critical pointsd$ The minimum pro ect duration

Task *urationK*a s)

>mmediate=redecessors

% 6 Ione0 2 %C 5 %* C

1 0 *

%ns&er'

K%) T&o paths are possi"le %C* and %0 $K0) The critical path is %C*KC) The slack availa"le at point 0 is *uration K%C* ) X *uration K%0 ) O 2 da s$ %ll other

points have no slack "ecause the are on the critical path$K*) The minimum pro ect duration is the length of the critical path %C* &hich is O 1 da s*iff' Topic' =ro ect management techni#ues' = RT and C=:%%CS0' %nal tic Skills," ective' .,2-5

1-1<ACop right ; <11 =earson ducation >nc$

Page 110: OM Exam 1 Chap 1-5

8/13/2019 OM Exam 1 Chap 1-5

http://slidepdf.com/reader/full/om-exam-1-chap-1-5 110/187

Operations Management, 10e (Heizer/Render)Chapter , -orecastin$

True/False

1) % naYve forecast for Septem"er sales of a product &ould "e e#ual to the forecast for %ugust$%ns&er' F%.S*iff' Topic' Time-series forecasting," ective' .,5-2

) The forecasting time hori!on and the forecasting techni#ues used tend to var over the lifec cle of a product$%ns&er' TR(*iff' Topic' 3hat is forecasting4

," ective' .,5-12) *emand Ksales) forecasts serve as inputs to financial marketing and personnel planning$%ns&er' TR(*iff' Topic' T pes of forecasts," ective' .,5-1

5) Forecasts of individual products tend to "e more accurate than forecasts of product families$%ns&er' F%.S*iff' Topic' Seven steps in the forecasting s stem," ective' .,5-1

6) :ost forecasting techni#ues assume that there is some underl ing sta"ilit in the s stem$%ns&er' TR(*iff' Topic' Seven steps in the forecasting s stem," ective' .,5-1

9) The sales force composite forecasting method relies on salespersonsB estimates of e7pectedsales$%ns&er' TR(*iff' 1Topic' Forecasting approaches," ective' .,5-

1-11<Cop right ; <11 =earson ducation >nc$

Page 111: OM Exam 1 Chap 1-5

8/13/2019 OM Exam 1 Chap 1-5

http://slidepdf.com/reader/full/om-exam-1-chap-1-5 111/187

?) % time-series model uses a series of past data points to make the forecast$%ns&er' TR(*iff' Topic' Forecasting approaches," ective' .,5-

@) The #uarterl 8make meeting8 of .e7us dealers is an e7ample of a sales force compositeforecast$%ns&er' TR(*iff' 1Topic' Forecasting approaches," ective' .,5-

A) C cles and random variations are "oth components of time series$%ns&er' TR(*iff' 1Topic' Time-series forecasting," ective' .,5-2

1<) % naive forecast for Septem"er sales of a product &ould "e e#ual to the sales in %ugust$%ns&er' TR(*iff' 1Topic' Time-series forecasting," ective' .,5-2

11) ,ne advantage of e7ponential smoothing is the limited amount of record keeping involved$%ns&er' TR(*iff' Topic' Time-series forecasting," ective' .,5-2

1 ) The larger the num"er of periods in the simple moving average forecasting method thegreater the methodBs responsiveness to changes in demand$%ns&er' F%.S*iff' Topic' Time-series forecasting," ective' .,5-2

12) :ean S#uared rror and Coefficient of Correlation are t&o measures of the overall error of aforecasting model$%ns&er' F%.S*iff' 1Topic' Time-series forecasting," ective' .,5-5

1-111Cop right ; <11 =earson ducation >nc$

Page 112: OM Exam 1 Chap 1-5

8/13/2019 OM Exam 1 Chap 1-5

http://slidepdf.com/reader/full/om-exam-1-chap-1-5 112/187

15) >n trend pro ection the trend component is the slope of the regression e#uation$%ns&er' TR(*iff' 1Topic' Time-series forecasting," ective' .,5-2

16) >n trend pro ection a negative regression slope is mathematicall impossi"le$%ns&er' F%.S*iff' Topic' Time-series forecasting," ective' .,5-2

19) Seasonal indices ad ust ra& data for patterns that repeat at regular time intervals$%ns&er' TR(*iff' Topic' Time-series forecasting," ective' .,5-6

1?) =atterns in the data that occur ever several ears are called circuits$%ns&er' F%.S*iff' 1Topic' Time-series forecasting," ective' no .,

1@) .inear-regression anal sis is a straight-line mathematical model to descri"e the functionalrelationships "et&een independent and dependent varia"les$%ns&er' TR(*iff' 1Topic' %ssociative forecasting methods' Regression and correlation anal sis," ective' .,5-9

1A) The larger the standard error of the estimate the more accurate the forecasting model$%ns&er' F%.S*iff' 1Topic' %ssociative forecasting methods' Regression and correlation anal sis," ective' .,5-5

<) % trend pro ection e#uation &ith a slope of <$?@ means that there is a <$?@ unit rise in N forever unit of time that passes$%ns&er' TR(*iff' Topic' Time-series forecasting' Trend pro ections," ective' .,5-9

1-11Cop right ; <11 =earson ducation >nc$

Page 113: OM Exam 1 Chap 1-5

8/13/2019 OM Exam 1 Chap 1-5

http://slidepdf.com/reader/full/om-exam-1-chap-1-5 113/187

1) >n a regression e#uation &here N is demand and is advertising a coefficient of

determination KR ) of $?< means that ?< of the variance in advertising is e7plained " demand$%ns&er' F%.S*iff' Topic' %ssociative forecasting methods' Regression and correlation anal sis

," ective' .,5-9

) *emand c cles for individual products can "e driven " product life c cles$%ns&er' TR(*iff' Topic' Time-series forecasting," ective' .,5-6

2) >f a forecast is consistentl greater than Kor less than) actual values the forecast is said to "e "iased$%ns&er' TR(

*iff' Topic' :onitoring and controlling forecasts," ective' .,5-5

5) Focus forecasting tries a variet of computer models and selects the "est one for a particularapplication$%ns&er' TR(*iff' Topic' :onitoring and controlling forecasts," ective' .,5-5

6) :an service firms use point-of-sale computers to collect detailed records needed foraccurate short-term forecasts$%ns&er' TR(*iff' Topic' Forecasting in the service sector ," ective' .,5-5

9) Technological forecasts address the "usiness c cle " predicting inflation rates monesupplies housing starts and other planning indicators$%ns&er' F%.S*iff'

Topic' 3hat is forecasting4," ective' no .,

?) Regression lines graphicall depict 8cause-and-effect8 relationships$%ns&er' F%.S*iff' Topic' Correlation coefficients for regression lines," ective' .,5-9

1-112Cop right ; <11 =earson ducation >nc$

Page 114: OM Exam 1 Chap 1-5

8/13/2019 OM Exam 1 Chap 1-5

http://slidepdf.com/reader/full/om-exam-1-chap-1-5 114/187

:ultiple Choice

1) 3hat t&o num"ers are contained in the dail report to the C , of 3alt *isne =arks ZResorts regarding the si7 ,rlando parks4%) esterda Bs forecasted attendance and esterda Bs actual attendance

0) esterda Bs actual attendance and toda Bs forecasted attendanceC) esterda Bs forecasted attendance and toda Bs forecasted attendance*) esterda Bs actual attendance and last earBs actual attendance) esterda Bs forecasted attendance and the ear-to-date average dail forecast error %ns&er' %*iff' Topic' +lo"al compan profile," ective' no .,

) Forecasts%) "ecome more accurate &ith longer time hori!ons

0) are rarel perfectC) are more accurate for individual items than for groups of items*) all of the a"ove) none of the a"ove%ns&er' 0*iff' Topic' 3hat is forecasting4," ective' .,5-1

2) ,ne use of short-range forecasts is to determine%) planning for ne& products0) capital e7pendituresC) research and development plans*) facilit location) o" assignments%ns&er' *iff' Topic' 3hat is forecasting4," ective' .,5-1

5) Forecasts are usuall classified " time hori!on into three categories%) short-range medium-range and long-range0) finance/accounting marketing and operationsC) strategic tactical and operational*) e7ponential smoothing regression and time series) departmental organi!ational and industrial%ns&er' %*iff' 1Topic' 3hat is forecasting4," ective' .,5-16) % forecast &ith a time hori!on of a"out 2 months to 2 ears is t picall called a%) long-range forecast

1-115Cop right ; <11 =earson ducation >nc$

Page 115: OM Exam 1 Chap 1-5

8/13/2019 OM Exam 1 Chap 1-5

http://slidepdf.com/reader/full/om-exam-1-chap-1-5 115/187

0) medium-range forecastC) short-range forecast*) &eather forecast) strategic forecast%ns&er' 0

*iff' Topic' 3hat is forecasting4," ective' .,5-1

9) Forecasts used for ne& product planning capital e7penditures facilit location or e7pansionand RZ* t picall utili!e a%) short-range time hori!on0) medium-range time hori!onC) long-range time hori!on*) naive method "ecause there is no data histor) all of the a"ove

%ns&er' C*iff' Topic' 3hat is forecasting4," ective' .,5-1

?) ,rgani!ations use &hich three ma or t pes of forecasts including t&o that ma fall outside therole of the operations manager4%) strategic tactical and operational0) economic technological and demandC) e7ponential smoothing *elphi and regression*) causal time-series and seasonal) departmental organi!ational and territorial%ns&er' 0*iff' Topic' T pes of forecasts," ective' .,5-

@) 3hich of the follo&ing is not a step in the forecasting process4%) *etermine the use of the forecast$0) liminate an assumptions$C) *etermine the time hori!on$*) Select forecasting model$) Ealidate and implement the results$%ns&er' 0*iff' Topic' The strategic importance of forecasting," ective' .,5-

1-116Cop right ; <11 =earson ducation >nc$

Page 116: OM Exam 1 Chap 1-5

8/13/2019 OM Exam 1 Chap 1-5

http://slidepdf.com/reader/full/om-exam-1-chap-1-5 116/187

A) The t&o general approaches to forecasting are%) #ualitative and #uantitative0) mathematical and statisticalC) udgmental and #ualitative*) historical and associative

) udgmental and associative%ns&er' %*iff' 1Topic' Forecasting approaches," ective' .,5-

1<) 3hich of the follo&ing uses three t pes of participants' decision makers staff personnel andrespondents4%) e7ecutive opinions0) sales force compositesC) the *elphi method

*) associative models) time series anal sis%ns&er' C*iff' Topic' Forecasting approaches," ective' .,5-

11) The forecasting model that pools the opinions of a group of e7perts or managers is kno&n asthe%) e7pert udgment model0) multiple regression modelC) ur of e7ecutive opinion model*) consumer market surve model) management coefficients model%ns&er' C*iff' Topic' Forecasting approaches," ective' .,5-

1 ) 3hich of the follo&ing is not a t pe of #ualitative forecasting4%) e7ecutive opinions0) sales force compositesC) consumer surve s*) the *elphi method) moving average%ns&er' *iff' Topic' Forecasting approaches," ective' .,5-

1-119Cop right ; <11 =earson ducation >nc$

Page 117: OM Exam 1 Chap 1-5

8/13/2019 OM Exam 1 Chap 1-5

http://slidepdf.com/reader/full/om-exam-1-chap-1-5 117/187

Page 118: OM Exam 1 Chap 1-5

8/13/2019 OM Exam 1 Chap 1-5

http://slidepdf.com/reader/full/om-exam-1-chap-1-5 118/187

1?) 3hich of the follo&ing is not present in a time series4%) seasonalit0) operational variationsC) trend*) c cles

) random variations%ns&er' 0*iff' Topic' Time-series forecasting," ective' .,5-2

1@) The fundamental difference "et&een c cles and seasonalit is the%) duration of the repeating patterns0) magnitude of the variationC) a"ilit to attri"ute the pattern to a cause*) all of the a"ove

) none of the a"ove%ns&er' %*iff' Topic' Time-series forecasting," ective' .,5-6

1A) >n time series &hich of the follo&ing cannot "e predicted4%) large increases in demand0) c clesC) seasonal fluctuations*) random fluctuations) large decreases in demand%ns&er' **iff' Topic' Time-series forecasting," ective' .,5-2

1-11@Cop right ; <11 =earson ducation >nc$

Page 119: OM Exam 1 Chap 1-5

8/13/2019 OM Exam 1 Chap 1-5

http://slidepdf.com/reader/full/om-exam-1-chap-1-5 119/187

Page 120: OM Exam 1 Chap 1-5

8/13/2019 OM Exam 1 Chap 1-5

http://slidepdf.com/reader/full/om-exam-1-chap-1-5 120/187

2) % si7-month moving average forecast is generall "etter than a three-month moving averageforecast if demand%) is rather sta"le0) has "een changing due to recent promotional effortsC) follo&s a do&n&ard trend

*) e7ceeds one million units per ear ) follo&s an up&ard trend%ns&er' %*iff' Topic' Time-series forecasting," ective' .,5-2

5) >ncreasing the num"er of periods in a moving average &ill accomplish greater smoothing "ut at the e7pense of %) manager understanding0) accurac

C) sta"ilit*) responsiveness to changes) %ll of the a"ove are diminished &hen the num"er of periods increases$%ns&er' **iff' Topic' Time-series forecasting," ective' .,5-2

6) 3hich of the follo&ing statements comparing the &eighted moving average techni#ue ande7ponential smoothing is true 4%) 7ponential smoothing is more easil used in com"ination &ith the *elphi method$0) :ore emphasis can "e placed on recent values using the &eighted moving average$C) 7ponential smoothing is considera"l more difficult to implement on a computer$*) 7ponential smoothing t picall re#uires less record keeping of past data$) 7ponential smoothing allo&s one to develop forecasts for multiple periods &hereas&eighted moving averages does not$%ns&er' **iff' Topic' Time-series forecasting," ective' .,5-2

1-1 <Cop right ; <11 =earson ducation >nc$

Page 121: OM Exam 1 Chap 1-5

8/13/2019 OM Exam 1 Chap 1-5

http://slidepdf.com/reader/full/om-exam-1-chap-1-5 121/187

9) 3hich time-series model uses past forecasts and past demand data to generate a ne&forecast4%) naive0) moving averageC) &eighted moving average

*) e7ponential smoothing) regression anal sis%ns&er' **iff' Topic' Time-series forecasting," ective' .,5-2

?) 3hich is not a characteristic of e7ponential smoothing4%) smoothes random variations in the data0) easil altered &eighting schemeC) &eights each historical value e#uall

*) has minimal data storage re#uirements) Ione of the a"oveG the are all characteristics of e7ponential smoothing$%ns&er' C*iff' Topic' Time-series forecasting," ective' .,5-2

@) 3hich of the follo&ing smoothing constants &ould make an e7ponential smoothing forecaste#uivalent to a naive forecast4%) <0) 1 divided " the num"er of periodsC) <$6*) 1$<) cannot "e determined%ns&er' **iff' Topic' Time-series forecasting," ective' .,5-2

1-1 1Cop right ; <11 =earson ducation >nc$

Page 122: OM Exam 1 Chap 1-5

8/13/2019 OM Exam 1 Chap 1-5

http://slidepdf.com/reader/full/om-exam-1-chap-1-5 122/187

A) +iven an actual demand of 1<2 a previous forecast value of AA and an alpha of $5 thee7ponential smoothing forecast for the ne7t period &ould "e%) A5$90) A?$5C) 1<<$9

*) 1<1$9) 1<2$<%ns&er' C*iff' Topic' Time-series forecasting%%CS0' %nal tic Skills," ective' .,5-2

2<) % forecast "ased on the previous forecast plus a percentage of the forecast error is aKn)%) #ualitative forecast0) naive forecastC) moving average forecast*) &eighted moving average forecast) e7ponentiall smoothed forecast%ns&er' *iff' Topic' Time-series forecasting," ective' .,5-2

21) +iven an actual demand of 91 a previous forecast of 6@ and an alpha of $2 &hat &ould theforecast for the ne7t period "e using simple e7ponential smoothing4%) 56$60) 6?$1C) 6@$A*) 91$<) 96$6%ns&er' C*iff' Topic' Time-series forecasting%%CS0' %nal tic Skills," ective' .,5-2

1-1Cop right ; <11 =earson ducation >nc$

Page 123: OM Exam 1 Chap 1-5

8/13/2019 OM Exam 1 Chap 1-5

http://slidepdf.com/reader/full/om-exam-1-chap-1-5 123/187

2 ) 3hich of the follo&ing values of alpha &ould cause e7ponential smoothing to respond themost slo&l to forecast errors4%) <$1<0) <$ <C) <$5<

*) <$@<) cannot "e determined%ns&er' %*iff' Topic' Time-series forecasting," ective' .,5-2

22) % forecasting method has produced the follo&ing over the past five months$ 3hat is themean a"solute deviation4

%ctual Forecast rror [ rror[

1< 11 -1 1@ 1< -1< @ 9 9 < <A @ 1 1

%) -<$0) -1$<C) <$<*) 1$) @$9

%ns&er' **iff' Topic' Time-series forecasting%%CS0' %nal tic Skills," ective' .,5-5

25) The primar purpose of the mean a"solute deviation K:%*) in forecasting is to%) estimate the trend line0) eliminate forecast errorsC) measure forecast accurac*) seasonall ad ust the forecast

) all of the a"ove%ns&er' C*iff' Topic' Time-series forecasting," ective' .,5-5

1-1 2Cop right ; <11 =earson ducation >nc$

Page 124: OM Exam 1 Chap 1-5

8/13/2019 OM Exam 1 Chap 1-5

http://slidepdf.com/reader/full/om-exam-1-chap-1-5 124/187

26) +iven forecast errors of -1 5 @ and -2 &hat is the mean a"solute deviation4%) 0) 2C) 5*) @

) 19%ns&er' C*iff' Topic' Time-series forecasting%%CS0' %nal tic Skills," ective' .,5-5

29) The last four months of sales &ere @ 1< 16 and A units$ The last four forecasts &ere 6 911 and 1 units$ The :ean %"solute *eviation K:%*) is%) 0) -1<C) 2$6*) A) 1<$6%ns&er' C*iff' Topic' Time-series forecasting%%CS0' %nal tic Skills," ective' .,5-5

2?) % time series trend e#uation is 6$2 Q $1 $ 3hat is our forecast for period ?4%) 2$0) 6$2C) ?$5*) 5<$<) cannot "e determined%ns&er' **iff' Topic' Time-series forecasting%%CS0' %nal tic Skills," ective' .,5-2

1-1 5Cop right ; <11 =earson ducation >nc$

Page 125: OM Exam 1 Chap 1-5

8/13/2019 OM Exam 1 Chap 1-5

http://slidepdf.com/reader/full/om-exam-1-chap-1-5 125/187

2@) For a given product demand the time series trend e#uation is 62 - 5 $ The negative sign onthe slope of the e#uation%) is a mathematical impossi"ilit0) is an indication that the forecast is "iased &ith forecast values lo&er than actual valuesC) is an indication that product demand is declining

*) implies that the coefficient of determination &ill also "e negative) implies that the cumulative error &ill "e negative%ns&er' C*iff' Topic' Time-series forecasting," ective' .,5-2

2A) Namaha manufactures &hich set of products &ith complementar demands to addressseasonal fluctuations4%) golf clu"s and skis0) s&imming suits and &inter ackets

C) et skis and sno&mo"iles*) pianos and guitars) ice skates and &ater skis%ns&er' C*iff' Topic' Time-series forecasting," ective' .,5-6

5<) 3hich of the follo&ing is true regarding the t&o smoothing constants of the Forecast>ncluding Trend KF>T) model4%) ,ne constant is positive &hile the other is negative$0) The are called :%* and cumulative error$C) %lpha is al&a s smaller than "eta$*) ,ne constant smoothes the regression intercept &hereas the other smoothes the regressionslope$) Their values are determined independentl $%ns&er' *iff' Topic' Time-series forecasting," ective' .,5-2

1-1 6Cop right ; <11 =earson ducation >nc$

Page 126: OM Exam 1 Chap 1-5

8/13/2019 OM Exam 1 Chap 1-5

http://slidepdf.com/reader/full/om-exam-1-chap-1-5 126/187

51) *emand for a certain product is forecast to "e @<< units per month averaged over all 1months of the ear$ The product follo&s a seasonal pattern for &hich the Januar monthl inde7is 1$ 6$ 3hat is the seasonall -ad usted sales forecast for Januar 4%) 95< units0) ?A@$?6 units

C) @<1$ 6 units*) 1<<< units) @@$22 units%ns&er' **iff' Topic' Time series forecasting%%CS0' %nal tic Skills," ective' .,5-6

5 ) % seasonal inde7 for a monthl series is a"out to "e calculated on the "asis of three earsBaccumulation of data$ The three previous Jul values &ere 11< 16< and 12<$ The average over

all months is 1A<$ The appro7imate seasonal inde7 for Jul is%) <$5@?0) <$9@5C) 1$59*) $<62) cannot "e calculated &ith the information given%ns&er' 0*iff' Topic' Time-series forecasting%%CS0' %nal tic Skills," ective' .,5-6

52) % fundamental distinction "et&een trend pro ection and linear regression is that%) trend pro ection uses least s#uares &hile linear regression does not0) onl linear regression can have a negative slopeC) in trend pro ection the independent varia"le is timeG in linear regression the independentvaria"le need not "e time "ut can "e an varia"le &ith e7planator po&er *) trend pro ection can "e a function of several varia"les &hile linear regression can onl "e afunction of one varia"le) trend pro ection uses t&o smoothing constants not ust one%ns&er' C*iff' Topic' %ssociative forecasting methods' Regression and correlation anal sis," ective' .,5-9

1-1 9Cop right ; <11 =earson ducation >nc$

Page 127: OM Exam 1 Chap 1-5

8/13/2019 OM Exam 1 Chap 1-5

http://slidepdf.com/reader/full/om-exam-1-chap-1-5 127/187

55) The degree or strength of a relationship "et&een t&o varia"les is sho&n " the%) alpha0) meanC) mean a"solute deviation*) correlation coefficient

) cumulative error %ns&er' **iff' Topic' %ssociative forecasting methods' Regression and correlation anal sis," ective' .,5-9

56) >f t&o varia"les &ere perfectl correlated the correlation coefficient r &ould e#ual%) <0) -1C) 1*) 0 or C

) none of the a"ove%ns&er' **iff' Topic' %ssociative forecasting methods' Regression and correlation anal sis," ective' .,5-9

59) The last four &eekl values of sales &ere @< 1<< 1<6 and A< units$ The last four forecasts&ere 9< @< A6 and ?6 units$ These forecasts illustrate%) #ualitative methods0) adaptive smoothingC) slope*) "ias) trend pro ection%ns&er' **iff' 1Topic' :onitoring and controlling forecasts," ective' .,5-?

5?) The tracking signal is the%) standard error of the estimate0) a"solute deviation of the last periodBs forecastC) mean a"solute deviation K:%*)*) ratio of cumulative error/:%*) mean a"solute percentage error K:%= )%ns&er' **iff' Topic' :onitoring and controlling forecasts," ective' .,5-?

1-1 ?Cop right ; <11 =earson ducation >nc$

Page 128: OM Exam 1 Chap 1-5

8/13/2019 OM Exam 1 Chap 1-5

http://slidepdf.com/reader/full/om-exam-1-chap-1-5 128/187

5@) Computer monitoring of tracking signals and self-ad ustment if a signal passes a preset limitis characteristic of %) e7ponential smoothing including trend0) adaptive smoothingC) trend pro ection

*) focus forecasting) multiple regression anal sis%ns&er' 0*iff' Topic' :onitoring and controlling forecasts," ective' .,5-?

5A) :an services maintain records of sales noting%) the da of the &eek 0) unusual eventsC) &eather

*) holida s) all of the a"ove%ns&er' *iff' Topic' Forecasting in the service sector ," ective' .,5-

6<) Taco 0ellBs uni#ue emplo ee scheduling practices are partl the result of using%) point-of-sale computers to track food sales in 16 minute intervals0) focus forecastingC) a si7-&eek moving average forecasting techni#ue*) multiple regression) % and C are "oth correct$%ns&er' *iff' Topic' Forecasting in the service sector ," ective' .,5-2

61) 3hich of the follo&ing most re#uires long-range forecasting Kas opposed to short-range ormedium-range forecasting) for its planning purposes4%) o" scheduling0) production levelsC) cash "udgeting*) capital e7penditures) purchasing%ns&er' **iff' Topic' Forecasting time hori!ons," ective' .,5-1

1-1 @Cop right ; <11 =earson ducation >nc$

Page 129: OM Exam 1 Chap 1-5

8/13/2019 OM Exam 1 Chap 1-5

http://slidepdf.com/reader/full/om-exam-1-chap-1-5 129/187

6 ) Suppose that demand in period 1 &as ? units and the demand in period &as A units$%ssume that the forecast for period 1 &as for 6 units$ >f the firm uses e7ponential smoothing &ithan alpha value of $ < &hat should "e the forecast for period 24 KRound ans&ers to t&o decimal

places$)%) A$<<

0) 2$?C) A$5@*) 6$<<) 9$1%ns&er' *iff' Topic' Time-series forecasting," ective' .,5-2

62) e7presses the error as a percent of the actual values undistorted " a singlelarge value$

%) :%*0) :SC) :%=*) F>T) The smoothing constant%ns&er' C*iff' Topic' :easuring forecast error ," ective' .,5-5

65) >f 0randon d&ard &ere &orking to develop a forecast using a moving averages approach "ut he noticed a detecta"le trend in the historical data he should%) use &eights to place more emphasis on recent data0) use &eights to minimi!e the importance of the trendC) change to a naYve approach*) use a simple moving average) change to a #ualitative approach%ns&er' %*iff' Topic' :oving averages," ective' .,5-2

Short %ns&er

1) forecasts are concerned &ith rates of technological progress &hich can result inthe "irth of e7citing ne& products re#uiring ne& plants and e#uipment$%ns&er' Technological*iff' 1Topic' T pes of forecasts," ective' .,5-1

) forecasts address the "usiness c cle " predicting inflation rates mone supplies1-1 A

Cop right ; <11 =earson ducation >nc$

Page 130: OM Exam 1 Chap 1-5

8/13/2019 OM Exam 1 Chap 1-5

http://slidepdf.com/reader/full/om-exam-1-chap-1-5 130/187

housing starts and other planning indicators$%ns&er' conomic*iff' Topic' T pes of forecasts," ective' .,5-1

2) *emand forecasts also called forecasts are pro ections of demand for acompan Bs products or services$%ns&er' sales*iff' Topic' T pes of forecasts," ective' .,5-1

5) forecasts emplo one or more mathematical models that rel on historical dataand/or associative varia"les to forecast demand$%ns&er' Duantitative

*iff' Topic' Forecasting approaches," ective' .,5-

6) is a forecasting techni#ue "ased upon salespersonsB estimates of e7pected sales$%ns&er' Sales force composite*iff' Topic' Forecasting approaches," ective' .,5-

9) forecasts use a series of past data points to make a forecast$%ns&er' Time-series*iff' Topic' Forecasting approaches," ective' .,5-

?) %Kn) forecast uses an average of the most recent periods of data to forecast thene7t period$%ns&er' moving average*iff' Topic' Forecasting approaches," ective' .,5-2

@) The smoothing constant is a &eighting factor used in $%ns&er' e7ponential smoothing*iff' Topic' Forecasting approaches," ective' .,5-2

1-12<Cop right ; <11 =earson ducation >nc$

Page 131: OM Exam 1 Chap 1-5

8/13/2019 OM Exam 1 Chap 1-5

http://slidepdf.com/reader/full/om-exam-1-chap-1-5 131/187

A) .inear regression is kno&n as aKn) "ecause it incorporates varia"les or factorsthat might influence the #uantit "eing forecast$%ns&er' associative model*iff' 1Topic' Forecasting approaches

," ective' .,5-

1<) % measure of forecast error that does not depend on the magnitude of the item "eing forecastis the $%ns&er' mean a"solute percent error or :%=*iff' 1Topic' Forecasting approaches," ective' .,5-5

11) is a measure of overall forecast error for a model$%ns&er' :%* or :ean %"solute *eviation

*iff' Topic' Forecasting approaches," ective' .,5-5

1 ) 3hen one constant is used to smooth the forecast average and a second constant is used tosmooth the trend the forecasting method is $%ns&er' e7ponential smoothing &ith trend ad ustment or trend-ad usted smoothing or second-order smoothing or dou"le smoothing*iff' Topic' Forecasting approaches," ective' .,5-2

12) is a time-series forecasting method that fits a trend line to a series of historicaldata points and then pro ects the line into the future for forecasts$%ns&er' Trend pro ection*iff' Topic' Forecasting approaches," ective' .,5-

15) The measures the strength of the relationship "et&een t&o varia"les$%ns&er' coefficient of correlation*iff' Topic' %ssociative forecasting methods' Regression and correlation anal sis," ective' .,5-9

1-121Cop right ; <11 =earson ducation >nc$

Page 132: OM Exam 1 Chap 1-5

8/13/2019 OM Exam 1 Chap 1-5

http://slidepdf.com/reader/full/om-exam-1-chap-1-5 132/187

16) >f a "ar"ershop operator noted that Tuesda Bs "usiness &as t picall t&ice as heav as3ednesda Bs and that Frida Bs "usiness &as t picall the "usiest of the &eek "usiness at the

"ar"ershop is su" ect to $%ns&er' seasonal variations*iff'

Topic' Forecasting approaches' Seasonal variations in data," ective' .,5-6

19) forecasting tries a variet of computer models and selects the "est one for a particular application$%ns&er' Focus*iff' Topic' :onitoring and controlling forecasts," ective' .,5-?

1?) are useful if &e can assume that market demands &ill sta fairl stead over

time$%ns&er' :oving averages*iff' Topic' :oving averages," ective' .,5-2

1@) %n approach to e7ponential smoothing in &hich the smoothing constant is automaticallchanged to keep errors to a minimum is called $%ns&er' adaptive smoothing*iff' Topic' :onitoring and controlling forecasts," ective' .,5-2

1A) % skeptical manager asks &hat short-range forecasts can "e used for$ +ive her three possi"leuses/purposes$%ns&er' %n three of' planning purchasing o" scheduling &ork force levels o" assignments

production levels$*iff' Topic' 3hat is forecasting4," ective' .,5-1

<) % skeptical manager asks &hat long-range forecasts can "e used for$ +ive her three possi"leuses/purposes$%ns&er' %n three of' planning ne& products capital e7penditures facilit location ore7pansion research and development$*iff' Topic' 3hat is forecasting4," ective' .,5-1

1-12Cop right ; <11 =earson ducation >nc$

Page 133: OM Exam 1 Chap 1-5

8/13/2019 OM Exam 1 Chap 1-5

http://slidepdf.com/reader/full/om-exam-1-chap-1-5 133/187

1) *escri"e the three forecasting time hori!ons and their use$%ns&er' Forecasting time horizons are' short rangeMgenerall less than three months used for

purchasing o" scheduling &ork force levels production levelsG medium rangeMusuall fromthree months up to three ears used for sales planning production planning and "udgeting cash

"udgeting anal !ing operating plansG long rangeMusuall three ears or more used for ne&

product development capital e7penditures facilit planning and RZ*$*iff' Topic' 3hat is forecasting4," ective' .,5-1

) .ist and "riefl descri"e the three ma or t pes of forecasts$%ns&er' The three t pes are economic technological and demandG economic refers tomacroeconomic gro&th and financial varia"lesG technological refers to forecasting amount oftechnological advance or futurismG demand refers to product demand$*iff' Topic' T pes of forecasts

," ective' .,5-2) >dentif the seven steps involved in forecasting$%ns&er'1$ *etermine the use of the forecast$$ Select the items that are to "e forecast$2$ *etermine the time hori!on of the forecast$5$ Select the forecasting modelKs)$6$ +ather the data needed to make the forecast$9$ :ake the forecast$?$ Ealidate the forecasting mode and implement the results$*iff' Topic' Seven steps in the forecasting s stem," ective' .,5-

5) 3hat are the realities of forecasting that companies face4%ns&er' First forecasts are seldom perfect$ Second most forecasting techni#ues assume thatthere is some underl ing sta"ilit in the s stem$ Finall "oth product famil and aggregatedforecasts are more accurate than individual product forecasts$*iff' Topic' Seven steps in the forecasting s stem," ective' .,5-

6) 3hat are the differences "et&een #uantitative and #ualitative forecasting methods4%ns&er' Duantitative methods use mathematical models to anal !e historical data$ Dualitativemethods incorporate such factors as the decision makerBs intuition emotions personale7periences and value s stems in determining the forecast$*iff' Topic' Forecasting approaches," ective' .,5-

9) >dentif four #uantitative forecasting methods$1-122

Cop right ; <11 =earson ducation >nc$

Page 134: OM Exam 1 Chap 1-5

8/13/2019 OM Exam 1 Chap 1-5

http://slidepdf.com/reader/full/om-exam-1-chap-1-5 134/187

%ns&er' The list includes naive moving averages e7ponential smoothing trend pro ection andlinear regression$*iff' Topic' Forecasting approaches," ective' .,5-

?) 3hat is a time-series forecasting model4%ns&er' % time-series forecasting model is an mathematical model that uses historical valuesof the #uantit of interest to predict future values of that #uantit $*iff' 1Topic' Forecasting approaches," ective' .,5-

@) 3hat is the difference "et&een an associative model and a time-series model4%ns&er' % time-series model uses onl historical values of the #uantit of interest to predictfuture values of that #uantit $ The associative model on the other hand attempts to identif

underl ing factors that control the variation of the #uantit of interest predict future values ofthese factors and use these predictions in a model to predict future values of the specific #uantitof interest$*iff' Topic' Forecasting approaches," ective' .,5-

A) Iame and discuss three #ualitative forecasting methods$%ns&er' Qualitative forecasting methods include' ur of e7ecutive opinion &here high-levelmanagers arrive at a group estimate of demandG sales force composite &here salespersonsBestimates are aggregatedG *elphi method &here respondents provide inputs to a group ofdecision makersG the group of decision makers often e7perts then make the actual forecastGconsumer market surve &here consumers are #ueried a"out their future purchase plans$*iff' Topic' Forecasting approaches," ective' .,5-

2<) >dentif four components of a time series$ 3hich one of these is rarel forecast4 3h is thisso4%ns&er' Trend seasonalit c cles and random variation$ Since random variations follo& nodiscerni"le pattern the cannot "e predicted and thus are not forecast$*iff' Topic' Time-series forecasting," ective' .,5-2

1-125Cop right ; <11 =earson ducation >nc$

Page 135: OM Exam 1 Chap 1-5

8/13/2019 OM Exam 1 Chap 1-5

http://slidepdf.com/reader/full/om-exam-1-chap-1-5 135/187

21) Compare seasonal effects and c clical effects$%ns&er' % c cle is longer Kt picall several ears) than a season Kt picall da s &eeks monthsor #uarters)$ % c cle has varia"le duration &hile a season has fi7ed duration and regularrepetition$*iff'

Topic' Time-series forecasting," ective' .,5-6

2 ) *istinguish "et&een a moving average model and an e7ponential smoothing model$%ns&er' 7ponential smoothing is a &eighted moving average model &herein previous valuesare &eighted in a specific manner--in particular all previous values are &eighted &ith a set of&eights that decline e7ponentiall $*iff' Topic' Time-series forecasting," ective' .,5-2

22) *escri"e three popular measures of forecast accurac $%ns&er' Measures of forecast accuracy include' Ka) MAD Kmean a"solute deviation) is a sum ofthe a"solute values of individual errors divided " the num"er of periods of data$ K") MSE Kmeans#uared error) is the average of the s#uared differences "et&een the forecast and o"servedvalues$ Kc) MA E Kmean a"solute percent error) is independent of the magnitude of the varia"le

"eing forecast$*iff' Topic' Forecasting approaches' :easuring forecast error ," ective' .,5-5

25) +ive an e7ampleMother than a restaurant or other food-service firmMof an organi!ation thate7periences an hourl seasonal pattern$ KThat is each hour of the da has a pattern that tends torepeat da after da $) 7plain$%ns&er' %ns&er &ill var $ Ho&ever t&o non-food e7amples &ould "e "anks and movietheaters$*iff' Topic' Time-series forecasting%%CS0' Reflective Thinking," ective' .,5-6

26) 7plain the role of regression models Ktime series and other&ise) in forecasting$ That is ho&is trend pro ection a"le to forecast4 Ho& is regression used for causal forecasting4%ns&er' For trend pro ection the independent varia"le is time$ The trend pro ection e#uationhas a slope that is the change in demand per period$ To forecast the demand for period t performthe calculation a Q "t$ For causal forecasting the independent varia"les are predictors of theforecast value or dependent varia"le$ The slope of the regression e#uation is the change in the Nvaria"le per unit change in the varia"le$*iff' 2Topic' Time-series forecasting," ective' .,5-9

29) >dentif three advantages of the moving average forecasting model$ >dentif three1-126

Cop right ; <11 =earson ducation >nc$

Page 136: OM Exam 1 Chap 1-5

8/13/2019 OM Exam 1 Chap 1-5

http://slidepdf.com/reader/full/om-exam-1-chap-1-5 136/187

Page 137: OM Exam 1 Chap 1-5

8/13/2019 OM Exam 1 Chap 1-5

http://slidepdf.com/reader/full/om-exam-1-chap-1-5 137/187

%ns&er' Simple moving averages are useful &here there is no identifia"le trend in the historicaldata i$e$ demand has "een fairl stead over time$ >f there &as an identifia"le trend &eightedmoving averages &ould provide a more accurate forecast since higher &eights &ould "e put onthe most recent data$*iff'

Topic' Time series forecastingG :oving averages," ective' .,5-2

ssa

1) 3eekl sales of ten-grain "read at the local organic food market are in the ta"le "elo&$ 0asedon this data forecast &eek A using a five-&eek moving average$

3eek Sales1 516

2@A2 5 <5 2@6 51<9 52? 5<6@ 5 1

%ns&er' K2@ Q51<Q52 Q5<6Q5 1)/6 O 51<$<*iff' 1Topic' Time-series forecasting%%CS0' %nal tic Skills," ective' .,5-2

1-12?Cop right ; <11 =earson ducation >nc$

Page 138: OM Exam 1 Chap 1-5

8/13/2019 OM Exam 1 Chap 1-5

http://slidepdf.com/reader/full/om-exam-1-chap-1-5 138/187

) +iven the follo&ing data calculate the three- ear moving averages for ears 5 through 1<$

Near *emand1 ?5

A<

2 6A5 A16 15<9 A@? 11<@ 1 2A AA

%ns&er'Near *emand 2-Near :oving %ve$

1 ?5A<

2 6A5 A1 ?5$226 15< @<$<<9 A@ A9$9?? 11< 1<A$9?@ 1 2 119$<<A AA 11<$22

11<$9?

*iff' Topic' Time-series forecasting%%CS0' %nal tic Skills," ective' .,5-2

2) 3hat is the forecast for :a "ased on a &eighted moving average applied to the follo&ing past demand data and using the &eights' 5 2 Klargest &eight is for most recent data)4

Iov$ *ec$ Jan$ Fe"$ :ar$ %pril2? 29 5< 5 5? 52

%ns&er' 75 Q 275? Q 5752 O @5 Q 151 Q 1? O 2A?G 2A?/A O 55$1*iff' 1Topic' Time-series forecasting%%CS0' %nal tic Skills," ective' .,5-2

1-12@Cop right ; <11 =earson ducation >nc$

Page 139: OM Exam 1 Chap 1-5

8/13/2019 OM Exam 1 Chap 1-5

http://slidepdf.com/reader/full/om-exam-1-chap-1-5 139/187

Page 140: OM Exam 1 Chap 1-5

8/13/2019 OM Exam 1 Chap 1-5

http://slidepdf.com/reader/full/om-exam-1-chap-1-5 140/187

6) The last four &eekl values of sales &ere @< 1<< 1<6 and A< units$ The last four forecastsKfor the same four &eeks) &ere 9< @< A6 and ?6 units$ Calculate :%* :S and :%= forthese four &eeks$

Sales Forecast rror rror s#uared =ct$ error

@< 9< < 5<< $ 61<< @< < 5<< $ <1<6 A6 1< 1<< $<A6A< ?6 16 6 $19?

%ns&er' :%* O 96/5 O 19$ 6G :S O 11 6/5 O @1$ 6G :%= O <$?1 /5 O $1?@ or 1?$@*iff' Topic' Time series forecasting' :easuring forecast error %%CS0' %nal tic Skills," ective' .,5-5

9) % management anal st is using e7ponential smoothing to predict merchandise returns at anupscale "ranch of a department store chain$ +iven an actual num"er of returns of 165 items inthe most recent period completed a forecast of 1? items for that period and a smoothingconstant of <$2 &hat is the forecast for the ne7t period4 Ho& &ould the forecast "e changed ifthe smoothing constant &ere <$94 7plain the difference in terms of alpha and responsiveness$%ns&er' 199$9G 191$ The larger the smoothing constant in an e7ponentiall smoothedforecast the more responsive the forecast$*iff' 1Topic' Time-series forecasting%%CS0' %nal tic Skills," ective' .,5-2

?) The follo&ing trend pro ection is used to predict #uarterl demand' N O 6< - $6t &here t O 1in the first #uarter$ Seasonal K#uarterl ) indices are Duarter 1 O 1$6G Duarter O <$@G Duarter 2 O1$1G and Duarter 5 O <$9$ 3hat is the seasonall ad usted forecast for the ne7t four #uarters4%ns&er' Duarter =ro ection%d usted

1 5?$6 2?1$ 656 1A9

2 5 $6 99$?65 5< 155

*iff'

Topic' Time-series forecasting%%CS0' %nal tic Skills," ective' .,5-6

1-15<Cop right ; <11 =earson ducation >nc$

Page 141: OM Exam 1 Chap 1-5

8/13/2019 OM Exam 1 Chap 1-5

http://slidepdf.com/reader/full/om-exam-1-chap-1-5 141/187

@) JimBs department at a local department store has tracked the sales of a product over the last ten&eeks$ Forecast demand using e7ponential smoothing &ith an alpha of <$5 and an initialforecast of @$< for period 1$ Calculate the :%*$ 3hat do ou recommend4

=eriod *emand

1 522 95 296 99 2<? 2@ 9A 6

1< @

%ns&er'=eriod *emand Forecast rror %"solute

1 5 @$<<2 9$5< -2$5< 2$5<

2 9 6$<5 <$A9 <$A95 29 6$5 1<$6@ 1<$6@6 9 A$96 -2$96 2$969 2< @$1A 1$@1 1$@1? 2 @$A 2$<@ 2$<@@ 9 2<$16 -5$16 5$16

A 6 @$5A -2$5A 2$5A1< @ ?$<A <$A1 <$A1Total $95 2 $<2

%verage <$ A 2$690ias :%*

The tracking signal RSF /:%* O $95/2$69 O $?5 is lo&G therefore keep using the forecastingmethod$*iff' Topic' Time-series forecasting and monitoring and controlling forecasts%%CS0' %nal tic Skills

," ective' .,5-5

1-151Cop right ; <11 =earson ducation >nc$

Page 142: OM Exam 1 Chap 1-5

8/13/2019 OM Exam 1 Chap 1-5

http://slidepdf.com/reader/full/om-exam-1-chap-1-5 142/187

A) Favors *istri"ution Compan purchases small imported trinkets in "ulk packages them andsells them to retail stores$ The are conducting an inventor control stud of all their items$ Thefollo&ing data are for one such item &hich is not seasonal$

a$ (se trend pro ection to estimate the relationship "et&een time and sales Kstate the e#uation)$

"$ Calculate forecasts for the first four months of the ne7t ear$

1 2 5 6 9 ? @ A 1< 11 1:onth Jan Fe" :ar %pr :a Jun Jul %ug Sep ,ct Iov *ecSales 61 66 65 6? 6< 9@ 99 6A 9? 9A ?6 ?2

%ns&er' The trend pro ection e#uation is N O 5@$2 Q $1<6 T$ The ne7t four months areforecast to "e ?6$9@ ??$?A ?A$@A and @ $<<$*iff' Topic' Time-series forecasting%%CS0' %nal tic Skills

," ective' .,5-9

1-15Cop right ; <11 =earson ducation >nc$

Page 143: OM Exam 1 Chap 1-5

8/13/2019 OM Exam 1 Chap 1-5

http://slidepdf.com/reader/full/om-exam-1-chap-1-5 143/187

1<) (se e7ponential smoothing &ith trend ad ustment to forecast deliveries for period 1<$ .etalpha O <$5 "eta O <$ and let the initial trend value "e 5 and the initial forecast "e <<$

=eriod%ctual

*emand

1 <<12 1556 299 1? 5<@ 55A 6<

1< 99

%ns&er'%ctual Forecast Trend F>T

1 << <<$<< 5$<<1 < $5< 2$9@ <9$<@

2 15 <@$56 5$16 1 $9<5 12$19 5$ ? 1?$526 29 1A$ 9 5$92 2$@A9 1 @$?2 6$9< 25$22? 5< A$<< 5$62 22$62@ 55 29$1 6$<6 51$1?A 6< 5 $2< 6$ @ 5?$6@

1< 99 5@$66 6$5? 65$<

*iff' Topic' Time-series forecasting%%CS0' %nal tic Skills," ective' .,5-2

1-152Cop right ; <11 =earson ducation >nc$

Page 144: OM Exam 1 Chap 1-5

8/13/2019 OM Exam 1 Chap 1-5

http://slidepdf.com/reader/full/om-exam-1-chap-1-5 144/187

11) % small famil -o&ned restaurant uses a seven-da moving average model to determinemanpo&er re#uirements$ These forecasts need to "e seasonali!ed "ecause each da of the &eekhas its o&n demand pattern$ The seasonal indices for each da of the &eek are' :onda <$556GTuesda <$?A1G 3ednesda <$A ?G Thursda 1$<22G Frida 1$5 G Saturda 1$5?@G andSunda <$A<2$ %verage dail demand "ased on the most recent moving average is 1A5 patrons$

3hat is the seasonali!ed forecast for each da of ne7t &eek4%ns&er' The average value multiplied " each da Bs seasonal inde7$ :onda ' 1A5 7 $556 O @9GTuesda ' 1A5 7 $?A1 O 162G 3ednesda ' 1A5 7 $A ? O 1@<G Thursda ' 1A5 7 1$<22 O <<G Frida '1A5 7 1$5 O ?9G Saturda ' 1A5 7 1$5?@ O @?G and Sunda ' 1A5 7 $A<2 O 1?6$*iff' Topic' %ssociative forecasting methods' Regression and correlation anal sis%%CS0' %nal tic Skills," ective' .,5-6

1 ) % restaurant has tracked the num"er of meals served at lunch over the last four &eeks$ Thedata sho&s little in terms of trends "ut does displa su"stantial variation " da of the &eek$

(se the follo&ing information to determine the seasonal Kdail ) inde7 for this restaurant$3eek

*a 1 2 5Sunda 5< 26 2A 52:onda 65 66 61 6ATuesda 91 9< 96 953ednesda ? ?? ?@ 9AThursda @A @< @1 ?AFrida A1 A< AA A6Saturda @< @ @1 @2

%ns&er'*a >nde7

Sunda <$69 ?:onda <$?@66Tuesda <$@A92

3ednesda 1$<91@Thursda 1$1@<<

Frida 1$2555Saturda 1$19A

*iff' Topic' Time-series forecasting%%CS0' %nal tic Skills," ective' .,5-6

1-155Cop right ; <11 =earson ducation >nc$

Page 145: OM Exam 1 Chap 1-5

8/13/2019 OM Exam 1 Chap 1-5

http://slidepdf.com/reader/full/om-exam-1-chap-1-5 145/187

Page 146: OM Exam 1 Chap 1-5

8/13/2019 OM Exam 1 Chap 1-5

http://slidepdf.com/reader/full/om-exam-1-chap-1-5 146/187

19) :arie 0ain is the production manager at a compan that manufactures hot &ater heaters$:arie needs a demand forecast for the ne7t fe& ears to help decide &hether to add ne&

production capacit $ The compan Bs sales histor Kin thousands of units) is sho&n in the ta"le "elo&$ (se e7ponential smoothing &ith trend ad ustment to forecast demand for period 9$ Theinitial forecast for period 1 &as 11 unitsG the initial estimate of trend &as <$ The smoothing

constants are \ O $2 and ] O $2

=eriod %ctual1 1

162 195 196 1@9 <

%ns&er'

=eriod %ctual Forecast Trend F>T1 1 11$<< <$<<

16 11$2< <$<A 11$2A2 19 1 $5? <$51 1 $@A5 19 12$@ <$9A 15$66 1@ 15$A9 <$@2 16$?A9 < 19$56 1$<2 1?$5@

*iff' Topic' Time-series forecasting%%CS0' %nal tic Skills," ective' .,5-2

1-159Cop right ; <11 =earson ducation >nc$

Page 147: OM Exam 1 Chap 1-5

8/13/2019 OM Exam 1 Chap 1-5

http://slidepdf.com/reader/full/om-exam-1-chap-1-5 147/187

1?) The #uarterl sales for specific educational soft&are over the past three ears are given in thefollo&ing ta"le$ Compute the four seasonal factors$

N %R 1 N %R N %R 2Duarter 1 1?1< 1@ < 1@2<

Duarter A9< A1< 1<A<Duarter 2 ? < @5< A<<Duarter 5 52< << 6A<

%ns&er'%vg$ Sea$ Fact$

Duarter 1 1?@9$9? <$@A22Duarter A@9$9? <$5A22Duarter 2 @ <$<< 1$51<<Duarter 5 5<9$9? 1$ <22+rand %verage <<<$<<

*iff' Topic' Time-series forecasting%%CS0' %nal tic Skills," ective' .,5-6

1@) %n innovative restaurateur o&ns and operates a do!en 8(ltimate .o&-Car"8 restaurants innorthern %rkansas$ His signature item is a cheese-encrusted "eef medallion &rapped in lettuce$Sales K in millions of dollars) is related to =rofits KN in hundreds of thousands of dollars) "the regression e#uation N O @$ 1 Q <$?9 $ 3hat is our forecast of profit for a store &ith salesof L5< million4 L6< million4

%ns&er' Students must recogni!e that sales is the independent varia"le and profits is dependentGthe pro"lem is not a time series$ % store &ith L5< million in sales' 5< 7 <$?9 O 2<$5G 2<$5 Q @$ 1O 2@$91 or L2 @91 <<< in profitG L6< million in sales is estimated to profit 59$ 1 or L5 9 1 <<<$*iff' Topic' %ssociative forecasting methods' Regression and correlation anal sis%%CS0' %nal tic Skills," ective' .,5-9

1-15?Cop right ; <11 =earson ducation >nc$

Page 148: OM Exam 1 Chap 1-5

8/13/2019 OM Exam 1 Chap 1-5

http://slidepdf.com/reader/full/om-exam-1-chap-1-5 148/187

1A) %rnold Tofu o&ns and operates a chain of 1 vegeta"le protein 8ham"urger8 restaurants innorthern .ouisiana$ Sales figures and profits for the stores are in the ta"le "elo&$ Sales are givenin millions of dollarsG profits are in hundreds of thousands of dollars$ Calculate a regression linefor the data$ 3hat is our forecast of profit for a store &ith sales of L 5 million4 L2< million4

Store =rofits Sales1 15 911 2

2 16 65 19 66 5 169 @ 1@? 1?@ 1 1A 9 16

1< 52 <11 25 151 A 6

%ns&er' Students must recogni!e that 8sales8 is the independent varia"le and profits isdependent$ Store num"er is not a varia"le and the pro"lem is not a time series$ The regressione#uation is N O 6$A29 Q 1$5 1 KN O profit O sales)$ % store &ith L 5 million in sales isestimated to profit 5<$<5 or L5 <<5 <<<G L2< million in sales should ield 5@$699 or L5 @69 9<<in profit$*iff' Topic' %ssociative forecasting methods' Regression and correlation anal sis," ective' .,5-9

1-15@Cop right ; <11 =earson ducation >nc$

Page 149: OM Exam 1 Chap 1-5

8/13/2019 OM Exam 1 Chap 1-5

http://slidepdf.com/reader/full/om-exam-1-chap-1-5 149/187

<) The department manager using a com"ination of methods has forecast sales of toasters at alocal department store$ Calculate the :%* for the managerBs forecast$ Compare the managerBsforecast against a naive forecast$ 3hich is "etter4

:onth (nit Sales

:anagerBs

ForecastJanuar 6Fe"ruar 91

:arch ?2%pril ?A:a 99June 61Jul 5? 6<

%ugust 55 66Septem"er 2< 6

,cto"er 66 5 Iovem"er ?5 9<*ecem"er 1 6 ?6

%ns&er':onth %ctual :anagerBs %"s$ rror Iaive %"s$ rror Januar 6Fe"ruar 91:arch ?2%pril ?A:a 99June 61Jul 5? 6< 2 61 5%ugust 55 66 11 5? 2Septem"er 2< 6 55 15,cto"er 66 5 12 2< 6

Iovem"er ?5 9< 15 66 1A*ecem"er 1 6 ?6 6< ?5 61

The managerBs forecast has a :%* of 1@$@2 &hile the naive is 1A$22$ Therefore the managerBsforecast is slightl "etter than the naive$*iff'

Topic' :onitoring and controlling forecasts%%CS0' %nal tic Skills," ective' .,5-5

1-15ACop right ; <11 =earson ducation >nc$

Page 150: OM Exam 1 Chap 1-5

8/13/2019 OM Exam 1 Chap 1-5

http://slidepdf.com/reader/full/om-exam-1-chap-1-5 150/187

1) The last seven &eeks of demand at a ne& car dealer are sho&n "elo&$ (se a three-period

&eighted-moving average to determine a forecast for the @ th &eek using &eights of 1 and 2$Calculate the :%* for this forecast$ 3hat does the :%* indicate4

3eek Sales

1 62<

2 ?5 216 ?9 A? 2<

%ns&er'3eek Sales 23:% [error[

1 62<

2 ?5 21 @ 26 ? 2< 29 A @ 1? 2< A 1@ A

:%* O @/5 O %n :%* of means that the forecasting techni#ue used &as t picall off " units each period$*iff' Topic' Time-series forecasting moving averages and measuring forecast error

," ective' .,5-5

1-16<Cop right ; <11 =earson ducation >nc$

Page 151: OM Exam 1 Chap 1-5

8/13/2019 OM Exam 1 Chap 1-5

http://slidepdf.com/reader/full/om-exam-1-chap-1-5 151/187

Operations Management, 10e (Heizer/Render)Chapter . esi$n o 'oods and ervices

True/False

1) Regal :arineBs attempts to keep in touch &ith customers and respond to the marketplace aremade impossi"le "ecause consumer tastes change and maritime engineering improves$%ns&er' F%.S*iff' 1Topic' +oods and services selection," ective' no .,

) % product strateg ma focus on differentiation lo& cost or rapid response$%ns&er' TR(*iff' 1Topic' +oods and services selection

," ective' .,6-12) The o" ective of the product decision is to develop and implement a product strateg thatmeets the demands of the marketplace &ith a competitive advantage$%ns&er' TR(*iff' Topic' +oods and services selection," ective' .,6-1

5) =olitical/legal change and economic change are "oth factors influencing market opportunitiesfor ne& products$%ns&er' TR(*iff' 1Topic' +enerating ne& products," ective' .,6-1

6) The four phases of the product life c cle are incu"ation introduction gro&th and decline$%ns&er' F%.S*iff' Topic' +oods and services selection," ective' .,6-1

9) >n the maturit stage of the product life c cle operations managers &ill "e concerned &ithadding capacit or enhancing e7isting capacit to accommodate the increase in product demand$%ns&er' F%.S*iff' Topic' +oods and services selection," ective' .,6-1

1-161Cop right ; <11 =earson ducation >nc$

Page 152: OM Exam 1 Chap 1-5

8/13/2019 OM Exam 1 Chap 1-5

http://slidepdf.com/reader/full/om-exam-1-chap-1-5 152/187

?) Relativel fe& ne& product ideas perhaps onl 1 in 6< "ecome successfull marketed products$%ns&er' TR(*iff' Topic' +enerating ne& products

," ective' .,6-1@) 2:Bs goal is to produce 2< of its profit from products introduced in the last 5 ears$%ns&er' TR(*iff' 2Topic' +oods and services selection," ective' .,6-1

A) Computer-aided design KC%*) refers to the use of speciali!ed computer programs to directand control manufacturing e#uipment$%ns&er' F%.S*iff' Topic' >ssues for product design%%CS0' (se of >T," ective' .,6-

1<) Eirtual realit technolog can improve designs less e7pensivel than the use of ph sicalmodels or protot pes$%ns&er' TR(*iff' Topic' >ssues for product design%%CS0' (se of >T," ective' .,6-

11) Dualit function deplo ment refers to first determining &hat &ill satisf the customer andsecond translating the customersB desires into a target design$%ns&er' TR(*iff' Topic' =roduct development," ective' .,6-2

1 ) Ro"ust design is a method that ensures that small variation in production or assem"l doesnot adversel affect the product$%ns&er' TR(*iff' Topic' >ssues for product design," ective' .,6-6

1-16Cop right ; <11 =earson ducation >nc$

Page 153: OM Exam 1 Chap 1-5

8/13/2019 OM Exam 1 Chap 1-5

http://slidepdf.com/reader/full/om-exam-1-chap-1-5 153/187

12) :odular design e7ists onl in tangi"le productsG it makes no sense in services$%ns&er' F%.S*iff' Topic' >ssues for product design," ective' .,6-6

15) ,ne environmentall friendl approach to product design is to use lighter components$%ns&er' TR(*iff' Topic' thics environmentall -friendl designs and sustaina"ilit%%CS0' thical Reasoning," ective' no .,

16) %n operations managerBs most ethical activit is to enhance productivit &hile deliveringdesired goods and services$ (nfortunatel this activit is not environmentall sound$%ns&er' F%.S*iff' Topic' thics environmentall -friendl designs and sustaina"ilit%%CS0' thical Reasoning," ective' no .,

19) T&o issuesMvie&ing a product in terms of its impact on the entire econom and consideringthe life c cle of a productMcom"ine to increase the likelihood of ethical decisions " managers$%ns&er' TR(*iff' Topic' thics environmentall -friendl designs and sustaina"ilit%%CS0' thical Reasoning," ective' no .,

1?) Rapidl developing products and moving them to the market is part of time-"asedcompetition$%ns&er' TR(*iff' Topic' Time-"ased competition," ective' .,6-5

1@) The enhancement of e7isting products is an e7ternal product development strateg $%ns&er' F%.S*iff' 1Topic' Time-"ased competition," ective' .,6-5

1-162Cop right ; <11 =earson ducation >nc$

Page 154: OM Exam 1 Chap 1-5

8/13/2019 OM Exam 1 Chap 1-5

http://slidepdf.com/reader/full/om-exam-1-chap-1-5 154/187

1A) The 8make-or-"u 8 decision distinguishes "et&een &hat an organi!ation chooses to produceand &hat it chooses to purchase from suppliers$%ns&er' TR(*iff' Topic' *efining the product

," ective' .,6-6

<) +roup technolog ena"les the grouping of parts into families "ased on similar processingre#uirements$%ns&er' TR(*iff' 1Topic' *efining the product," ective' .,6-6

1) % &ork order is a listing of the components their description and the #uantit of eachre#uired to make one unit of the product$

%ns&er' F%.S*iff' Topic' *ocuments for production," ective' .,6-9

) %n assem"l dra&ing lists the operations necessar to produce the component$%ns&er' F%.S*iff' Topic' *ocuments for production," ective' .,6-9

2) %n assem"l chart sho&s an e7ploded vie& of the product usuall a three-dimensional orisometric dra&ing$%ns&er' F%.S*iff' Topic' *ocuments for production," ective' .,6-9

5) The customer ma participate in the design of and in the deliver of services$%ns&er' TR(*iff' Topic' Service design," ective' .,6-?

6) The moment-of-truth is the crucial moment "et&een the service provider and the customerthat e7emplifies enhances or detracts from the customerBs e7pectation$%ns&er' TR(*iff' Topic' Service design," ective' .,6-?

9) The e7pected value of each course of action in a decision tree is determined " starting at the1-165

Cop right ; <11 =earson ducation >nc$

Page 155: OM Exam 1 Chap 1-5

8/13/2019 OM Exam 1 Chap 1-5

http://slidepdf.com/reader/full/om-exam-1-chap-1-5 155/187

Page 156: OM Exam 1 Chap 1-5

8/13/2019 OM Exam 1 Chap 1-5

http://slidepdf.com/reader/full/om-exam-1-chap-1-5 156/187

5) 3hich of the follo&ing statements is not true4%) Eirtuall all of HondaBs sales Kautos motorc cles generators la&n mo&ers) are "ased on itsoutstanding engine technolog $0) >ntel focuses on microprocessors$C) :ichelin focuses on tires$

*) Firms such as 2: esta"lish goals for profita"ilit from ne& products$) *ell Computers provides fast deliver to customers "ut in return customers ma onl selectfrom a limited choice of hard&are configurations$%ns&er' *iff' Topic' +oods and services selection," ective' .,6-1

6) ,perations managers must "e a"le to anticipate changes in &hich of the follo&ing4%) product mi70) product opportunities

C) the products themselves*) product volume) all of the a"ove%ns&er' *iff' Topic' +enerating ne& products," ective' .,6-1

9) >n &hich stage of the product life c cle should product strateg focus on processmodifications as the product is "eing ^fine-tuned_ for the market4%) introduction0) gro&thC) maturit*) decline) none of the a"ove%ns&er' %*iff' Topic' +oods and services selection," ective' .,6-1

?) 3hich of the follo&ing &ould likel cause a change in market opportunities "ased upon levelsof income and &ealth4%) economic change0) sociological and demographic changeC) technological change*) political change) legal change%ns&er' %*iff' Topic' +enerating ne& products," ective' .,6-1@) % productBs life c cle is divided into four stages &hich are

1-169Cop right ; <11 =earson ducation >nc$

Page 157: OM Exam 1 Chap 1-5

8/13/2019 OM Exam 1 Chap 1-5

http://slidepdf.com/reader/full/om-exam-1-chap-1-5 157/187

Page 158: OM Exam 1 Chap 1-5

8/13/2019 OM Exam 1 Chap 1-5

http://slidepdf.com/reader/full/om-exam-1-chap-1-5 158/187

1 ) >n &hich stage of the product life c cle should product strateg focus on improved costcontrol4%) introduction0) gro&thC) maturit

*) saturation) inflation%ns&er' C*iff' Topic' +oods and services selection," ective' .,6-1

12) is used to rank a compan Bs products to determine &hich products represent the "est use of the firmBs resources or perhaps to determine &hich products are to "e eliminated$%) Ealue anal sis0) Ealue engineering

C) Financial anal sis*) =roduct-" -value anal sis) =roduct cost ustification%ns&er' **iff' 1Topic' +oods and services selection," ective' .,6-1

15) 3hich of the follo&ing represents an opportunit for generating a ne& product4%) understanding the customer 0) demographic change such as decreasing famil si!eC) changes in professional standards*) economic change such as rising household incomes) %ll of the a"ove are such opportunities$%ns&er' *iff' Topic' +enerating ne& products," ective' .,6-

16) 3hat of sales from ne& products is indicative of industr leaders4%) 6<0) "elo& 6C) 6 to 26*) a"ove 9<) 26 to 56%ns&er' %*iff' Topic' +enerating ne& products," ective' .,6-

1-16@Cop right ; <11 =earson ducation >nc$

Page 159: OM Exam 1 Chap 1-5

8/13/2019 OM Exam 1 Chap 1-5

http://slidepdf.com/reader/full/om-exam-1-chap-1-5 159/187

19) 3hich of the follo&ing is true regarding value engineering4%) Ealue engineering occurs onl after the product is selected and designed$0) Ealue engineering is the same as value anal sis$C) Ealue engineering is oriented to&ard improvement of design$*) Ealue engineering occurs during production &hen it is clear the product is a success$

) Ealue engineering can save su"stantial amounts of product cost "ut #ualit suffers$%ns&er' C*iff' Topic' =roduct development," ective' .,6-2

1?) Reducing the comple7it of a product and improving a productBs maintaina"ilit for use areactivities of %) =roduct .ifec cle :anagement K=.:)0) product-" -value-anal sisC) manufactura"ilit and value engineering

*) organi!ing for product development) design for destruction K*F*)%ns&er' C*iff' Topic' =roduct development," ective' .,6-2

1@) Dualit function deplo ment KDF*)%) determines &hat &ill satisf the customer 0) translates customer desires into the target designC) is used earl in the design process*) is used to determine &here to deplo #ualit efforts) all of the a"ove%ns&er' *iff' Topic' =roduct development," ective' .,6-2

1A) % graphic techni#ue for defining the relationship "et&een customer desires and product/service is%) =roduct .ifec cle :anagement0) the House of DualitC) the moment of truth*) the assem"l dra&ing) the product development team%ns&er' 0*iff' Topic' =roduct development," ective' .,6-2

1-16ACop right ; <11 =earson ducation >nc$

Page 160: OM Exam 1 Chap 1-5

8/13/2019 OM Exam 1 Chap 1-5

http://slidepdf.com/reader/full/om-exam-1-chap-1-5 160/187

<) ,ne result of concurrent engineering in product design is%) speedier product development0) lo&er #ualitC) less customer demand*) higher costs

) all of the a"ove%ns&er' %*iff' Topic' =roduct development," ective' .,6-2

1) The use of product development teams in design and engineering activities is also kno&n as%) the Japanese approach0) concurrent engineeringC) DF**) the traditional ($S$ approach

) none of the a"ove%ns&er' 0*iff' Topic' =roduct development," ective' .,6-2

) 3hen developing the Trident Splash Cad"ur faced design pro"lems overcome "%) manufactura"ilit and value engineering0) DF*C) the House of Dualit*) C%*) decision trees%ns&er' %*iff' Topic' =roduct development," ective' .,6-2

2) The Japanese method of organi!ing for product design features%) teams0) product managers Kchampions)C) distinct departments &ith assigned tasks*) a single organi!ation &ithout su"division or individual teams) none of the a"ove%ns&er' **iff' Topic' =roduct development," ective' .,6-2

1-19<Cop right ; <11 =earson ducation >nc$

Page 161: OM Exam 1 Chap 1-5

8/13/2019 OM Exam 1 Chap 1-5

http://slidepdf.com/reader/full/om-exam-1-chap-1-5 161/187

5) :anufactura"ilit and Ealue ngineering has &hich of the follo&ing "enefits4>$ Reduced comple7it of the product>>$ Reduction of environmental impact>>>$ %dditional standardi!ation of components>E$ Ro"ust design

E$ >mproved o" design and safet%) > >>> E0) >> >EC) > >>> >E*) >E) > >> >>> >E E%ns&er' *iff' Topic' =roduct development," ective' .,6-2

6) 3hich of the follo&ing is true regarding computer-aided design4%) >t is too e7pensive to use in most manufacturing and design settings$0) >t is an old technologC) >t results in longer development c cles for virtuall all products$*) >t is the use of computers to interactivel design products and prepare engineeringdocumentation$) %ll of the a"ove are true$%ns&er' **iff' Topic' >ssues for product design%%CS0' (se of >T," ective' .,6-6

9) 3hich of the follo&ing is true concerning C%*4%) %ccurate information flo&s to other departments$0) :ost product costs are determined at the design stage$C) *esign options are easier to revie& "efore final commitments are made$*) Eirtuall all products have their development c cle shortened$) %ll of the a"ove are true$%ns&er' *iff' Topic' >ssues for product design%%CS0' (se of >T," ective' .,6-6

1-191Cop right ; <11 =earson ducation >nc$

Page 162: OM Exam 1 Chap 1-5

8/13/2019 OM Exam 1 Chap 1-5

http://slidepdf.com/reader/full/om-exam-1-chap-1-5 162/187

Page 163: OM Exam 1 Chap 1-5

8/13/2019 OM Exam 1 Chap 1-5

http://slidepdf.com/reader/full/om-exam-1-chap-1-5 163/187

%) &hen the product is selected and designed0) during the initial stages of production &hen something needs to "e done to assure productsuccessC) &hen the product is first conceived*) during the production process &hen it is clear the ne& product is a success

) &hen the product cost is ver lo&%ns&er' **iff' Topic' >ssues for product design," ective' .,6-6

2 ) 3hich of the follo&ing makes products that are friendlier to the environment4%) using less materials0) more rec cled materialsC) using less energ*) using less harmful ingredients

) all of the a"ove%ns&er' *iff' Topic' thics environmentall -friendl designs and sustaina"ilit%%CS0' thical Reasoning," ective' no .,

22) ,ne method used to determine a productBs total environmental impact from conversion ofinputs through production and to disposal is%) DF*0) *esign for manufactura"ilitC) nvironmental Scanning*) .ife C cle %ssessment) C%*%ns&er' **iff' Topic' thics environmentall -friendl designs and sustaina"ilit," ective' .,6-2

25) =roducts are more 8environmentall friendl 8 &hen the are made%) using cheaper materials0) using less energC) according to ,SH% standards*) &here environmental regulations are la7) more difficult to disassem"le%ns&er' 0*iff' 1Topic' thics environmentall -friendl designs and sustaina"ilit%%CS0' thical Reasoning," ective' no .,26) %n operations managerBs ethical responsi"ilities%) are limited to the production phase of product life c cle

1-192Cop right ; <11 =earson ducation >nc$

Page 164: OM Exam 1 Chap 1-5

8/13/2019 OM Exam 1 Chap 1-5

http://slidepdf.com/reader/full/om-exam-1-chap-1-5 164/187

0) are limited to follo&ing pu"lished la&s and regulationsC) e7tend from design to production to final destruction/disposition of a product*) do not include "eing efficient &ith the firmBs resources) need not consider a product designBs impact on the entire econom%ns&er' C

*iff' Topic' thics environmentall -friendl designs and sustaina"ilit%%CS0' thical Reasoning," ective' no .,

29) The change in cell-phone model life c cles from ears to under 9 months is the result of %) time-"ased competition0) design for manufactura"ilitC) life c cle assessment*) value anal sis) all of the a"ove

%ns&er' %*iff' Topic' Time-"ased competition," ective' .,6-5

2?) 3hich of the follo&ing is an e7ample of an e7ternal product development strateg 4%) ne& internall developed products0) enhancements to e7isting productsC) alliances*) migrations of e7isting products) %ll of the a"ove are e7amples of internal product development strateg $%ns&er' C*iff' Topic' Time-"ased competition," ective' .,6-5

2@) 3hich of the follo&ing product development strategies has the highest risk4%) ac#uiring the developer 0) alliancesC) oint ventures*) ne& internall developed products) purchasing technolog%ns&er' **iff' Topic' Time-"ased competition," ective' .,6-5

1-195Cop right ; <11 =earson ducation >nc$

Page 165: OM Exam 1 Chap 1-5

8/13/2019 OM Exam 1 Chap 1-5

http://slidepdf.com/reader/full/om-exam-1-chap-1-5 165/187

2A) %n engineering dra&ing sho&s the%) dimensions tolerances materials and finishes of a component0) dimensions tolerances cost and sales or use volume of a componentC) materials finishes machining operations and dimensions of a component*) cost materials tolerances and lead-time for a component

) cost dimensions and machining operations for a component%ns&er' %*iff' Topic' *efining the product," ective' .,6-6

5<) The dimensions tolerances materials and finishes of a component are t picall sho&n onaKn)%) assem"l chart0) engineering dra&ingC) "ill of material

*) assem"l dra&ing) route sheet%ns&er' 0*iff' Topic' *efining the product," ective' .,6-6

51) 3hich of the follo&ing does not result from the effective use of group technolog 4%) reduced tooling setup time0) "etter electronic communication "et&een team mem"ersC) reduced ra& materials and purchases*) simplified production planning and control) improved la out routing and machine loading%ns&er' 0*iff' Topic' *efining the product," ective' .,6-6

5 ) 3hich of the follo&ing t picall sho&s the components their description and the #uantitof each re#uired to make one unit of a product4%) an engineering dra&ing0) an assem"l dra&ingC) a "ill of material*) an assem"l chart) a route sheet%ns&er' C*iff' Topic' *efining the product," ective' .,6-9

1-196Cop right ; <11 =earson ducation >nc$

Page 166: OM Exam 1 Chap 1-5

8/13/2019 OM Exam 1 Chap 1-5

http://slidepdf.com/reader/full/om-exam-1-chap-1-5 166/187

Page 167: OM Exam 1 Chap 1-5

8/13/2019 OM Exam 1 Chap 1-5

http://slidepdf.com/reader/full/om-exam-1-chap-1-5 167/187

5?) %n assem"l dra&ing%) sho&s in schematic form ho& the product is assem"led0) sho&s an e7ploded vie& of the productC) lists the operations including assem"l and inspection necessar to produce the component&ith the material specified in the "ill of material

*) provides detailed instructions on ho& to perform a given task ) descri"es the dimensions and finish of each component%ns&er' 0*iff' 1Topic' *ocuments for production," ective' .,6-9

5@) %n assem"l chart%) sho&s graphicall ho& the product is assem"led0) sho&s an e7ploded vie& of the productC) lists the operations including assem"l and inspection necessar to produce the component

&ith the material specified in the "ill of material*) provides detailed instructions on ho& to perform a given task ) descri"es the dimensions and finish of each component%ns&er' %*iff' 1Topic' *ocuments for production," ective' .,6-9

5A) % process sheet is a t pe of %) assem"l dra&ing0) assem"l chartC) route sheet*) &ork order ) "ill of material%ns&er' C*iff' Topic' *ocuments for production," ective' .,6-9

6<) 3hich of the follo&ing documents lists the operations Kincluding assem"l and inspection)necessar to produce the component &ith the material specified in the "ill of material4%) an engineering dra&ing0) an assem"l dra&ingC) a route sheet*) an assem"l chart) an operations chart%ns&er' C*iff' Topic' *ocuments for production," ective' .,6-9

61) % restaurant kitchen contains a &all poster that sho&s for each sand&ich on the menu a1-19?

Cop right ; <11 =earson ducation >nc$

Page 168: OM Exam 1 Chap 1-5

8/13/2019 OM Exam 1 Chap 1-5

http://slidepdf.com/reader/full/om-exam-1-chap-1-5 168/187

sketch of the ingredients and ho& the are arranged to make the sand&ich$ This is an e7ample ofaKn)%) assem"l dra&ing0) route sheetC) "ill of material

*) &ork order ) virtual technolog%ns&er' %*iff' Topic' *ocuments for production," ective' .,6-9

6 ) The document for production that gives the instruction to make a given #uantit of a particular item usuall to a given schedule is%) the &ork order 0) the route sheet

C) the "ill of information*) the assem"l chart) the value anal sis%ns&er' %*iff' Topic' *ocuments for production," ective' .,6-9

62) 3hich of the follo&ing is not a service design techni#ue used to reduce cost4%) increasing customer interaction0) dela ing customi!ationC) automation*) modulari!ing the product) reducing customer interaction%ns&er' %*iff' Topic' Service design," ective' .,6-?

1-19@Cop right ; <11 =earson ducation >nc$

Page 169: OM Exam 1 Chap 1-5

8/13/2019 OM Exam 1 Chap 1-5

http://slidepdf.com/reader/full/om-exam-1-chap-1-5 169/187

65) 3hich of the follo&ing e7amples involves customer participation in the design of theservice4%) investing in a mutual fund0) getting a chest 7-ra at the hospitalC) providing the stock"roker &ith the desired distri"ution of the portfolio

*) seeing a movie at the theater ) eating at a fast-food restaurant%ns&er' C*iff' Topic' Service design%%CS0' Communication," ective' .,6-?

66) :odern %T: machines are an automated e7ample of a service design that%) reduces customer interaction0) modulari!es the service

C) dela s service customi!ation*) has no moment of truth) has insufficient #ualit function deplo ment%ns&er' %*iff' Topic' Service design," ective' .,6-?

69) For a full-time college student &hich of the follo&ing moments of truth e7emplifies thecustomerBs standard e7pectations4%) Nour advisor made ou &ait even though ou had an appointment$0) Nou had to visit more than once to reach our academic advisor$C) Nour advisor &as competent helpful and understanding$*) Nour advisor failed to keep her appointment &ith ou$) Nour advisor offered to &ork &ith ou after regular &ork hours$%ns&er' C*iff' 2Topic' Service design," ective' .,6-?

1-19ACop right ; <11 =earson ducation >nc$

Page 170: OM Exam 1 Chap 1-5

8/13/2019 OM Exam 1 Chap 1-5

http://slidepdf.com/reader/full/om-exam-1-chap-1-5 170/187

6?) 3hich service design techni#ueKs) &ould ordinaril not "e appropriate for full-servicerestaurant meals4%) modulari!ing the product0) customi!ation as late in the process as possi"leC) reducing customer interaction

*) the moment of truth) %ll of the a"ove are appropriate$%ns&er' C*iff' Topic' Service design," ective' .,6-?

6@) 3hich of the follo&ing scenarios illustrates a moment that exceeds the customerBse7pectations4%) an e7press mail service that guarantees overnight deliver0) a flight attendant that responds shortl after "eing calledC) a hairdresser that cuts our hair at the right length*) a professor that contacts people in several companies to find ou a o") a "ank that sends ou monthl account statements%ns&er' **iff' 2Topic' Service design," ective' .,6-?

6A) 3hich of the follo&ing scenarios illustrates a moment that "eets the customerBse7pectations4%) a sales clerk that called ou " our name on our second visit to the store0) an e7press mail service that guarantees same da deliverC) a flight attendant that responds shortl after "eing called*) a professor that contacts people in several companies to find ou a o") a sales clerk at the clothing store that asked ou to &ait for a minute as soon as a &ealth -looking customer entered the store%ns&er' C*iff' 2Topic' Service design," ective' .,6-?

9<) The role of decision trees in product design is%) to rank products in descending order of their dollar contri"ution to the firm0) to "etter understand the customersB &antsC) to calculate the e7pected value of each course of action*) to calculate the value of #ualit function deplo ment) to calculate the value of the moment of truth%ns&er' C*iff' 1Topic' %pplication of decision trees to product design," ective' .,6-@91) =a offs alternatives and e7pected monetar values are terms associated &ith

1-1?<Cop right ; <11 =earson ducation >nc$

Page 171: OM Exam 1 Chap 1-5

8/13/2019 OM Exam 1 Chap 1-5

http://slidepdf.com/reader/full/om-exam-1-chap-1-5 171/187

%) virtual realit0) =roduct .ifec cle :anagementC) Dualit Function *eplo ment*) decision trees) make-or-"u anal sis

%ns&er' **iff' 1Topic' %pplication of decision trees to product design," ective' .,6-@

Short %ns&er

1) Regal :arineBs use of has reduced product development time and reduced pro"lems &ith tooling and production$%ns&er' C%* or Computer-aided *esign*iff' Topic' +lo"al compan profile%%CS0' (se of >T," ective' no .,

) >n the phase of the product life c cle the product design has "egun to sta"ili!e$%ns&er' gro&th*iff' Topic' +oods and services selection," ective' .,6-1

2) lists products in descending order of the individual dollar contri"ution to thefirm$%ns&er' =roduct-" -value anal sis*iff' Topic' +oods and services selection," ective' .,6-1

5) is a process for determining customer re#uirements and translating them intoattri"utes that each functional area can understand and act upon$%ns&er' Dualit Function *eplo ment or DF**iff' Topic' =roduct development," ective' .,6-2

6) The is a part of the #ualit function deplo ment process that utili!es a planningmatri7 to relate customer 8&ants8 to 8ho&8 the firm is going to meet those 8&ants$8%ns&er' House of Dualit*iff' Topic' =roduct development," ective' .,6-29) provides a format allo&ing the electronic transmittal of three-dimensional data$%ns&er' Standard for the e7change of product data or ST =

1-1?1Cop right ; <11 =earson ducation >nc$

Page 172: OM Exam 1 Chap 1-5

8/13/2019 OM Exam 1 Chap 1-5

http://slidepdf.com/reader/full/om-exam-1-chap-1-5 172/187

*iff' Topic' +enerating ne& products%%CS0' (se of >T," ective' .,6-6

?) The use of information technolog to control machiner is called $%ns&er' computer-aided manufacturing or C%:*iff' Topic' +enerating ne& products%%CS0' (se of >T," ective' .,6-6

@) >f a design can "e produced to re#uirements even &hen the production process hasunfavora"le conditions the design is said to "e $%ns&er' ro"ust*iff' Topic' >ssues for product design," ective' .,6-6

A) =roducts or services designed in easil segmented components are kno&n as $%ns&er' modular designs*iff' Topic' >ssues for product design," ective' .,6-6

1<) revie&s successful products for improvement during the production process$%ns&er' Ealue anal sis*iff' Topic' >ssues for product design," ective' .,6-6

11) Sensitivit to a &ide variet of environmental issues in production processes is referred to as $%ns&er' green manufacturing*iff' Topic' >ssues for product design%%CS0' thical Reasoning," ective' .,6-6

1 ) To vie& product design from a 8s stems8 perspective managers must vie& a product interms of its impact on $%ns&er' the entire econom*iff' 1Topic' >ssues for product design," ective' .,6-612) 0oeingBs use of epo7 composites and titanium graphite laminate represent the ethical andenvironmentall friendl concept of $%ns&er' using lighter components

1-1?Cop right ; <11 =earson ducation >nc$

Page 173: OM Exam 1 Chap 1-5

8/13/2019 OM Exam 1 Chap 1-5

http://slidepdf.com/reader/full/om-exam-1-chap-1-5 173/187

*iff' 1Topic' >ssues for product design%%CS0' thical Reasoning," ective' .,6-6

15) Rapidl developing products and moving them to the market is referred to as $%ns&er' time-"ased competition*iff' 1Topic' Time-"ased competition," ective' .,6-5

16) % dra&ing that sho&s the dimensions tolerances materials and finishes of a component isaKn) $%ns&er' engineering dra&ing*iff' 1Topic' *ocuments for production," ective' .,6-9

19) % listing of the components their description and the #uantit of each re#uired to make oneunit of product is the $%ns&er' "ill of materials*iff' Topic' *ocuments for production," ective' .,6-9

1?) 0lack Z *eckerBs hand-po&ered tools and He&lett-=ackardBs printer "usiness are e7amplesof using enhancements and migrations of e7isting products to "uild on $%ns&er' product platforms*iff' Topic' Time-"ased competition," ective' .,6-5

1@) %n e7ploded vie& of the product is aKn) $%ns&er' assem"l dra&ing*iff' Topic' *ocuments for production," ective' .,6-9

1A) The is the crucial moment "et&een the service provider and the customer thate7emplifies enhances or detracts from the customerBs e7pectations$%ns&er' moment-of-truth*iff' Topic' Service design," ective' .,6-?<) >n anal !ing product design decisions decision trees determine the of eachcourse of action$%ns&er' e7pected value or :E*iff'

1-1?2Cop right ; <11 =earson ducation >nc$

Page 174: OM Exam 1 Chap 1-5

8/13/2019 OM Exam 1 Chap 1-5

http://slidepdf.com/reader/full/om-exam-1-chap-1-5 174/187

Page 175: OM Exam 1 Chap 1-5

8/13/2019 OM Exam 1 Chap 1-5

http://slidepdf.com/reader/full/om-exam-1-chap-1-5 175/187

6) >dentif the general "enefits derived from C%*$%ns&er' =roduct #ualit K"etter adherence to standards)G shorter design timeG production costreductions Kfrom "etter design)G data"ase availa"ilit Kfor ne& products)G and ne& range ofcapa"ilities Keas programming of CIC machines)$*iff'

Topic' +enerating ne& products%%CS0' (se of >T," ective' .,6-6

9) >s it possi"le for a productBs life c cle stage to affect its product strateg 4 >n particulardescri"e ho& one product in gro&th and another in maturit might have different productstrategies$%ns&er' There is no reason for the strateg to "e static through the life c cle stages$,rgani!ations often treat ne& products differentl than older ones in terms of support forchanges aggressiveness in pursuit of market etc$ >n particular gro&th is associated &ithsta"ili!ation of design and &ith ensuring that sufficient capacit e7ists$ :aturit is a time for

high-volume operations and cost control$*iff' Topic' +oods and services selection," ective' .,6-1

?) *escri"e the "enefits associated &ith value engineering$%ns&er' 0enefits include immediate cost reductionsG reduced product comple7it G additionalstandardi!ation of componentsG improvement of functional aspects of the productsG improved o"design and o" safet G improved maintaina"ilit Kservicea"ilit ) of the productG and ro"ustdesign$*iff' Topic' =roduct development," ective' .,6-

@) =rovide some e7amples of recent product changes i$e$ ne& products that are replacing olderones$%ns&er' %ns&ers &ill var $ The list in the te7t includes' TE to H*TE radio to satellite radiocoffee shops to Star"ucks lifest le coffee traveling circuses to Cir#ue du Soleil land lines to cell

phones cell phone to 0lack"err 3alkman to i=od and mops to S&iffers$*iff' Topic' +enerating ne& products%%CS0' Reflective Thinking," ective' .,6-1

A) 3hat is #ualit function deplo ment KDF*)4%ns&er' DF* refers to determining &hat &ill satisf the customer and translating thosecustomer desires into the target design$*iff' 1Topic' =roduct development," ective' .,6-2

2<) >dentif the steps involved in "uilding the House of Dualit $1-1?6

Cop right ; <11 =earson ducation >nc$

Page 176: OM Exam 1 Chap 1-5

8/13/2019 OM Exam 1 Chap 1-5

http://slidepdf.com/reader/full/om-exam-1-chap-1-5 176/187

%ns&er' >dentif customersB &ants identif product/service attri"utes relate the customersB&ants to the product/service ho&Bs conduct an evaluation of competing products develop

performance specifications for product or service ho&Bs and assign ho&Bs to the appropriate place in the transformation process$*iff'

Topic' =roduct development," ective' .,6-2

21) 7plain the difference "et&een value anal sis and value engineering$%ns&er' Ealue engineering is concerned &ith reducing cost and improving function in a

preproduction settingG value anal sis &ith similar aims takes place during production &hen the product has sho&n that it &ill succeed$ Techni#ues are similar$*iff' Topic' =roduct development and >ssues for product design," ective' .,6-

2 ) %ggressive ne& product development re#uires that organi!ations "uild structures internallthat contain &hat features4%ns&er' The should have open communication &ith customers innovative organi!ationalcultures aggressive RZ* strong leadership formal incentives and training$*iff' 2Topic' +enerating ne& products%%CS0' Communication," ective' .,6-

22) 3hat t&o issues should "e considered in com"ination in order to enhance the likelihood ofethical decision in the realm of product design4 3h is each important4%ns&er' The t&o issues are K1) vie& product design from a 8s stems8 perspective and K )consider the life c cle of the product$ The first issue causes managers to go "e ond 8&hat is "estor cheapest for the firm48 The second causes managers to address environmental concerns overthe life of the product not ust at time of manufacture$*iff' Topic' >ssues for product design%%CS0' thical Reasoning," ective' .,6-

25) >dentif the specific guidelines that can help an operations manager achieve environmentallfriendl designs$%ns&er' Specificall the follo&ing ideas are presented in the te7t' make products rec cla"leuse rec cled material use less harmful ingredients use lighter components use less energ anduse less material$*iff' Topic' >ssues for product design%%CS0' thical Reasoning," ective' .,6-

26) 3hat are some of the "enefits from using environmental teams for product design4%ns&er' nvironmental teams help develop safe and more environmentall sound products

1-1?9Cop right ; <11 =earson ducation >nc$

Page 177: OM Exam 1 Chap 1-5

8/13/2019 OM Exam 1 Chap 1-5

http://slidepdf.com/reader/full/om-exam-1-chap-1-5 177/187

minimi!e the &aste of ra& materials and energ differentiate products from competition reduceenvironmental lia"ilities increase cost-effectiveness of compl ing &ith environmentalregulations and help the organi!ation "e recogni!ed as a good corporate citi!en$*iff' Topic' >ssues for product design

%%CS0' thical Reasoning," ective' .,6-

29) *iscuss the advisa"ilit of using modular assem"lies in manufacturing$ K3hat are theadvantages and disadvantages4) To &hat e7tent can these arguments "e applied to service

products4%ns&er' :odules are easil segmented componentsG the add fle7i"ilit to production andmarketingG allo&s mi7-and-match of components Kcustomi!ation at point of customer contact)$(se of modules usuall means fe&er parts less design and tooling e7pense$ *isadvantagesinclude using a module in a product for &hich a more specific component &ould have "een

"etter$ :odules e7ist in services as in fast-food meals "uilt to customer specification$

*iff' 2Topic' >ssues for product design," ective' .,6-

2?) 3hat is time-"ased competition4%ns&er' Time-"ased competition is competition that is "ased on time$ >t involves rapiddevelopment of products fast introduction to the marketplace$ ,ften the first compan into

production ma have its product adopted as the 8standard$8*iff' Topic' Time-"ased competition," ective' .,6-5

2@) State the "enefits of implementing group technolog $%ns&er' 0enefits include improved designG reduced ra& materials and purchasesG simplified

production planning and controlG improved routing and machine loadingG reduced tooling setuptime &ork-in-process and production timeG and development of &ork cells$*iff' Topic' *efining the product," ective' .,6-6

1-1??Cop right ; <11 =earson ducation >nc$

Page 178: OM Exam 1 Chap 1-5

8/13/2019 OM Exam 1 Chap 1-5

http://slidepdf.com/reader/full/om-exam-1-chap-1-5 178/187

2A) >dentif the e7ternal product development strategiesG descri"e each in a sentence or t&o$%ns&er' The e7ternal product development strategies include alliances oint ventures and

purchase of technolog or e7pertise " ac#uiring the developer$ %lliances are cooperativeagreements that allo& firms to remain independent "ut use complementing strengths to pursuestrategies consistent &ith their individual missions$ Joint ventures are com"ined o&nership to

pursue ne& products or markets$ =urchasing technolog or e7pertise is usuall accomplished "ac#uiring entrepreneurial firms that have alread developed the technolog that fits the mission$The issue then "ecomes fitting the purchased organi!ation its technolog and its product lineinto the "u ing firm rather than a product development issue$*iff' Topic' Time-"ased competition," ective' .,6-5

5<) Ho& does configuration management manifest itself &hen ou ask for service on ourautomo"ile4%ns&er' Configuration management is used " ever automo"ile manufacturer to track all of the

changes "et&een and during a model ear$ To "e specific a"out the part that is needed the E>IKvehicle identification num"er) is often used$*iff' Topic' *ocuments for production," ective' .,6-9

51) 0riefl e7plain ho& =roduct .ifec cle :anagement K=.:) impacts product design$%ns&er' =.: is an um"rella of soft&are programs that attempts to "ring together man phasesof product design and manufacture$ =.: products often start &ith product design KC%*/C%:)Gmove on to design for manufacture and assem"l K*F:%)G and then into product routingmaterials la out assem"l maintenance and even environmental issues$*iff' Topic' *ocuments for production," ective' .,6-9

5 ) >dentif the four methods of service design that can reduce costs and enhance the product$%ns&er' Customi!ing as late in the process as possi"leG modulari!ing the productG identif ingthe service parts that lend themselves to automation or reduced customer interactionG andfocusing design on the moment of truth$*iff' Topic' Service design," ective' .,6-?

1-1?@Cop right ; <11 =earson ducation >nc$

Page 179: OM Exam 1 Chap 1-5

8/13/2019 OM Exam 1 Chap 1-5

http://slidepdf.com/reader/full/om-exam-1-chap-1-5 179/187

Page 180: OM Exam 1 Chap 1-5

8/13/2019 OM Exam 1 Chap 1-5

http://slidepdf.com/reader/full/om-exam-1-chap-1-5 180/187

Page 181: OM Exam 1 Chap 1-5

8/13/2019 OM Exam 1 Chap 1-5

http://slidepdf.com/reader/full/om-exam-1-chap-1-5 181/187

%ns&er' =rocess % promises the highest profit &ith an :E of L22< <<<$

*iff' Topic' %pplication of decision trees to product design%%CS0' %nal tic Skills

," ective' .,6-@

1-1@1Cop right ; <11 =earson ducation >nc$

Page 182: OM Exam 1 Chap 1-5

8/13/2019 OM Exam 1 Chap 1-5

http://slidepdf.com/reader/full/om-exam-1-chap-1-5 182/187

Page 183: OM Exam 1 Chap 1-5

8/13/2019 OM Exam 1 Chap 1-5

http://slidepdf.com/reader/full/om-exam-1-chap-1-5 183/187

5) % s&im clu" is designing a ne& pool to replace its old pool$ The ne& pool &ould need to lastfor 1< ears since the clu" is planning on relocating after 1< ears$ % concrete shell &ould costL1<< <<< and last for all 1< ears$ %nother option is to install a vin l liner that &ould cost onlL?6 <<< to install$ Ho&ever the vin l is not guaranteed to last for all 1< ears and has a 22chance of "reaking do&n$ Repair of the vin l &ould cost L5< <<< and &ould e7tend the life of

the vin l liner to the 1< ear mark$ >f "oth options are accepta"le to the s&im clu" &hich oneminimi!es their cost4%ns&er'

:EKin thousands) of vin l is $22K116)Q$9?K?6) O L@@$:E Kin thousands) of concrete is L1<<The clu" should install the vin l option to minimi!e cost

*iff' Topic' %pplication of decision trees to product design%%CS0' %nal tic Skills," ective' .,6-@

1-1@2Cop right ; <11 =earson ducation >nc$

Page 184: OM Exam 1 Chap 1-5

8/13/2019 OM Exam 1 Chap 1-5

http://slidepdf.com/reader/full/om-exam-1-chap-1-5 184/187

6) % compan producing apps for a social net&orking site is deciding &hich path to pursue$ Thefirst is to create an app that has universal appeal "ut faces a cro&ded market$ This app % &ouldhave sales of 1<< <<< copies at L1 each under ideal conditions "ut under tough conditions &ouldhave sales of onl 9< <<< copies at L$@< each$ The other app 0 &ould have sales of 6<< <<<units at L$6< each under ideal conditions "ut sales &ould "e reduced to 1< <<< units at L$6< each

under tough conditions$ >f ideal and rough conditions occur &ith the same fre#uenc &hich appshould the compan produce4 Iote- "oth apps cost the same to develop$%ns&er'

:EKin thousands) of %pp % is $6K1<<)Q$6K5@) O L?5:EKin thousands) of %pp 0 is $6K 6<)Q$6K6) O L1 ?$6The compan should pursue %pp 0 to ma7imi!e e7pected revenue$*iff' Topic' %pplication of decision trees to product design%%CS0' %nal tic Skills

," ective' .,6-@

1-1@5Cop right ; <11 =earson ducation >nc$

Page 185: OM Exam 1 Chap 1-5

8/13/2019 OM Exam 1 Chap 1-5

http://slidepdf.com/reader/full/om-exam-1-chap-1-5 185/187

9) % compan is deciding if it should design an advertising s stem for use on T&itter;$ Thefirst option is to skip out on designing &ith no net costs or gains$ The second option is S stem% &hich &ould result in additional sales of either L6< <<< under good conditions or L1< <<<under "ad conditions$ The final choice is S stem 0 &hich &ould increase sales " L < <<<under "oth good and "ad conditions$ Suppose that good conditions are t&ice as likel as "ad

conditions$ 3hich option should the compan pursue if developing a s stem costs L 6 <<<4%ns&er'

:E of S stem % is /2K 6<<<)QK-16 <<<)/2OL11999$9?:E of S stem 0 is -L6 <<<:E of nothing is L<$Thus the compan should develop s stem %$*iff' Topic' %pplication of decision trees to product design%%CS0' %nal tic Skills

," ective' .,6-@

1-1@6Cop right ; <11 =earson ducation >nc$

Page 186: OM Exam 1 Chap 1-5

8/13/2019 OM Exam 1 Chap 1-5

http://slidepdf.com/reader/full/om-exam-1-chap-1-5 186/187

?) % compan looking for venture capitalist funding is deciding on the design of its operatings stem for its ne& phone$ The first option is to simpl "u the ,S from another compan $ This&ould result in sales of either 1< <<< units if the market is not cro&ded &ith similar phones orsales of onl 2 <<< units if the market is cro&ded$ >f the compan decides to design its o&n ,Sthe phone &ould have sales of ?< <<< units if the ,S &as popular "ut sales of onl <<< if the

,S &as a failure$ Suppose that to recoup the cost of designing their o&n ,S the compan &ouldneed to sell t&ice as man phones as &hen the simpl "u the ,S for the profit from thescenarios to "e e#ual$ 3hich option should the compan choose if the pro"a"ilit that themarket is/ is not cro&ded is 6< and the pro"a"ilit that the ,S is popular is ?6 4%ns&er'

:E of "u ,S is $6K2<<<)Q$6K1<<<<)O96<< units:E of design ,S is $?6K?<<<<)Q$ 6K <<<)O62<<< unitsHo&ever the firm needs to sell t&ice as man designed ,S phones as "ought ,S phones fore#ual profit so that means 96<< "ought ,S phones are &orth 12<<< designed ,S phones$ Since12 <<< is less than the 62 <<< e7pected sales from the designed ,S the compan should designtheir o&n ,S$*iff'

Topic' %pplication of decision trees to product design%%CS0' %nal tic Skills," ective' .,6-@

1-1@9Cop right ; <11 =earson ducation >nc$

Page 187: OM Exam 1 Chap 1-5

8/13/2019 OM Exam 1 Chap 1-5

http://slidepdf.com/reader/full/om-exam-1-chap-1-5 187/187

@) % compan is deciding if it should do design of a product in-house or outsource the design$ >foutsourced the cost for a lo& "idder &ould "e L5< <<< and the cost for an e7pensive "idderL1<< <<<$ To do the design in-house &ould re#uire over-time for an alread stretched designteam &ith total costs of L?< <<<$ >f all three designs Kin-house lo& "id high "id) are e#ual in#ualit and the likelihood of finding a lo& "idder is 9< Kmeaning that 5< of the time the firm

&ill have to hire the e7pensive "idder) should the compan outsource design or do it in-house4%ns&er'

:E of outsource is $5K1<< <<<)Q$9K5< <<<)OL95 <<<